*Tiffeny's Comprehensive AANP Review (LEIK/Barkley/Fitzgerald) pt 2*.

Pataasin ang iyong marka sa homework at exams ngayon gamit ang Quizwiz!

Pregnancy Induced Hypertension (PIH)

*BP: >140/90 or >30/15 over baseline with readings 6hrs apart* RF: HTN, DM, multiple gestation, primigravida, very young, >35 Dx: monitor BPs, baseline CBC, LFTs, US, nonstress test, fetal kick counts Tx: bedrest in *left lateral* position

Physical exam with epididymitis?

*Enlarged, tender epididymis* Urethral discharge (can be bloody) *Positive Prehn's sign (lifting the testicle)

Take meglitinides ______ to _______ minutes prior to meals

1 to 30

You examine a 32-year-old man with chancroid and anticipate finding: A. a verruciform lesion. B. a painful ulcer. C. a painless, crater-like lesion. D. a plaquelike lesion.

B. a painful ulcer.

An adverse effect associated with the use of glucosamine is: A. elevated ALT and AST. B. bronchospasm. C. increased bleeding risk. D. QT prolongation.

B. bronchospasm.`

You see a 28-year-old man who is having an anaphylactic reaction following a bee sting and is experiencing trouble breathing. Your initial response is to administer: A. oral antihistamine B. injectable epinephrine C. supplemental oxygen D. vasopressor therapy

B. injectable epinephrine

Therapy for acne vulgaris that is cystic and unresponsive to conventional therapy. A. oral antibiotics. B. isotretinoin. C. benzoyl peroxide. D. hydrocortisone cream

B. isotretinoin (Accutane)

The use of lindane (Kwell) to treat scabies is discouraged because of its potential for: A. hepatotoxicity B. neurotoxicity C. nephrotoxicity D. pancreatitis

B. neurotoxicity

Common clinical conditions included in the atopy subgroup of type I hypersensitivity reactions include all of the following except: A. allergic rhinitis B. rosacea C. atopic dermatitis D. allergic gastroenteropathy

B. rosacea

The use of all of the following medications can trigger gout except: A. aspirin. B. statins. C. diuretics. D. niacin.

B. statins.

underweight is defined as:

BMI <18 or 15-20% below ideal weight pg 835

During an IBD flare, serological markers of inflammation, including ___________ and _____________ are usually elevated. _____________ is usually present

C-reactive protein Erythrocyte sedimentation rate Leukocytosis

The most likely causative pathogen in a 23-year-old woman with PID is: A. Escherichia coli. B. Enterobacteriaceae. C. C. trachomatis. D. Pseudomonas.

C. C. trachomatis.

Treatment of pseudogout can include all of the following except: A. NSAIDs. B. colchicine. C. allopurinol. D. oral corticosteroids.

C. allopurinol.

First-line therapy for acne vulgaris with closed comedones includes: A. oral antibiotics. B. isotretinoin. C. benzoyl peroxide. D. hydrocortisone cream

C. benzoyl peroxide.

Which of the following is the most potent risk factor for lower-extremity vascular occlusive disease? A. hypertension B. older age C. cigarette smoking D. leg injury

C. cigarette smoking

The zoster vaccine (Zostavax) is a(n): A. inactivated/killed virus vaccine B. conjugate vaccine containing a virus-like particle (VLP) C. live, attenuated vaccine D. inactivated toxin vaccine

C. live, attenuated vaccine

Treatment options for venous stasis ulcers in the lower extremities include: A. cleansing with hydrogen peroxide B. applying Burow solution C. prescribing a systemic corticosteroid D. applying a moisture- retaining dressing.

D. applying a moisture-retaining dressing.

Clinical manifestations of vitamin D deficiency include all of the following except: A. rickets. B. osteomalacia. C. antigravity muscle weakness. D. azotemia.

D. azotemia.

Where are Heberneds nodes?

DIPs

Galeazzi's sign evaluates

Developmental dysplasia of the hip (DDH) + sign= unequal knee height Test not helpful if DDH is bilateral

Anti-HCV(-)

Does not have HepC

In men men, presentation typically includes dysuria with a milky, occasionally blood-tinged penile discharge. Most men are asymptomatic,

GONORRHEA

Where does ulcerative colitis most often occur?

In the rectum and sigmoid colon The mucosa is thinner and has a dark red and velvety appearance

Oligomenorrhea

Infrequent Irregular > 40 days

First-line therapy for treating patients with acute gouty arthritis usually includes:

Naproxen sodium

How to estimate a due date

Negelli's Rule: LMP + 9 mo + 7 days

What anemia would you see in RA?

Normocytic Normochromic

Emergency contraception options

OCs (Yuzpe Method) progestin only (Plan B, Next Choice) -not as effective if obese ulipristal acetate progesterone (Ella) Copper IUD (Paraguard)

Carefully Monitor patients on DDP-4 inhibitors (gliptins - januvia) for __________ after initiation or dose increases

Pancreatitis

Which disease can Carpal Tunnel Sydrome also be a part of? (4)

RA Sarcoidosis pregnancy Untreated hypothyroid

What is used when assessing the severity of pancreatitis? What is a significant finding?

Ranson criteria

Anti-HBc, Anti-HBsAG

Recovered Hep B

begins with vague colicky umbilical pain, after several hours pain shifts to RLQ n with 1-2 episodes of vomiting, pain worsened and localized with coughing.

Sx of appendicitis

What do you do if two pills is missed?

Take two today, take two tomorrow Use back up

T/F: Women < 35 years who have migraine without aura, no other risk factors for stroke, low dose estrogen ≤ 50 mcg OK (ACOG); (WHO: ≤ 35 mcg estrogen is ok)

True

T/F: Women > 35 years who have migraine + OC; risks outweigh benefits

True

Crohn's or UC Antibiotics discouraged colorectal cancer risk rectally administered medications such as mesalamine changed noted in the colon mucosa inflammation

US

E. coli most common in women Proteus most common in men Dysuria is key sxs Hematuria in 40-60% of patients UA shows >10 WBC Nitrates on dipstick-very specific, not sensitive Esterase on dipstick-very sensitive, not specific

UTI

A hernia that has caused a bowel obstruction as a result of the protrusion

What is an incarcerated hernia?

an example of a drug that targets the renin-angiotensin-aldosterone system is: a. an ACE inhibitor b. a beta blocker c. a calcium channel blocker d. a diuretic

a. an ACE inhibitor

possible consequences of excessive levoythyroxine use include: a. bone thinning b. fatigue c. renal impairment d. constipation

a. bone thinning

in elderly females, which screening test has demonstrated greatest reduction in mortality from cancer? a. breast b. cervical c. ovarian d. lung

a. breast

a male with gonorrhea might c/o: a. dysuria b. a penile lesion c. abdominal pain d. fatigue

a. dysuria

Which of the following is essential for diagnosing thyroid cancer? a. fine needle aspiration b. thyroid ultrasound c. computed tomography (CT) scan d. magnetic resonance imaging (MRI)

a. fine needle aspiration

possible adverse outcomes from heatstroke include: a. rhabdomyolysis b. anemia c. hypernatremia d. leukopenia

a. rhabdomyolysis

in the evaluation of acute diverticulitis, the most appropriate diagnostic approach to r/o free air in the abdomen includes: a. barium enema b. plain abdominal film c. abdominal ultrasound d. lower endoscopy

b. plain abdominal film

monitoring for hepatoma in a pt with chronic hepatitis B or C often includes periodic evaluation of: a. erythrocyte sedimentation rate b. HBsAb c. alpha-fetoprotein d. bilirubin

c. alpha-fetoprotein

the most common type of genitourinary dysfunction after a transurethral resection of the prostate is: a. erectile dysfunction b. urinary incontinence c. retrograde ejaculation d. decreased libido

c. retrograde ejaculation

The preferred treatment for uncomplicated gonococcal proctitis is:

ceftriaxone 250 mg IM as a single dose plus a single dose of azithromycin 1 g po.

a localized tumor in the prostate gland associated with early stage prostate cancer is likely to produce: a. urinary hesitancy b. low back pain'c. urinary frequency d. an absence of sxs

d. an absence of sxs

a 55 y/o male pt describes severe pain at the base of his lft first toe. he is limping and says he can't remember hurting his toe. which sx below suggests something other than gout? a. pain b. ESR c. erythema d. fever

d. fever

antiviral txment for chronic HBV infection includes all of the following EXCEPT: a. entecavir b. tenofovir c. lamivudine d. fidaxomicin

d. fidaxomicin

when answering questions about hepatitis A vaccine, you consider that all of the following are true EXCEPT: a. it does not contain live virus b. it should be offered to individuals who frequently travel to developing countries c. it is a recommended immunization for healthcare workers d. it is given as a single dose

d. it is given as a single dose

which of the following medications is a PPI? a. loperamide b. metoclopramide c. nizatidine d. lansoprazole

d. lansoprazole

which of the following SSRIs is associated with the greatest anticholinergic effect? a. fluvoxamine b. sertraline c. fluoxetine d. paroxetine

d. paroxetine

hyperbilirubinemia can cause all of the following EXCEPT: a. potential displacement of highly protein-bound drugs b. scleral icterus c. cola-colored urine d. reduction in urobilinogen

d. reduction in urobilinogen

Signs of too much estrogen and not enough progestin combo

heavy bleeding dysmennorhea headaches irritability bloating syncope

Graves disease treatment places individual at risk for ______ toxicity

hepato methimazole or propylthiouracil

Risk Factors for Sarcoidosis

higher incidence in women than men Family History African Ancestry Ages 20 - 40

Hyperthyroidism, Hypothyroidism or Both -menorrhagia

hypo

The nurse knows that normal splitting of the second heart sound is associated with:

inspiration

posterior fontanel should be closed by

2-3 months

How long after contact do clinical manifestations of syphilis typically occur?

2-4wks

kerning's sign causes pain on extension and is found in what

meningitis

precontemplation

non interested in change

Sx of Carpal Tunnel

numbness, tingling, burning,nocturnal pain

Name Triptans

o Sumatriptan (Imitrex, Treximet) o Eletriptan (Relpax) o Almotriptan (Axert) o Rizatriptan (Maxalt®, Maxalt® MLT) o Zolmitriptan (Zomig®) o Several other

A 36 year old female has migraine headaches. Which medication could be used as an abortive agent?? 1. Feverfew 2. Sumatriptan 3. Propanolol 4. Prednisone

2. Sumatriptan

FSH findings during peri and true menopause

peri-menopause: FSH > 20IU/mL menopause: FSH >30IU/mL

Change Theory

ready to move forward with change

When to do triple or quad screen for Down's

20wks

ED medication: 30min, lasts 4 hours, 50-100mg

sildenafil (Viagra)

PE diagnosis goal standard

spiral CT

ADA recommends periodic fasting plasm glucose screening every ____ in ALL adults regardless of appearance of risk

3 year

lateral epicondylitis

tennis elbow- repetitive lifting

What non-narcotic can be taken short term for MSK pain?

tramadol

T/F: Thiazides can precipitate gout in someone with a gout genetic predisposition.

true

T/F: When ordering laboratory tests to confirm chancroid, the NP considers that concomitant infection with herpes simplex is often found.

true

What are less common risk factors? (3)

use of opioids, corticosteroids, thiazides viral infection blunt abdominal trauma

ED medication: 60min, lasts 4 hours, 10-20mg

vardenafil (Levitra)

What med can be used in hepatitis for prolonged PT (>15 seconds)?

vitamin k

When to do a Group B beta-hemolytic strep genital culture?

34-36 weeks

Motor nerve supplying the sternocleidomastoid and trapezius muscles.

• CN XI: Spinal Accessory

Health Promotion Migraine

• Prophylactic treatment for > 2 per month • Early treatment and diagnosis • Limit use of triptans (overuse leads to rebound headaches unresponsive to triptans)

AANP young gerontologicals are those aged

65-84

First-line therapy for prepatellar bursitis should include: A. bursal aspiration. B. intrabursal corticosteroid injection. C. acetaminophen. D. knee splinting.

A. bursal aspiration

A significant rabies risk is associated with a bite from all of the following except: A. humans B. foxes C. bats D. skunks

A. humans

Which of the following represents the most effective method of cancer screening? A. skin examination B. stool for occult blood C. pelvic examination D. chest radiography

A. skin examination

An urticarial lesion is usually described as a: A. wheal B. plaque C. patch D. papule

A. wheal

During a health history, a patient tells the nurse that he has trouble in starting his urine stream. This problem is known as:

ANS: hesitancy. Hesitancy is trouble in starting the urine stream. Urgency is the feeling that one cannot wait to urinate. Dribbling is the lost of urine before or after the main act of urination. Frequency is urinating more often than usual.

The nurse knows that a common assessment finding in a boy younger than 2 years old is:

ANS: the presence of a hydrocele, or fluid in the scrotum. A common scrotal finding in boys younger than 2 years of age is a hydrocele, or fluid in the scrotum. The other options are not correct.

Danger Signals GI (3)

Acute abd (sx), Acute appendicitis and pancreatitis

Clinical conditions with a presentation similar to acute bursitis include: (More than one option can apply.) A. rheumatoid arthritis. B. septic arthritis. C. joint trauma. D. pseudogout.

All

Pharm management for BPH?

Alpha-blockers -osin suffix: Terazosin Prazosin tamsulosin

Screening for Genetic Disorders **Ashkenazi Jews***, French Canadian, Cajun ancestry Test Ideally prior to pregnancy; pregnancy and oral contraceptive use increase risk of false positive test results

(Tay-Sachs disease) Tay-Sachs disease is a rare inherited disorder that progressively destroys nerve cells (neurons) in the brain and spinal cord. ■ Tay-Sachs disease is a fatal neurological disease with no known cure. It is more common among Eastern Europeans of Jewish-descent (Ashkenazi Jews).

Signs of deficient estrogen in BCP

(menopause sx) hot flashes night sweats depression irritability weight gain

Signs of excessive estrogen in BCP

(pregnancy sx) dysmennorhea nausea cholasma (mask-like) cardioembolic events telangiactasias breast changes

**LEIK** [Serotonin Syndrome] Result of high serotonin levels Acute onset; rapid progression [S/S] sudden high fever muscular rigidity LOC change hyperreflexia/clonus uncontrolled shivering Potentially life threatening When switching/adding drugs that can affect serotonin, wait 2 weeks

**LEIK** [Serotonin Syndrome]

Supportive care for OA?

*Weight loss* Canes Ice/Moist heat PT Joint replacement

Complications of celiac disease (5)

- malnutrition (Fe, Ca, fat soluble vitamins) -osteopenia & osteoporosis - lactose indigestion - intestinal cancers - refractory sprue (severe villous atrophy)

Concerning Legg Calve Perthes Disease, when do you just observe?

-Full ROM is preserved. -Child is less than 6 -Less than 1/2 of femoral head involved

Danger Signals of Pulmonary System (3)

-anaphylaxis, severe asthmatic exacerbation,PE

Danger Signal Men's Health (2)

-testicular torsion, priapism

Anti-thyroid medication such as propylthiouracil (PTU) and methimazole (Tapazole) both carry with them black box warnings for acute _____________ failure!

...

CLASSIC CASE:

...

Naegele's rule _____ year - _______ months + ______ days from _______

1 year -3 months + 7days from LMP

What are possible causes of secondary dysmenorrhea? Select all that apply. 1. Endometriosis 2. Pelvic infection (PID) 3. Fibroids 4. Urinary tract infection

1. Endometriosis 2. Pelvic infection (PID) 3. Fibroids

Calculation for levothyroxine replacement....

1.6mcg/kg/day, based on ideal body weight, unless the patient is under weight, than you use actual weight Fitzgerald pg 285 4th edition.

Short acting rapid onset insulin (aspart insulin aka novolog) onset in ___________minutes and peak @ ___________ hours and last for ___________ hours

10-20 minutes 1-3 hours 3-5 hours

Normal fasting glucose Is < _____ goal is ____ to ____

100 70 - 130

NPH (Novolin N) (Humulin N) *intermittent acting* duration: ?

16-24 hr

how long does it take for the medications

2-8 wks

What cranial nerve is responsible for vision? 1. CN II 2. CN III 3. CN IV 4. CN V

2. CN III

A 16 year old female was at school this morning when she states that she saw zigzagging lines in both eyes. This lasted for about 30 minutes. It made her nauseated and now she complains of a headache over her right eyebrow. She presents to your clinic at this time. Her neuro exam is normal. What symptom might be unusual? 1. Pounding headache 2. Photophobia 3. Phonophobia 4. Teary, red eye

4. Teary, red eye

Nomal PSA?

<4.0

What is primary biliary cirrhosis?

A disease that almost exclusively affects women aged 40-60 Autoimmune disease that causes destruction of the intra-hepatic bile ducts, resulting in cholestasis

What is the strangulated hernia?

A non-reducible herniation in which the blood supply to the herniated tissue is compromised.

A burn that is about twice as large as an adult's palmar surface of the hand including the fingers encompasses aBSA of approximately ____%. A. 1 B. 2 C. 3 D. 4term-1873

A. 1

Which of the following statements is most accurate regarding cellulitis? A. Insect bites, abrasion, or other skin trauma can be the origin of cellulitis. B. Cellulitis most often occurs on the chest and abdomen. C. Necrosis is a common complication of cellulitis. D. Cellulitis often occurs spontaneously without any identifiable skin wound

A. Insect bites, abrasion, or other skin trauma can be the origin of cellulitis

Skin lesions associated with actinic keratoses can be described as: A. a slightly rough, pink or flesh-colored lesion in a sun- exposed area. B. a well-defined, slightly raised, red, scaly plaque in a skinfold. C. a blistering lesion along a dermatome. D. a crusting lesion along flexor aspects of the finger

A. a slightly rough, pink or flesh-colored lesion in a sun-exposed area

In a 13-year-old female patient with mild acne and who experiences an inadequate response to benzoyl peroxide treatment, an appropriate treatment option would be to: A. add a topical retinoid. B. add an oral antibiotic. C. consider isotretinoin. D. consider hormonal therapy.

A. add a topical retinoid.

Which of the following medications can contribute to the development of acute urinary retention in an older man with BPH? A. amitriptyline B. loratadine C. enalapril D. lorazepam

A. amitriptyline

All of the following are associated with secondary Raynaud phenomenon except: A. hypertension B. scleroderma C. repeated use of vibrating tools. D. use of beta-adrenergic antagonists.

A. hypertension

Treatment options in postherpetic neuralgia include all of the following except: A. injectable methylprednisolone B. oral pregabalin C. oral nortriptyline D. topical lidocaine

A. injectable methylprednisolone

The use of which of the following medications contributes to the development of acne vulgaris? Select all that apply. A. lithium B. propranolol C. sertraline D. clonidine E. phenytoin

A. lithium E. phenytoin

Which of the following is a common finding in a man with varicocele? A. lower sperm count with increased number of abnormal forms B. increased rate of testicular cancer C. recurrent scrotal pain D. BPH

A. lower sperm count with increased number of abnormal forms

Compared with unfractionated heparin, characteristics of low-molecular-weight heparin (LMWH) include all of the following except: A. more antiplatelet effect B. decreased need for monitoring of anticoagulant effect C. longer half-life D. superior bioavailability

A. more antiplatelet effect

Patients with rosacea are recommended to use daily: A. sunscreen B. astringents C. exfoliant D. antimicrobial cream

A. sunscreen

Patients with olecranon bursitis typically present with: A. swelling and redness over the affected area. B. limited elbow range of motion (ROM). C. nerve impingement. D. destruction of the joint space.

A. swelling and redness over the affected area.

A 22-year-old woman taking a 35-mcg ethinyl estradiol COC calls after forgetting to take her pills for 2 consecutive days. She is 2 weeks into the pack. You advise her to: A. take the last pill missed immediately, even if this means taking 2 pills today. B. discard two pills and take two pills today C. discard the rest of the pack and start a new pack with the first day of her next menses. D. continue taking one pill daily for the rest of the cycle.

A. take the last pill missed immediately, even if this means taking 2 pills today.

When the nurse is performing a testicular examination on a 25-year-old man, which of these findings is considered normal?

ANS: Testes that feel oval and movable and are slightly sensitive to compression Testes normally feel oval, firm and rubbery, smooth, and equal bilaterally and are freely movable and slightly tender to moderate pressure. The scrotal skin should not be dry, scaly, or nodular or contain subcutaneous plaques. Any mass would be an abnormal finding.

A newborn baby boy is about to have a circumcision. The nurse knows that indications for circumcision include:

ANS: cultural and religious beliefs. Indications for circumcision include cultural and religious beliefs, prevention of phimosis and inflammation of the glans penis and foreskin, decreasing the incidence of cancer of the penis, and decreasing the incidence of urinary tract infections in infancy.

A male patient with possible fertility problems asks the nurse where sperm is produced. The nurse knows that sperm production occurs in the:

ANS: testes. Sperm production occurs in the testes, not in the other structures listed.

What do these labs indicate? Anti-HAV, IgM

Active Hep A

What do these labs indicate? Anti-HCV, HCV RNA

Acute and/or chronic HCV

Rare genetic defect causing severe lung damage at early age; Alpha-1 protects lungs from oxidative and environmental damage.

Alpha-1 trypsin deficiency

Rare genetic defect causing severe lung damage at early age; Alpha-1 protects lungs from oxidative and environmental damage. (i.e COPD)

Alpha-1 trypsin deficiency

secretes FSH, LH, TSH, ACTH, Prolactin

Anterior pituitary gland

The daily amount of vitamin D3 recommended for pregnant or lactating women is: A. 300 IU. B. 600 IU. C. 1000 IU. D. 1200 IU.

B. 600 IU.

You examine a patient with a red, tender thermal burn that has excellent capillary refill involving the entire surface of the anterior right thigh. The estimated involved body surface area (BSA) is approximately: A. 5% B. 9% C. 13% D. 18%

B. 9%

Medications that are often helpful in relieving symptoms associated with Raynaud phenomenon include: A. (NSAIDs). B. ACEIs and CCBs C. beta-adrenergic antagonists. D. diuretics

B. ACEIs and CCBs

The most common method of preventing venous thromboembolism in higher risk surgical patients is the use of: A. vitamin K B. LMWH C. vena cava filter D. warfarin

B. LMWH i.e] enoxaparin (Lovenox) dalteprin (Fragmin) ardeparin (Normiflo)

Fibromyalgia is more common in patients with: A. type 2 diabetes. B. RA and SLE C. migraine headaches. D. COPD.

B. RA and SLE

You see a 33-year-old man diagnosed with reactive arthritis and urethritis. You recommend antimicrobial treatment with: A. amoxicillin. B. doxycycline. C. TMP-SMX. D. nitrofurantoin.

B. doxycycline.

When caring for an adult with an outbreak of shingles, you advise that: A. there is no known treatment for this condition B. during outbreaks, the chickenpox (varicella) virus is shed C. although they are acutely painful, the lesions heal well without scarring or lingering discomfort D. this condition commonly strikes young and old alike

B. during outbreaks, the chickenpox (varicella) virus is shed

Risk factors for malignant melanoma include: A. Asian ancestry B. history of blistering sunburn C. family history of psoriasis vulgaris D. presence of atopic dermatitis

B. history of blistering sunburn

A significant adverse effect of biologic therapy for treating RA is: A. myopathy. B. infections. C. renal impairment. D. elevated liver enzymes.

B. infections.

You examine a patient with psoriasis vulgaris and expect to find the following lesions: A. lichenified areas in flexor areas B. well-demarcated plaques on the knees C. greasy lesions throughout the scalp D. vesicular lesions over the upper thorax

B. well-demarcated plaques on the knee -most typically found on extensor surfaces (most commonly elbows and knees)

-reduces hepatic glucose production and intestinal absorption **insulin sensitizer** via increased peripheral glucose uptake and utilization A1C reduction = 1-2% with intensified use

BIGUANIDE

How does inflammation present with RA?

Bilateral, symmetric

Skin Lesion: Example(s) impetigo, 2nd degree burns, SJS lesions -superficial blister w/serous fluid >1cm

Bulla

Stage 3 Lyme disease, characterized by joint pain and neuropsychiatric symptoms, typically occurs how long after initial infection? A. 1 month B. 4 months C. 1 year D. 5 years

C. 1 year

The onset of anticoagulation effect of warfarin usually occurs how soon after the initiation of therapy? A. immediately B. 1 to 2 days C. 3 to 5 days D. 5 to 7 days

C. 3 to 5 days

Type I hypersensitivity reactions, such as atopic dermatitis, involve the action of which antibodies binding to receptor sites on mast cells? A. IgG B. IgM C. IgE D. IgA

C. IgE

Which of the following is not characteristic of rheumatoid arthritis (RA)? A. It is more common in women at a 3:1 ratio. B. Family hxof autoimmune conditions often is reported. C. Peak age for disease onset in individuals is age 50 to 70. D. Wrists, ankles, and toes often are involved.

C. Peak age for disease onset in individuals is age 50 to 70 years. Peak age is 20-40

Felicia is 18-weeks pregnant with her first child. Which of the following is most consistent with her gestational age? A. Uterine fundus palpable through the abdominal wall B. Nausea and breast tenderness C. Report of quickening D. Fetal heart tones detectable by abdominal Doppler

C. Report of quickening

One of the mechanisms of action of a topical corticos-teroid preparation is as: A. an antimitotic B. an exfoliant C. a vasoconstrictor D. a humectant

C. a vasoconstricto

An impetigo lesion that becomes deeply ulcerated is known as: A. cellulitis. B. erythema. C. ecthyma. D. empyema

C. ecthyma

During type I hypersensitivity reactions, histamine re-leased from degraded mast cells causes all of the following except: A. vasodilation B. mucous gland stimulation C. enhanced sebum production D. tissue swelling

C. enhanced sebum production

As part of the evaluation of patients with OA, the NP anticipates finding: A. anemia of chronic disease. B. elevated CRP level. C. no disease-specific laboratory abnormalities. D. elevated antinuclear antibody (ANA) titer.

C. no disease-specific laboratory abnormalities.

Pseudogout has been linked with abnormal activity of the: A. liver. B. kidneys. C. parathyroid. D. adrenal gland.

C. parathyroid.

What is the most effective protection against shingles? A. previous episode of chickenpox as a child B. prior episode of shingles C. receipt of varicella-zoster immunization D. avoiding children and daycare centers

C. receipt of varicella-zoster immunization

Hypothyroidism developing in infancy or early childhood

Cretinism Impaired development of the skeletal system and CNS, with severe mental retardation, short stature, coarse facial features, a protruding tongue, and umbilical herni

To transmit the bacterium that causes Lyme disease, an infected tick must feed on a human host for at least: A. 5 minutes B. 30 minutes C. 2 hours D. 24 hours

D. 24 hours

191. A child must consume ___ oz. of fortified milk each day to receive the recommended 400 IU daily of vitamin D. A. 8 B. 16 C. 32 D. 48

D. 48

Surgical intervention in BPH should be considered with all of the following except: A. recurrent urinary tract infection. B. bladder stones. C. persistent obstruction despite medical therapy. D. acute tubular necrosis.

D. acute tubular necrosis.

Which of the following is found in tertiary syphilis? A. arthralgia B. lymphadenopathy C. macular or papular lesions involving the palms and soles D. gumma

D. gumma

The most common locations for tophi include all of the following except: A. the auricles. B. the elbows. C. extensor surfaces of hands. D. the shoulders.

D. the shoulders.

What drugs are started as soon as the diagnosis of RA is made? Why? Major SE?

DMARDS help minimize the risk of joint damage and disease progression infection (need vaccination) i

Disadvantage with Depo?

Delayed fertility (12-18 mo) Injection Reduction in bone density (since estrogen=bone density)

What is the gold standard for diagnosing acute Hepatitis A?

Detection of IgM

S/S of excess estrogen?

Dysmenorrhea N/V Chloasma CVA-DVT-Thromboembolic events Telangiectasias Hepatic carcinoma Breast tenderness

Supportive care for epididymitis?

Elevate scrotum, NSAIDS, Ice then heat Bedrest

virchow triad's significance

Endothelial Damage, Stasis of blood and Hyper-coaguable state

DM Visits

Every visit: check BP, feet, weight, BMI, blood sugar diary Check feet: vibration sense w/ 128 tuning fork on MTP joint, light and deep touch, numbness w/ monofilament, pedal pulses, ankle reflexes, skin

True or False: All CA-MRSA strains are capable of causing necrotizing infection

False

complete ankle instability, significant swelling, and moderate to severe ecchymosis. sprain grade?

Grade III

What do these labs indicate? HbsAg-negative Anti-HBc-negative *Anti-HBs-postive*

Immune due to immunzation

Teaching for nuva ring: What to do if the nuva ring comes out?

Insert once monthly Leave in for 21 days Remove and bleed At day 28 replace What to do if the nuva ring comes out?

What needs to be done for repeated intercourse with the diaphragm?

Instill spermicide in the vagina-- *DO NOT take the diaphragm out to do it!

Laboratory evaluation of Addison's disease includes:

K, NA, Cortisol, and ACTH

Patient fell on his knees, what (3) test do you do?

Lachman (Drawer Test) McMurray Apley Grind Test

Drug induced thyroid disease are from: (4)

Lithium, amiodarone, interferon-alfa, dopamine

What DM Medication? [ACTION] non-sulfonylurea insulin secretagogue agent (enhances release) similar to sulfonylureas primarily work to lower blood sugar levels after meals, which, in turn, helps to lower A1C. fast acting w/short half-life A1C reduction of 0.5-1.5% w/ intensified use. [EFFECTIVENESS] ? [COST] ? [COMMON SE] -hypoglycemia -wt gain has been reported [SAFETY] -can cause severe hyperglycemia, especially in older adults repaglinide (Prandin) main -adjust dose w/renal impairment [CONTRAINDICATIONS] -none just use w/ caution is known sulfa allergy [ELIMINATION OF DRUG] -renally eliminated, hence the the need adjust dose based on renal impairment.

MEGLITINIDE ANALOGUE repaglinide (Prandin) nateglinide (Starlix)

Example of H. pylori antibiotic eradication regimen? (3)

MOC: Metro+omeprazole +clarithromycin AOC: Amox+omeprazole +clarithromycin MOA: Metro+omeprazole +amoxacillin

Skin Lesion: Example: freckle, lentigenes, small cherry angiomas -flat non palpable lesion <1cm diameter

Macule

What is seen on X-Ray in OA?

Narrowing of joint spaces, Osteophytes Juxta-articular sclerosis Subchondral bone

Is obesity a risk factor for osteoporosis

No

• Chronic, progressive, neurodegenerative disorder characterized by rest tremor, rigidity, bradykinesia and gait disturbance o 1-2/1000 people in North America o Mean age dx: 70.5 years

PARKINSON'S DISEASE

Which joints are affected most with RA?

PIP, MCP, WRIST

Cramping that radiates to LB and upper thighs Result of high levels of PG No evidence of pelvic abnormality. Leading cause of short-term recurrent school absenteeism in adolescent girls in the United States.

Primary Dysmenorrhea

(+) anti-HBc, anti-HBsAg

RECOVERED Hep B

Mgmt of abnormal pap

Refer if CIN II, III or CIS If ASCUS (hpv testing, repeat PAP, colposcopy)

What do alpha-blockers do? What to 5-Alpha-reductase inhibitors do? What is concerning with them? Examples of 5-Alpha-reductase inhibitors?

Relax muscles of the bladder Relax muscles of the bladder Finasteride Dutasteride

The direction of blood flow through the heart is best described by which of these?

Right atrium- right ventricle- pulmonary artery-lungs-pulmonary vein-left atrium-left ventricle

Hyper pigmentation of the buccal mucosa and skin creases, Diffuse tanning and freckles Orthostasis Hypotension Scant pubic and axillary hair

S/S of Addisons?

___________ burn -Blisters present -Surface is raw and moist

Second-degree

Painful menstruation usually related to pelvic pathology (PID, etc) Age > 20 Tx underlying etiology

Secondary Dysmenorrhea

DANGER SIGNS OF? BS <50 Passing out feeling, HA, clammy, anxiety Difficult to concentrate / think Progress to coma

Severe hypoglycemia danger signs

painless abnormal bleeding Pap shows "atypical glandular cells of undetermined significance" (AGCUS) Transvag US shows endometrial stripe >/= 10mm in postmenopausal women Any bleeding in postmenopausal women

Signs of endometrial cancer

Disadvantages of diaphragm use?

Skin/vaginal irritation and infections

How do you initiate basal insulin?

Start at 0.1-0.2 u/kg as a bolus at bedtime or given 10 units insulin glargine (Lantus) (Toujeo) insulin detemir (Levemir)

Diabetic Rx that cause weight gain

Sulonylureas, TZDs (Actos), insulin

How does Depo-Provera prevent pregnancy?

Suppresses FSH/LH=inhibits ovulation Thickens mucus Alters endometrium

(T/F) One of the potential serious adverse effects of unfractionated heparin is thrombocytopenia.

T

2nd line for depression

TCAs amitriptyline (Elavil) nortriptyline (Pamelor)

Treatment for Bullous impetigo caused by MRSA? (2 options)

TMP-SMX Clindamycin

Teaching with Misoprostol?

Take with food Stimulates mucus production Stimulates UC's/ abortion

Female - breast enlargement WITHOUT separate nipple contour Male - penis elongates

Tanner III

In PTSD, pt has For irritability and impulsitivity.... drug choice?

Tegretol

__________ burn -Minimal pain -Affected area surrounded by painful areas -Affected area white and leathery

Third-degree

DANGER SIGNS OF? -Single large nodule >2.5cm on one lobe -24 radioactive iodine uptake test = cold nodule -Hx of facial, neck, or chest radiation therapy

Thyroid Cancer

Dietary Management of DM1?

Total carbs: 50-60% Fats: 25% Fiber: 25 grams Protein:10-20%

- frothy "yellow-green" discharge, malodorous, pruritus, vaginal erythema - "strawberry patches" on cervix and vagina, dyspareunia, dysuria

Trichomonas (vulvovaginitis) MGMT: Flagyl 2g PO x1 OR 500 BID x7

Specific Headache Clues for: Headache with nausea, vomiting

Tumor

DANGER SIGNS OF? -School aged kid with recent hx of viral illness c/o excessive hunger / thirst / urinating more frequently -Loosing weight despite increase eating -Fruity odor breath -Large ketones in urine

Type 1 DM danger signs

Diagnostics with BPH?

UA, to rule out UTI Uroflowmetry Abdominal US Creatinine/BUN PSA >4

Concerning insulin administration, when is the individual at greatest risk for developing hypoglycemia? Likely to have a test question about this on boards

When the medication peaks. ** Remember Lantus does not peak, so if you see a ? on boards like this, select when the Lantus peaked.

When does urine leak with stress incontinence?

With increased pressure on bladder: Laugh Cough Exercise Sneezing

When should diaphragm be refitted?

With weight change of 20 lbs or more

Raised yellow-colored soft plaque that are located under the brow or upper and lower eyelids on the nasal side. It may be a sign of hyperlipidemia if present in persons under 40

Xanthelasma

drug class that limit the amount of uric acid the body produces.

Xanthin oxidase inhibitors (XOIs), allopurinol (Aloprim, Lopurin, Zyloprim) febuxostat (Uloric)

What is Al-Anon

a support group for an alcoholic's family and friend.

what is the common cause of an anal fisure? a. constipation b. recurrent or recent severe diarrhea c. recent childbirth d. anal intercourse

a. constipation

a pt has 2 non-fasting glucose values of 111 mg/dL and 124 mg/dL that were measured on 2 separate days in the same week. This pt: a. has normal blood glucose values b. has impaired fasting glucose c. should have further glucose testing done for dx d. should have a HgbA1c performed

a. has normal blood glucose values

early neurological changes in pts with lumbar radiculopathy include: a. loss of DTRs b. poor two-point discrimination c. reduced muscle strength d. footdrop

a. loss of DTRs

How do patients with pancreatic cancer present? (5)

abdominal pain weight loss anorexia nausea vomiting

positive obturator and psoas are (+) in ________ and indicate _________

acute appendicitis peritoneal inflammation

Male with BPH and hypertension: Start with

alpha-blocker (Hytrin) firs

what is the most common cause of increased neck size?

an enlarged thyroid gland pg 833

What causes Addison's disease

an inadequate amount of hormone production from the adrenal glands.

a 26 y/o has been dx with gonorrhea. How should he be managed? a. ceftriaxone only b. ceftriaxone and azithromycin c. cefixime and azithromycin d. Penicillin G

b. ceftriaxone and azithromycin

a 26 y/o male pt has been dx with gonorrhea. how should he be managed? a. ceftriaxone only b. ceftriaxone and azithromycin c. cifiixime and azithromycin d. penicillin G

b. ceftriaxone and azithromycin

a young athlete is found to have a depression of the longitudinal arch of both feet. he c/o of heel pain bilaterally. the rest of his foot is normal and he has continued with his activities. what could be recommended for his heel pain? a. an x-ray of the foot is needed first b. he needs some heel support in his shoes c. NSAIDs should be used initially d. rigid orthotics could be ordered for his shoes

b. he needs some heel support in his shoes

a 70 y/o male who is diabetic presents with gait difficulty, cognitive disturbance, and urinary incontinence. What is part of the NP's differential dx? a. diabetic neuropathy b. normal pressure hydrocephalus c. Parkinson's disease d. multiple sclerosis

b. normal pressure hydrocephalus

what joints are least commonly involved in osteoarthritis? a. fingers b. wrists c. hips d. knees

b. wrists

Avoid the use of _______ in the evaluation of acute diverticulitis

barium enema

What are the most common risks for pancreatitis? (3)

biliary tract disease (gallstones) excessive alcohol use elevated triglyceride levels

What psych medication is CI in anorexic/bulimic patient? why?

bupropion (Wellbutrin) Seizure threshold is increased.

when using trazodone to aid sleep, the drug should be optimally taken ___ prior to sleep a. immediately b. 15 min c. 1 hr d. 2 hrs

c. 1 hr

approximately what percentage of the population will develop diverticulosis by the time they reach 50 yrs of age? a. 10% b. 20% c. 33% d. 50%

c. 33%

with the straight-leg-raising test, the NP is evaluating tension on which of the following nerve roots? a. L1 and L2 b. L3 and L4 c. L5 and S1 d. S2 and S3

c. L5 and S1

which test is used to dx an Archilles tendon rupture? a. boutonniere test b. Lachman test c. Thompson test d. Drawer test

c. Thompson test

all of the following meds are recommended for tx of concomitant hypertension when seen with type 2 diabetes EXCEPT: a. BBs b. CCBS c. alpha adrenergic receptor antagonist (alpha blocker) d. ARBs

c. alpha adrenergic receptor antagonist "alpha blockers" Doxazosin (Cardura) Prazosin (Minipress) Terazosin.

which of the following is an appropriate tx for IBS? a. high fat, low residue diet b. high fiber, low fat diet and stress modification c. antispasmotics and loperamide for diarrhea predominance d. tricyclic antidepressants for constipation predominance

c. antispasmotics and loperamide for diarrhea predominance

the recommended time to initiate screening for cervical cancer in women is: a. prior to becoming sexually active b. at age 18 years c. at age 21 years d. 3 years after first sexual intercourse

c. at age 21 years

an older adult has osteopenia. her healthcare provider has recommended calicum 500mg TID. what is the most common side effect of calcium supplementation? a. stomach upset b. diarrhea c. constipation d. mild nausea intially

c. constipation

Cushing's syndrome results from an excess of: a. luteinizing hormone b. follicle stimulating hormone c. cortisol d. aldosterone

c. cortisol

metformin should be dc for the day of and up to 48 hrs after surgery because of increased risk of: a. hypoglycemia b. hepatic impairment c. lactic acidosis d. interaction with most anesthetic agents

c. lactic acidosis

routine testing for the presence of HBsAb after immunization with the HBV vaccine is recommended for all of the following EXCEPT: a. healthcare providers b. immunocompromised pts c. restaurant workers d. dialysis pts

c. restaurant workers

a 57 y/o male is in need of eval for Barrett esophagus. You recommend: a. H. pylori testing b. CT scan c. upper GI endoscopy with biopsy d. barium swallow

c. upper GI endoscopy with biopsy

elevated WBC, BILI, AST,ALT,LDH and amylase, U/S= gold standard most effective test

cholecystitis

Sequelae of acute HBV

chronic HBV, hepatocellular carcinoma (HCC, primary liver ca, hepatoma), hepatic failure or cirrhosis

at what age does the foreskin become fully retractable? a. 3 months b. 6 months c. 9 months d. 1 yr

d. 1 yr

a 64 y/o male with dxed Barrett esophagus has shown no sign of dysplasia in two consecutive evaluations within the past yr. You recommend additional surveillance testing should be conducted every: a. 6 months b. 12 months c. 2 yrs d. 3 yrs

d. 3 yrs

an elderly male has benign prostatic hyperplasia (BPH). what drug should be avoided in him? a. acetaminophen b. NSAIDs c. cortisone d. Hydrochlorothiazide (HCTZ)

d. Hydrochlorothiazide (HCTZ)

immune modulators are often used for intervention in: a. ulcerative colitis b. IBS c. Crohn's disease d. UC and Crohn's disease

d. UC and Crohn's disease

the use of a thiazolidinedione is not recommended in all of the following clinical scenarios EXCEPT: a. 57 y/o man who is taking a nitrate b. 62 y/o woman with heart failure c. a 45 y/o man who is using insulin d. a 35 y/o pt with newly dx type 2 diabetes

d. a 35 y/o pt with newly dx type 2 diabetes TZDs "glitazones" pioglitazone (Actos) rosiglitazone (Avandia) Remember they are contraindicated with: -nitrate use - insulin use -heart failure diagnosis They can cause or exacerbate heart failure

txment with venlafaxine (effexor) can lead to dose-dependent increases in: a. heart rate b. serum glucose c. AST/ALT d. blood pressure

d. blood pressure

a young male pt with a herniated disc reports bilateral sciatica and leg weakness. if he calls the NP with c/o urinary incontinence, what should be suspected? a. opioid overuse b. medial or lateral herniation c. rupture of the disc d. cauda equina syndrome

d. cauda equina syndrome

which dx is the least likely cause of extrinsic shoulder pain? a. angina b. hepatobiliary disease c. pneumonia d. gout

d. gout

depression often manifests with all of the following except: a. psychomotor retardation b. irritability c. palpitations d. increased feelings of guilt

d. increased feelings of guilt

peptic ulcer disease can occur in any of the following locations except: a. duodenum b. stomach c. esophagus d. large intestine

d. large intestine

which of the following is true regarding panic d/o and agoraphobia? a. more men than women experience painc d/o without agoraphobia b. more women than men experience pain d/o without agoraphobia c. more men than women experience panic d/o with agoraphobia d. more women than men experience panic d/o with agoraphobia

d. more women than men experience panic d/o with agoraphobia

with the use of a lipid-lowering resin, which of the following enzymes should be periodically monitored? a. ALP b. LDH c. AST d. no particular monitoring is recommended

d. no particular monitoring is recommended

H. has BPH and complains of some incontinence. your first step in dxing overflow incontinence would be to order a: a. urinalysis b. cystometrogram c. cystoscopy d. postvoid residual (PVR) urine measurement

d. postvoid residual (PVR) urine measurement

you see a woman who has been sexually involved without condom use with a man newly dxed with acute hepatitis B. She has not received hepatitis B immunization. You advise her to: a. start hepatitis B immunization series b. limit the number of sexual partners c. be tested for HBsAB d. receive hepatitis B immune globulin and start hepatitis B immunization series

d. receive hepatitis B immune globulin and start hepatitis B immunization series

you suspect a herniated disk on S., age 72. you elevate her affected leg when she is in the supine position and it elicits back pain and sciatic nerve pain. which indicates a positive test. this is known as which test or sign? a. femoral stretch test b. cross straight-leg-raising c. doorbell sign d. straight-leg-raising test

d. straight-leg-raising test

a fixed, painless thyroid mass accompanied by hoarseness and dysphagia should raise the suspicion of: a. autonomously functioning adenoma b, Graves' disease c. Hashimoto's disease d. thyroid malignancy

d. thyroid malignancy

diagnostic criteria for irritable bowel syndrome include abdominal pain that is associated with all of the following EXCEPT: a. improvement with defecation b. a change in frequency of stool c. a change of stool form d. unexplained wt loss

d. unexplained wt loss

Primary adrenal insufficiency refers to the gland being

damaged

What special examinations should be periodically obtained during hydroxychloroquine sulfate use?

dilated eye retinal examination

DDx of nipple discharge

duct ectasia endocrine alterations inhibition of dopamine prolactinoma malignancy Meds: methyldopa, illicit drugs, neuroleptics, phenothiazines, oral contraceptives, tricyclics, opiates)

In men older than age 35 years, acute epididymitis is often seen secondary to prostatitis and is typically caused by a ________organism.

gram (-)

Hyperthyroidism, Hypothyroidism or Both -thick dry nails

hypo

NPH is an example of what type of insulin

intermittent and can be used BID

Compared with the oral form, is transdermal estrogen use is associated with a higher or lower thromboembolic risk in short-term studies?

lower

brudzinski sign is positive with

meningitis

Be suspicious of ovarian cancer if you find what on exam?

palpable ovary in females of NON-childbearing age OR females of childbearing age with a persistent ovarian cyst >/=5cm for >6wks

ED medication? 15min, lasts up 36 hours, 10-20mg Approved for daily use

tadalafil (Cialis)

how do you test for Chvostek's sign?

tapping the facial nerve below the zygomatic arch anterior to the ear lobe pg 831

Drugs that cause gout

thiazide niacin aspirin cyclosporine Alcohol can precipitate gout by causing hyperuricemia

is a result of subclavian artery compression resulting in severe ischemia of the arm. absent radial pulse with positional change

thoracic outlet syndrome (TOS)

Treatment for Psoriasis include: Select all that apply 1. topical steroids 2. topical retinoids (Tazorotene) 3. tar preparations (psoralen drug class) 4. Biologicals/TNF agents (methotrexate, Humira,Enabrel and Remicade)

topical steroids, topical retinoids (Tazorotene) and tar preparations (psoralen drug class)

T/F: Ages 30-65 years: Should be screened at a maximum every 5 years (Pap test should be done)

true

T/F: An observed increased risk of breast cancer in women who use HT has also been noted, particularly with long-term use.

true

T/F: Most men have asymptomatic gonococcal infection.

true

what does the glandular enlargement of gynecomastia feel like?

usually resistive and ropy in texture pg 831

Skin Lesion: Example: Herpetic lesion --superficial blister w/serous fluid <1cm

vesicle

Celiac Disease s/s

weight loss, chronic diarrhea, and muscle wasting. Flatulence and abdominal distension. Growth restriction when diagnosed in children less than 2 years old.

when is a mammogram indicated in gynecomastia?

when it is accompanied by other breast abnormalities, especially if it is unilateral, a disc > than 4cm in diameter, does not resolve after two years after puberty, occurs in the presence of abnormal serum levels of free testosterone and luteinizing hormones or abnormal testis pg 831-832

when should thyroid cancer be considered?

when the thyroid gland is hard and enlarged or when nodules are palpated pg 842

According to the American College of Gastroenterology guidelines on diagnosing acute pancreatitis, 2 of the 3 must be present. name all 3

-severe abdominal pain -serum amylase and/or lipase 3x normal -characteristic abdominal imaging findings

What is the average age of menopause?

51 years (45-55)

When prescribing itraconazole (Sporanox), the NPconsiders that: A. the drug is a cytochrome P-450 3A4 inhibitor. B. one pulse cycle is recommended for fingernail treatment, and two cycles are needed for toenail therapy. C. continuous therapy is preferred in the presence of hepatic disease. D. taking the drug on an empty stomach enhances the efficacy of the product

A. the drug is a cytochrome P-450 3A4 inhibitor

The presentation of acute epididymitis in an otherwisewell 22-year-old man includes: A. the presence of a positive Prehn sign. B. low back pain. C. absent cremasteric reflex. D. diffuse abdominal pain.

A. the presence of a positive Prehn sign. pain relieved when teste is elevated

A transvaginal ultrasound in the woman with PID will likely show: A. tubal thickening with or without free pelvic fluid. B. cervical thickening. C. endometrial thinning. D. inflammation of the ovaries.

A. tubal thickening with or without free pelvic fluid.

Absolute contraindications to postmenopausal HT include: A. unexplained vaginal bleeding. B. seizure disorder. C. dyslipidemia. D. migraine headache.

A. unexplained vaginal bleeding.

Which of the following is the most likely to be found in deep vein thrombophlebitis (DVT)? A. unilateral leg edema B. leg pain C. warmth over the affected area D. positive obturator sign

A. unilateral leg edema

Which of the following is the preferred medication to reverse the anticoagulant effects of warfarin? A. vitamin K B. protamine sulfate C. platelet transfusion D. plasma components

A. vitamin K

hich of the following is recommended by the Centers for Disease Control and Prevention as single -dose therapy for uncomplicated urethritis caused by N. gonorrhea when an oral product is the most appropriate option? A. cefixime B. metronidazole C. azithromycin D. amoxicillin

A. cefixime

First-line therapy for angular cheilitis therapy includes the use of: A. metronidazole gel B. hydrocortisone cream C. topical nystatin D. oral ketoconazole

C. topical nystatin

A woman was treated as an inpatient for a serious soft tissue infection with parenteral linezolid and now is being seen on day 3 of her illness and is being discharged to home. She is feeling better and appears by examination to be clinically improved. Culture results reveal MRSA, sensitive to trimethoprim-sulfamethoxazole, linezolid, daptomycin, vancomycin,and clindamycin and resistant to cephalothin and erythromycin. Her antimicrobial therapy should be completed with: A. oral cephalexin. B. oral trimethoprim-sulfamethoxazole. C. parenteral vancomycin. D. oral linezolid

B. oral trimethoprim-sulfamethoxazole

A 29-year-old woman has a sudden onset of right-sided facial asymmetry. She is unable to close her right eyelid tightly or frown or smile on the affected side. Her examination is otherwise unremarkable.Which of the following represents the most important diagnostic test? A. complete blood cell count with white blood cell differential B. serum testing for Borrelia burgdorferi infection C. computed tomography (CT) scan of the head with contrast enhancement D. serum protein electrophoresis

B. serum testing for Borrelia burgdorferi infection

Risk factors for the development of postherpetic neuralgia include: A. age younger than 50 years at the time of the outbreak B. severe prodromal symptoms C. lumbar location of lesions D. low volume of lesions

B. severe prodromal symptoms

A common site for atopic dermatitis in an infant is: A. the diaper area. B. the face. C. the neck. D. the posterior trunk

B. the face. Occlusive damp environments are usually spared. Babies juice neck and moist diaper area.

When counseling a patient about the use of tacrolimus (Protopic) or pimecrolimus (Elidel), you mention that: A. this is the preferred atopic dermatitis treatment in infants B. there is a possibility of increased cancer risk with its use. C. the product is used interchangeably with topical corticosteroids. D. the product is a potent antihistamine

B. there is a possibility of increased cancer risk with its use

Which of the following is recommended for preventing a burn wound infection? A. topical corticosteroid B. topical silver sulfadiazine C. oral erythromycin D. oral moxifloxacin

B. topical silver sulfadiazine

Topical therapies for the treatment of rosacea include all of the following except: A. metronidazole cream B. azelaic acid gel C. medium-potency corticosteroid cream D. benzoyl peroxide

C. medium-potency corticosteroid cream Can make it worse

Which of the following is an oral antimicrobial option for the treatment of a community-acquired methicillin-resistant S. aureus cutaneous infection? A. amoxicillin B. dicloxacillin C. cephalexin D. trimethoprim-sulfamethoxazole

D. trimethoprim-sulfamethoxazole

Well-controlled human study fails to demonstrate fetal risk in 1st trimester • No evidence of risk in 2nd, 3rd trimesters • Risk to fetus appears remote - <1% of all medications • Needed for health • Produced by the body • Vitamins in recommended doses - Vitamin A caution • Risk factor X in doses >8000 U • Levothyroxine (Synthroid®) - A bioidentical hormone • In urgent care with "unexpected" pregnancy dx, what should you advise? what category

Category A

Acute onset of saddle anesthesia bladder incontinence/retention or fecal incontinence. bilateral leg numbness & weakness. Due to pressure on sacral nerve root....ischemia ***A surgical emergency***** Needs spinal decompression. Refer to ED.

Cauda Equina Syndrome

What dietary change increases bilirubin and ammonia levels with hepatitis?

^ protein diet

an elderly hypertensive pt has osteoporosis. which antihypertensive agent would have the secondary effect of improving her osteoporosis? a. thiazide diuretic b. calcium channel blocker c. an ACE inhibitor d. a beta-blocker

a. thiazide diuretic

when advising a person who will be using orlistat (xenical, alli) as part of a weight loss program, the NP provides the following information about when to take the medication: a. within an hr of each meal that contains fat b. before any food with high carbohydrate content c. only in the am, to avoid sleep disturbance d. up to 3 hrs after any meal, regardless of type of food eaten

a. within an hr of each meal that contains fat

Medications that worsen GERD, select all that apply: a] Estrogen/Progestin b] Theophylline c] CCBs d] Nicotine e] ACEIs/ARBs

a] Estrogen/Progestin b] Theophylline c] CCBs d] Nicotine

what is the recommendation of American Cancer Society for screening an average risk 40 y/o African America male for prostate cancer? a. digital rectal exam starting at age 40 years b. PSA starting at age 45 years c. discussions starting at age 40-45 years d. he should be screened starting at age 50 years

c. discussions starting at age 40-45 years

which of the following is the most cost-effective method of distinguishing a malignant from a benign thyroid nodule? a. ultrasound b. magnetic resonance imaging c. fine-needle aspiration biopsy d. radioactive iodine scan

c. fine-needle aspiration biopsy

untreated hypothyroidism can result in which of the following changes in the lipid profile? a. increased HDL and decreased triglycerides b. increased LDL and total cholesterol c. increased LDL, total cholesterol, and triglycerides d. decreased LDL and HDL

c. increased LDL, total cholesterol, and triglycerides

ideally, a pt should have a fasting glucose that is: a. between 60-100 mg/dL b. less than 126 mg/dL c. less than 100 mg/dL d. repeated in a non-fasting state

c. less than 100 mg/dL

When listening to heart sounds, the nurse knows that S1:

coincides with the carotid artery pulse

in a pt who presents with a hx consistent with anal fissure but with notation of an atypical anal lesion, alternative dx to consider include all of the following EXCEPT: a. condyloma acuminata b. Crohn's disease c. anal squamous cell carcinoma d. C. difficile colitis

d. C. difficile colitis

a dx of type 2 diabetes can be made: a. if risk factors plus a family hx of diabetes are present b. with an Hgb A1C of 7% or greater c. if glucose values of 100, 119, and 115 are observed on different days d. following fasting glucose values of 126 and 130 on different days

d. following fasting glucose values of 126 and 130 on different days

which of the following is true concerning colorectal cancer? a. most colorectal cancer are found during rectal exam b. rectal carcinoma is more common than cancers involving the colon c. early manifestations include abdominal pain & cramping d. later disease presentation often includes iron-deficiency anemia

d. later disease presentation often includes iron-deficiency anemia

You have a patient on Metformin, A1C is not at goal....what do you add

depends on side effects, cost, will pt inject etc typically a sulfonylurea good insurance, GLP-1

During inspection of the precordium of an adult patient, the nurse notices the chest moving in a forceful manner along the sternal border. This finding most likely suggests:

enlargement of the right ventricle

a 50 y/o pt reports acute pain in his lower back that started 2 wks ago after working in his yard. the pain radiates into his rt leg intermittently. he has been managing his pain with over the counter NSAIDs. there are no red flags in his hx or exam. when should consideration be given to imaging studies? a. now b. at 4 wks c. at 8 wks d. there is no need

b. at 4 wks

the most common polyneuropathy in the elderly is: a. Charcot-Marie-Tooth disease b. diabetes mellitus c. urinary incontinence d. Guillain-Barre syndrome

b. diabetes mellitus

which of the following is most consistent with the presentation of a pt with bipolar I d/o? a. increased need for sleep b. impulsive behavior c. fatigue d. anhedonia

b. impulsive behavior

an adolescent has suspected varicocele. he has dull scrotal pain that is relieved by: a. standing b. recumbency c. having a bowel movement d. elevation of the testicle

b. recumbency

a pt with hypothyroidism has been in a euthyroid state for several yrs. On screening, her TSH is elevated. The most likely cause of this is: a. change in lab's method of measuring TSH b. substitution of levothyroxine for a generic medication c. aging d. taking extra doses of T4

b. substitution of levothyroxine for a generic medication

identify the following findings as associated with hyperthyroidism, hypothyroidism, or both: goiter, frequent, low volume, loose stools, atypical presentation in an elderly person change in mental status

both

GERD, endoscopy indicated when

only in the presence of alarm symptoms, not bc of typical GERD symptoms. Alarm.... -weight loss -dysphagia -vomiting blood

Symptoms of ____ are similar to those of testicular torsion. These can include: Ejaculation of blood Hematuria (blood in the urine) Severe pain Swelling of a testicle or testicles and often the inguinal lymph nodes on the affected side.

orchitis causes: Older adolescent/adults: chlamydia and gonorrhea. Adolescent: Orchitis can be seen during active mumps

gradual onset of numbness and tingling in hands and feet burning pain, sharp electric-like pain muscle weakness extreme sensitivity to touch

peripheral neuropathy

Bipolar (Manic-depressive disorder) type 1 __________ type 2 __________ Strong _____ component PEAK-______s ( RANGE ______)

(classic mania) (hypomanic) genetic 20s ( RANGE 14-30)

Hale's Lactation Risk Category • L5 - Contraindicated

- Significant and documented risk • Radioactive isotopes • Cocaine

- thick, white, curd-like discharge; vulvovaginal erythema with pruritus - KOH mixture shows pseudo hyphae

- thick, white, curd-like discharge; vulvovaginal erythema with pruritus - KOH mixture shows pseudo hyphae MGMT: Miconazole Qhs x 7

Danger Signals Mental Health (3)

-Depression w/suicidal plan, acute mania w/psychosis, severe anorexia

Danger Signals CV (4)

-MI,CHF,DVT,AAA

DM Dietary Recommendations

-Saturated fat <7% (animal fats) -Minimal trans fats - fried and junk food -Dietitian at least once for carb counting -Vitamin supplements not recommended

Signs of hip fracture are:

-Sudden onset of one-sided hip pain. -Unable to walk and bear weight on affected hip. (If mild fracture, may bear weight on affected hip.) ***displaced fracture*** presence of severe hip pain with EXTERNAL ROTATION of the hip/leg (ABDUCTION) and leg shortening. More common in elderly. Elderly have a 1-year mortality rate from 12% to 37%.

Danger Signals of Renal (1)

-acute pyelonephritis

fibromyalgia tender points A diagnosis of fibromyalgia requires detecting at least how many tender points? A. 4 B. 7 C. 11 D. 18

-back of head; -chest (upper & lower); -upper back (medial & lateral); -low back; -buttocks; -gr trochanter ; -medial knee; -lateral epicondyles C. 11

Risk factors for HBV infection

-sex w/ > 1 partner MSM -sharing needles w/injection drug use, -job w/ blood exposure, -traveling to areas w/^rates (Africa, Central & Southeast Asia, Central Europe), -predominant is through sexual activity & injection drug use, -cannot be transmitted by food or water

Danger Signals of HEENT (4)

-sudden vision loss or rapid worsening vision, herpes keratitis, temporal arteritis, acute angle-closure glaucoma

all of the following agents can be considered for the txment of severe cases of SLE except: a. leflunomide b. azathioprine c. rituximab d. belimuma

. rituximab

***LEIK*** [Suicide} men vs women (^success/^attempts) Ppl w/ bipolar @ higher risk during depressive phase

...

Regular insulin onset of action =30min-1 hr peak=2-3 hr duration=4-6 hr

...

Who is likely to get epididymitis? Cause?

1. <35 yo 2. Chlamydia

Number one cancer killer in Men and Women?

1. Lung 2. Colorectal

the two major types of IBD are

1. ulcerative colitis in which the pathological changes are limited to the colon 2. Crohn's disease in which the changes can involve any part of the GI tract

What is NOT an example of an aura secondary to a migraine headache? 1. Metallic taste in mouth 2. Sneezing 3. Hyperosmia 4. Blurred vision

2. Sneezing

How many steroid shots can one get in a joint per year?

3

NPH (Novolin N) (Humulin N) *intermittent acting* peak:

6-14 hours

You see a 33-year-old male with a minor dog bite on his hand. The examination reveals a superficial wound on the left palm. The dog is up-to-date on immunizations. In deciding whether to initiate antimicrobial therapy, you consider that ______ of dog bites become bacterially infected. A. 5% B. 20% C. 50% D. 75%

A. 5%

The mechanism of action of imiquimod is as a/an: A. immunomodulator. B. antimitotic. C. keratolytic. D. irritant

A. immunomodulator

When assessing a 78-year-old man with suspected BPH, the NP considers that: A. prostate size does not correlate well with severity of symptoms. B. BPH affects less than 50% of men of this age. C. he is at increased risk for prostate cancer. D. limiting fluids is a helpful method of relieving severe symptoms.

A. prostate size does not correlate well with

During a physical examination, the nurse finds that a male patient's foreskin is fixed and tight and will not retract over the glans. The nurse recognizes that this condition is

ANS: phimosis. With phimosis, the foreskin is nonretractable, forming a pointy tip of the penis with a tiny orifice at the end of the glans. The foreskin is advanced and so tight that it is impossible to retract over the glans. This may be congenital or acquired from adhesions related to infection.

When assessing the scrotum of a male patient, the nurse notices the presence of multiple firm, nontender, yellow 1-cm nodules. The nurse knows that these nodules are most likely:

ANS: sebaceous cysts. Sebaceous cysts are commonly found on the scrotum. These are yellowish 1-cm nodules and are firm, nontender, and often multiple. The other options are not correct.

How soon after unprotected sex must emergency contraception be used to be effective?

ASAP is ideal. Recommended within 72 hr (3 days), but effective up to 120 hr (5 days)

If the IgM is elevated in the absence of IgG, what should be suspected?

Acute hepatitis

What test? flex knee 90 degrees with pt prone, put pressure on heel with one hand while rotation get lower leg internally and externally. *pain or click is positive for medial or lateral collateral ligament damage or meniscus injury

Apley's grind test

osgood-schlatter disease treatment

Apophysitis of the tibial tubercle caused by trauma or overuse occurs in adolescence, growth. -Localized pain/swelling over tibial tubercle. Self-limiting, heals when the epiphysis closes. Localized pain/swelling over tibial tubercle. Treat with refraining from PE, stretching, ice, NSAIDs

lab? 0-31, increases in response to hepatocyte injury, as often occurs in alcohol abuse, acetaminophen misuse/overdose, and quite rarely the therapeutic use of HMG-COA reductase inhibitors; this enzyme is also found in skeletal muscle, myocardium, brain & kidneys which can cause slight increase in enzyme, half-life 12-24 hrs

Aspartate aminotransferase (AST)

Somatic S/S of PMS?

Body aches, breast tenderness, bloating, HA, food cravings,

Anti-HBs [HBsAb] (-) (HepB surfaceantibody)

Cant have antibody and antigen on board at the same time!

_________________results in diffuse damage to the proximal small intestinal mucosa with malabsorption of nutrients.

Celiac Disease

HBsAg, Anti-HBc, IgM, IgG

Chronic Hep B:

A test for knee stability. A diagnostic sign of a torn or ruptured ligament. The positive anterior drawer sign is the test for the anterior cruciate ligament (ACL). The posterior drawer sign is the test for the posterior cruciate ligament (PCL).

Drawer Sign (lachman)

To assess whether the supraspinatus muscle is intact (rotator cuff) this test is performed:

Drop arm test The patient is asked to abduct his affected arm laterally to 90 degrees. He is then asked to slowly lower it to his side. A positive test is noted when the arm drops.

CLASSIC CASE: Middle-age to older adult complains of episodic epigastric pain, burning/gnawing pain, or ache (80%), Pain relieved by food and or antacids (50%) with recurrence 2-4 hours after meal. Self-medicating with OTC antacids. Maybe taking NSAIDs or ASA.

Duodenal ulcer

Normal pelvic exam findings post menopause

Flattening of vaginal rugae Vaginal pH = 5.6 Scant white vaginal dishcharge

Chancroid is caused by what organism

H. ducreyi

HGSIL

High-grade squamous intraepithelial lesion CIN II (i.e. moderate dysplasia) CIN III (severe dysplasia) CIS (carcinoma in situ) Squamous cell carcinoma

older kids, jaundice, bilious vomit, n/v/d, failure to pass meconium

Hirschsprungs

How long should back up method be used after getting a Depo shot?

If given on DOC 1-5= No back up needed If given after DOC 5=2 weeks

When do you increase basal insulin? By how much?

If they are not at their AM fasting glucose goal (80-130) Increase basal insulin by 2-3 units every 2-3 days until goal.

Specific Headache Clues: Blurred vision on bending of the head

Intracranial lesion

Why should you not administer laxatives to the patient with appendicitis? Can you give stool softeners then?

It could cause the appendix to rupture Yes, you can give stool softeners if complaining of constipation

What is HBc?

It is a test that detects the presence of both IgM and IgG antibody to hepatitis B core antigen. HBc indicates ongoing or previous hepatitis B viral infection and remains for life

Who would you not give vasoconstricting drugs to?

Known/suspected cardiovascular disease (angina, MI, PAD) Suspected/known CVA/TIA hyperlipidemia in males over 40 or menopausal women uncontrolled HTN complex migraine (basilar, hemiplegic migraine)

Alcoholic who has chronic depletion of thiamine (B1 ) develops permanent brain damage...manifested how?

Korsakoff's amnesic sydrome S/s: confabulation, disorientation, attention deficit, and visual impairement.

________________, approved for the treatment of constipation that is not amenable to standard therapies, promotes fluid secretion into the intestinal lumen and is a helpful option in constipation-dominant IBS.

Lubiprostone (Amitiza)

CLASSIC CASE: Symptoms include sudden onset of sore throat, cough, fever, headache, stiff neck, photophobia and changes in LOC. The appearance could be toxic. There may be an abrupt onset of petechial to hemorrhagic rash (pink-purple colored) in the axillae, flanks, wrist and ankles. Rapid progression in fulminant cases result in death wiithin 48h. The risk higher for college students who reside in dorms. Spread by aerosol droplets. Rifampin prophylaxis is recommended for close contacts

Meningococcemia

The most common inherited neuromuscular disease in children

Muscular dystrophy

• Most common disorder of neuromuscular transmission • Affects 20/100,000 • Autoimmune disorder (autoantibodies to acetylcholine receptor) • Variable weakness in ocular, limb, and respiratory muscles

Myasthenia Gravis

DDP-4 inhibitors (gliptins - januvia) are weight ______

Neutral

If E. Coli resistance to Bactrum is > 20% or sulfa allergy then what do you prescribe for acute uncomplicated UTI in a nonpregnant woman ?

Nitrofurantoin (macrobid) 100mg BID x 5 days Fosfomycin (Monourol) 3g x 1 dose ALL PLUS PHENAZOPYRIDINE (PYRIDIUM) - UTI analgesis!

what is the most prevalent skeletal problem in the US? a. osteoarthritis b. stress fx c. osteoporosis d. gouty arthritis

OA

What lubricants should be avoided with diaphragm?

Oil based--destroys latex

How is the Ortho Evra patch used?

Once per week x 3 weeks, off for one week Start on the first day of bleeding This day is now known at "Patch Change Day"

-Failure rate 1-3% -Copper IUD -MOA: inhibits fertilization -Lasts for 10-12 yrs -Increased risk for PID during initial 6 weeks after insertion -May help to prevent Asherman's syndrome -ABSOLUTE CONTRAINDICATION: IUP; active, recent or recurrent STI or PID; Undiagnosed AUB

ParaGard

Skin Lesion: Example: psoriatic lesion -flattened elevated lesion w/ variable shape that is >1cm

Plaque

carpal tunnel test: reproduction of symptoms after 1 min of wrist flexion

Positive Phalen's test

improves the kidney's ability to remove uric acid from the body.

Probenecid (Probalan) SE of drug use higher concentrations of uric acid in the urine = risk of kidney stones - rash - abdominal pain.

Central obesity, moon face, buffalo hump, acne, purple striae, hirsutism, amenorrhea, poor wound healing, frequent infections, labile mood, *hypertension, hyperglycemia

S/S of Cushings?

Alternate drugs to HT for hot flashes and vasomotor symptoms

SSRIs SNRIs Gabapentin

1st line for OCD and depression

SSRIs citalopram (Celexa) escitalopram (Lexapro) fluoxetine (Prozac) sertaline (Zoloft) paroxetine (Paxil)

Transillumination Serous fluid =? Blood or mass?

Serous fluid - bright red glow (hydrocele) Blood or mass - dull glow (ie. tumor)

howell-jolly bodies is associated with?

Sickle cell anemia seen on hemoglobin electrophoresis

autosoml recessive disorder that causes Hbg S homozygous incidence= 1 in 400

Sickle cell anemia (Hbg SS)

Complication with more than 3 steroid shots in a year?

Soft tissue breakdown Endocrine difficulty (Addisons)

The levothyroxine dose should be titrated so that the _____ is normal

TSH

major risk factors for diverticulosis include all of the following except: a. low-fiber diet b. family hx of the condition c. older age d. select connective tissue disorders (e.g. Marfan syndrome)

a. low-fiber diet

the straight leg raising maneuver can be used to dx: a. nerve root compression b. a fx hip c. an anterior cruciate ligament tear d. tendinitis

a. nerve root compression

obesity is defined as having a body mass index (BMI) equal to or greater than ____kg/m2 a. 25 b. 30 c. 35 d. 40

b. 30

a pt who is scheduled for pelvic exam with PAP smear should be advised to avoid douching, sexual intercourse, and tampon use before her exam. For how long should she be advised to avoid these activities for optimal evaluation? a. 24 hrs b. 48 hrs c. 36 hrs d. one week

b. 48 hrs

what is the average LDL reduction achieved with a change in diet as a single lifestyle modification? a. less than 5% b. 5% to 10% c. 11% to 15% d. 16% to 20% or more

b. 5% to 10%

M. age 52 is a postal worker who drive a truck every day. he presents with lower back pain and has decreased sensation to a pinprick int he lateral leg and web of the great toe. this indicates discogenic disease int he dermatomal pattern of which area? a. L3/L4 (L4 root involvement) b. L4/L5 (L5 root involvement) c. L5/S1 (S1 root involvement) d. none of the above

b. L4/L5 (L5 root involvement)

which of the following is true concerning hepatitis B vaccine? a. the vaccine contains live hepatitis B virus b. most individuals born since 1986 in the US who have been fully immunized have received vaccine against HBV c. the vaccine is contraindicated in the presence of HIV vaccine d. post-vaccination arthralgias are often reported

b. most individuals born since 1986 in the US who have been fully immunized have received vaccine against HBV

which of the following is often reported by anxious pts? a. constipation b. muscle tension c. hive-form skin lesions d. somnolence

b. muscle tension

In PTSD, pt has agoraphobia, detachment hyperarousal... drug choice?

clonidine

pts with a Grade III ankle sprain should be advised that full recovery is likely to take: a. few days b. 2 to 3 weeks c. 4 to 6 weeks d. many months

d. many months

What is the first course of action in treatment of a human bite?

debride and irrigate thoroughly

what is abduction?

moving limb away from body

Screening for Genetic Disorders ***African***, Latino, Arabic, Greek, Maltese, Italian, Sardinian, Turkish and Indian ancestry Prior to or in early pregnancy

(sickle cell anemia)

What the presence of anti-HBs indicate? (2)

1. Anti-HBs is generally interpreted as recovery and immunity from hepatitis B virus infection. 2. Anti-HBs also develops after vaccination.

Before what age would menopause be considered "premature"?

40

When to start the DRE? When to start PSA? Diagnostics with prostate cancer?

40 y.o. 50 y.o or 10 years before family history age PSA Needle biopsy US REFER

Treatment options for lymphogranuloma venereum include: A. doxycycline. B. penicillin. C. ceftriaxone. D. dapsone.

A. doxycycline. also Erythromycin

Which of the following agents should be avoided in heavy alcohol users due to a potential risk for hepatotoxicity? A. duloxetine B. desvenlafaxine C. escitalopram D. bupropion

A. duloxetine

Early neurological changes in patients with lumbar radiculopathy include: A. loss of DTRs B. poor two-point discrimination. C. reduced muscle strength. D. footdrop.

A. loss of DTRs

When counseling a person who has a 2-mm verruca-form lesion on the hand, you advise that: A. bacteria are the most common cause of these lesions. B. lesions usually resolve without therapy in 12 to 24 months. C. there is a significant risk for future dermatologic malignancy. D. surgical excision is the treatment of choice

B. lesions usually resolve without therapy in 12 to 24 months

The spectrum of antimicrobial activity of mupirocin(Bactroban) includes: A. primarily gram-negative organisms. B. select gram-positive organisms. C. Pseudomonas species and anaerobic organisms. D. only organisms that do not produce beta-lactamase.

B. select gram-positive organisms.

All of the following are typical findings for a patient with chancroid except: A. multiple lesions. B. spontaneous rupture of affected nodes. C. blood-tinged penile discharge. D. dense, matted lymphadenopathy on the ipsilateral side of the lesion.

C. blood-tinged penile discharge.

-median nerve compression of the wrists beneath the transverse carpal ligament -Associated with repeated wrist flexion, Women, dominant hand

Carpal Tunnel Syndrome

carpal tunnel test: pressure with the examiner's thumb over the patient's carpal tunnel for 30 sec elicits sx

Carpal compression test

What is the Alternative if e.coli resistance to bactrum > 20% and can not use nitrofurantoin or fosfomycin? For acute uncomplicated UTI in nonpregnant woman

Ciprofloxacin 250mg BID Ciprofloxacin 500mg ER QD Levofoxacin 250 mg QD Moxifloxacin 400 mg QD ALL 3 days PLUS Pyridium

What test should be done before initiating coagulation therapy (8')

Clotting studies: protein s protein c antithrombin III fibrinogen lupus anticoagulant factor V Leiden assess for strong family history of prior thromboembolic events

In advising a perimenopausal woman about HT, you consider that it may: A. reduce the risk of venous thrombotic events. B. significantly reduce serum triglyceride levels. C. worsen hypertension in most women. D. help preserve bone density.

D. help preserve bone density.

You see a 67-year-old man with seborrheic dermatitis that has failed to respond to treatment with ketoconazole shampoo. An appropriate second-line treatment option can include all of the following except: A. oral fluconazole. B. a topical immune modulator. C. topical propylene glycol. D. high-potency topical corticosteroid

D. high-potency topical corticosteroid With ANY skin condition, high-potency topical corticosteroids are discouraged due to rise for atrophy etc.

Common clinical manifestations of anaphylaxis can include all of the following except: A. upper airway edema B. itch without rash C. dizziness with syncope D. hypertension

D. hypertension

Painless can be unilateral or bilateral when unilateral there is unequal knee height due to abnormal dislocation (luxation or sub-luxation) in which the femoral head is partially or completely displaced from the acetabulum may not be observed in newborns or infants once child begins to walk it is noted, painless limp, decreased hip abduction

Developmental Dysplaia of the hip (DDH) order X-ray refer to ortho

______________, found in the COC products Yasmin and Yaz, is an analogue of an aldosterone antagonist and has potassium-sparing qualities. It should be used with caution in hepatic or renal dysfunction or with concomitant use of ARBs, ACEIs, salt substitute, or potassium-sparing diuretics.

Drospirenone

What are the hallmark S/S of Crohns?

Fatigue/fever abdominal pain/diarrhea with/without bleeding, weight loss Cobblestoning or skip lesions seen

True or False: The mechanism of resistance of MRSA is via the production of beta-lactamase.

False

T/F: You should always order a Total T4 (normal 4.5 - 12.0 mcg/dL) when evaluating thyroid function.

False. Total T4 reflects the total protein bound and free thyroxine.

1. Deep pain or 'catch' on inspiration while fingers are placed under the right rib cage--also called inspiratory arrest 2. Gallbladder assessment

How is Murphy's sign assessed? When is it done?

Pharmacologic Management Migraine • Moderately severe:

Moderately severe: o Triptans/DHE o Analgesics (Fiorinal, Codeine, others) o Ketorolac (Toradol) IM

Prenatal visit schedule

Monthly until 28 weeks Biweekly until 36 weeks Weekly until delivery

a feeling of standing on a pebble in shoe shooting pain affecting the contiguous halves of 2 toes tingling or numbness in the toes are signs of what? how would you diagnose and manage it?

Morton's neuroma U/S, MRI for lesions orthotics and corticosteroid injections refer

destruction of articular cartilage; ASYMMETRICAL age 53-64, men and women =, hips, knees, fingers, wrists, swelling edema, NO redness or heat Pain better in morning, Worse with activity

Osteoarthritis

Where are Bouchards nodes?

PIPs

Caution with bisphosphonates?

Rare osteonecrosis of the jaw in women with an underlying diagnosis of cancer

Quadruple Screen Test

Quadruple Screen Test Combination of the Triple Screen hormones plus inhibin-A (hormone release by the placenta). The Triple or Quadruple Screen tests are more sensitive than the AFP alone (but higher rate of false positives). Gold standard test for genetic disorders is testing of fetal chromosomes/DNA.

Rx Management with acute bacterial prostatitis? Symptomatic treatment of acute bacterial prostatitis? Sex?

Refer for septicemia or urinary retention 1. TMZ-SMZ, 2. Levofloxacin 3. Ofloxacin Sitz bath 30 minutes TID No Sex until acute phase is resolved

Mechanism of action w/ OCPs with progesterone?

Thickens mucus=decrease sperm transport Slows ovum transport Suppression of endometrium=hinders implantation Inhibits ovulation by interfering with HPO axis

Management of fibrosystic breast disease?

Warm soaks, Low sodium diet, vitamin E, Hormones, surgical intervention

What lab findings are found in pancreatic cancer? (4)

normochromic, normocytic anemia elevated: total & direct bilirubin & alkaline phosphatase amylase only elevated with concomitant pancreatitis

lack of the protein child has weak pelvic girdle firm large woody caves goal is to delay progression OT/PT

mucular dystrophy

the tests in male infants descend from the retroperitoneal space through the inguinal canal and into the scrotal sacs. this most commonly occurs: a. during 2nd trimester b. during 3rd trimester c. during neonatal period d. by age 6 months

b. during 3rd trimester

the maximum recommended dose of citalopram for pts older than 60 yrs is: a 10mg/day b. 20mg/day c. 30mg/day d. 40mg/day

b. 20mg/day

which of the following is likely to be reported in a pt with persistent GERD? a. hematemesis b. chronic sore throat c. diarrhea d. melena

b. chronic sore throat

a common side effect of thiazide diuretics is: a. prostatitis b. erectile dysfunction c. fatigue d. hyperkalemia

b. erectile dysfunction

use of St. John's wort is know to impact the effectiveness of all of the following meds EXCEPT: a. oral contraceptives b. fluoroquinolones c. cyclosporine d. select antiretrovirals

b. fluoroquinolones

which of the following meds is most likely to cause sexual dysfunction? a. vilazodone (viibryd) b. fluoxetine (prozac) c. nortiptyline (pamelor) d. bupropion (wellbutrin)

b. fluoxetine (prozac)

an elderly male pt is taking finasteride, a 5-alpha-reductase inhibitor. what affect might this have on his PSA level? a. it will increase b. it will decrease c. there is no predictable change d. there will be no change

b. it will decrease

initial tx for pts with carpal tunnel syndrome includes: a. intra-articular injection b. joint splinting c. systemic corticosteroids d. referral for surgery

b. joint splinting

tx for hemorrhoids includes all of the following EXCEPT: a. wt control b. low-fat diet c. topical corticosteroids d. the use of a stool softener

b. low-fat diet

which of the following is most likely to be found in a person with acute cholecystitis? a. fever b. vomiting c. jaundice d. palpable gallbladder

b. vomiting

Creatine kinase measures what

brain and muscle damage

a high purine diet can contribute to gouty arthritis. Which food listed below contributes most to a high purine diet? a. coffee b. eggs c. beef d. bread

c. beef

L., age 52, who has just been given a dx of sarcoidoxis, has joint sxs including arthralgias and arthritis. your next plan of action would be to: a. order a bone scan b. obtain a tissue biopsy c. begin a course of glucocorticoids d. obtain an electrocardiogram

c. begin a course of glucocorticoids

S. age 50, presents with Paget's disease that has been stable for several yrs. recently, his serum alkaline phosphatase level has been steadily rising. you determine that it is time to start him on: a. NSAIDs b. corticosteroids c. bisphosphonates d. calcitonin

c. bisphosphonates

digital rectal exam may be performed to assess the prostate gland. Which term does NOT describe a prostate gland that may have a tumor? a. nodular b. asymmetrical c. boggy d. indurated

c. boggy

activities that commonly contribute to the development of rotator cuff tendonitis include all of the following except: a. swimming b. throwing a football c. bowling d. pitching a baseball

c. bowling

when caring for a pt with DM, HTN, & persistent proteinuria, the NP prioritizes the choice of antihypertension and prescribes: a. furosemide b. methyldopa c. fosinopril d. nifedipine

c. fosinopril fosinopril is an ACEI. The ADA are known for their hepatic protection and this is needed in diabetics

NSAID-induced peptic ulcer can be best limited by the use of: a. timed antacid doses b. an histamine2-receptor antagonist c. an appropriate antimicrobial d. misoprostol

d. misoprostol

a patient who is 5 feet tall and weighs 205 pounds (40kg/m2). How would the healthcare provider classify this patient? a. overweight b. mild obese c. moderate obese d. morbidly obese

d. morbidly obese

common causes of sciatica include all of the following except: a. herniated disk b. spinal stenosis c. compression fx d. soft tissue abnormality

d. soft tissue abnormality

Regular insulin (Humulin R, Novolin R) *short acting* what is the duration?

duration=4-6 hr

first line of choice for osteoporosis

oral bisphosphonates (fosamax, actonel, boniva, "dronate" meds)

Neurofibromatois (von Recklinghausen Disease)

neurocutaneous characterized by "numerous cafe'-au-lait spots" on the body and nerve tumors on the skin and in the body progressive disorder refer to neurology Diagnostic criteria (must have at least ) 1.) 6 or more spots 2.) 2+ cutaneous neurofibromas 3.) axillary/inguinal freckling 4.) 2+iris Lisch nodules 5.) osseous lesions

Secondary amennorhea (definition and DDx)

no menses for 90 days (3 missed cycles) after having established menses DDx: pregnancy, PCOS, thyroid, hypothalmic dysfunction, hyperprolactemia, ovarian failure, endocrine (DM, thyroid), meds like antidepressants, anti-HTN, steroids

Acroparesthesia? Associated with?

waking up at night with numbness and burning pain in the fingers. Carpel Tunnel Syndrome (CTS)

Bell's Palsy: Assessment Ask Patient to:........

• Close eyes • Elevate brow, frown • Show teeth • Pucker lips • Tense soft tissue of neck

Side effects of allopurinol

rash and low blood counts

Non-tender, firm nodules located in soft tissue.

trophi

What is the grace period between Depo shots?

two week grace period--if > two weeks do a HcG before giving

What

...

drop arm test

if pt can't hold arm fully abducted at shoulder level or lower slowly and controlled->positive for rotator cuff tear

Tender high rise testicle

testicular torsion

The presence of anterior and posterior anal fissures should raise suspicion for ___________ disease.

Crohn's

Which of the following is least likely to be found in patient with pulmonary embolus (PE) A. pleuritic chest pain B. tachypnea C. DVT signs and symptoms D. hemoptysis

D. hemoptysis

Treatment options for chancroid include all of the following except: A. azithromycin. B. ciprofloxacin. C. ceftriaxone. D. amoxicillin.

D. amoxicillin.

hematuria is not a common clinical manifestation in: a. early prostate cancer b. benign prostatic hyperplasia c. bladder cancer d. renal cancer

a. early prostate cancer

Up to what age in Genu Valgum normal

age 7

Adjust BCP dose based on complaint: pre-hypertension

decrease progestin

Hyperthyroidism, Hypothyroidism or Both -amenorrhea or oligomenorrhea

hyper

What is acute medication management with strain/strains? (Barclays rule of 3-4)

3-4 Ibuprofen 3-4 times day for 3-4 days---then stop OR 400-800 mg TID

A 44 year old presents with suspected BPPV. What historical finding is most likely? 1. She has a family hx of this. 2. There is ringing in her ears. 3. Symptoms are reproducible w/ head movement. 4. She has a headache.

3. Symptoms are reproducible w/ head movement.

What is the type of antibiotic of choice for appendicitis?

3rd generation cephalosporins

Bismuth regimen for H pylori eradication

4 times a day dosing BMT: bismuth, metronidazole, Tetracycline or BMT +omeprazole

**Diagnosing Diabetes** A1C is ________ Fasting glucose is ________ Random glucose is ________ OGTT is ________

=>6.5% 126mg/dl =<200mg/dl with symptoms 2-hr plasma glucose =<200mg/dl after a 75g glucose load

What is usual cause of acute bacterial prostatitis? >35? <35?

>35 yo: Gram-negative bacteria: E.Coli <35 yo: Chlamydia, mycoplasma, gardnerella

hepatitis? Fecal-oral, self limiting, small RNA virus, low mortality rate, iz available, postexposure prophylaxis w/ IZ & IG, no sequelae, public health department should be notified of outbreak

A incubation: 15-50 days w/avg of 28 Age acquiring hep A: by age 5 in developing countries with impure water, 20-39 50% of cases in US, more than 70% of children < 6 have few or no s/s w/ HAV killing it: heat sensitive virus, killed with heating food to 185 degrees for 1 minute, adequate chlorination of water, capable of surviving on select surfaces for many weeks, proper hand hygiene & using 1:100 bleach solution can minimize spread

What best describes the presentation of a patient with OA: worst symptoms in weight-bearing joints ______ in the day

later

Anthralin (Drithocreme) is helpful in treating psoriasis because it has what kind of activity? A. antimitotic B. exfoliative C. vasoconstrictor D. humectant

A. antimitotic -anthralin (Drithocreme) is a topical agent. -more expensive than corticosteroids -should be reserved for corticosteroid-resistant conditions

You see a 36-year-old man with no chronic health problems who presents with two furuncles, each around 4 cm in diameter, on the right anterior thigh. These lesions have been present for 3 days, slightly increasing in size during this time. He has no fever or other systemic symptoms. You advise the following: A. incision and drainage of the lesion B. a systemic antibiotic empirically C. a topical antibiotic D. aspiration of the lesion contents and prescription of a systemic antibiotic based on culture results

A. incision and drainage of the lesion

Patients with gluteus medius or deep trochanteric bursitis typically present with: A. increased pain from resisted hip abduction. B. limited hip ROM. C. sciatic nerve pain. D. heat over the affected area.

A. increased pain from resisted hip abduction.

Which of the following is not part of the criteria for an older woman to cease having any future Pap tests performed? A. over 55 years of age B. negative screening results on three consecutive cytology or two consecutive co-test results within 10 years C. the most recent cytology occurring within the past 5 years D. no history of cervical intraepithelial neoplasm (CIN) 2 or greater within the past 20 year

A. over 55 years of age

Clinical presentation of acute lower-extremity atherosclerotic arterial disease most likely includes: A. pain and paresthesia B. pallor and pulselessness C. poikilothermy D. paralysis or loss of limb strength

A. pain and paresthesia

What DM Medication? [ACTION] Delays intestinal carbohydrate absorption by reducing postprandial digestion of starches and disaccharides via "enzyme action inhibitors". DOES NOT ENHANCE INSULIN SECRECTION OR SENSITIVITY [EFFECTIVENESS] A1C reduction of 0.3-0.9% w/ intensified use. ***TAKE WITH 1ST BITE OF MEAL*** ***helpful in mgmt of postprandial hyperglycemia*** [COMMON SE] ***GI upset!!!*** [SAFETY] NR [CONTRAINDICATION] -IBS -Impaired renal function

ALPHA-GLUCOSIDASE INHIBITORs acarbose (Precose) miglitol (Glyset)

Which is more specific of liver function.... AST or ALT

ALT AST is found in the liver, cardiac & skeletal muscles and lungs.

Most sensitive marker for lupus (95%) RA (only 30-50%)

ANA

Labs for RA? What changes are seen on joint aspiration with RA?

ANA (positive in 1/5 patients) ESR (elevated) Rheumatoid factor 1. Inflammatory changes 2. WBC

A 2-month-old uncircumcised infant has been brought to the clinic for a well-baby checkup. How would the nurse proceed with the genital examination?

ANS: Avoid retracting the foreskin until the infant is 3 months old. If uncircumcised, then the foreskin is normally tight during the first 3 months and should not be retracted because of the risk of tearing the membrane attaching the foreskin to the shaft. The other options are not correct.

Pregnant woman who is in the late third trimester of pregnancy complains of sudden onset of vaginal bleeding accompanied by a contracted uterus that feels hard (hypertonic) and is very painful. Associated with a sudden onset of dark red-colored vaginal bleeding. Up to 20% of women do not have vaginal bleeding (blood is trapped between placenta and uterine wall). If mild, blood is reabsorbed and affected area reimplants. Severe cases cause hemorrhage; fetus must be delivered to save mother's life. Higher risk in females with history of hypertension, preeclampsia/eclampsia, cocaine use, or history of abruptio placentae.

Abruptio Placentae

Talar tilt

Assess for ligamentous Instability (Calcaneofibular Ligament) of the ankle Move foot into adduction testing for calcaneofibular ligament. Move foot into abduction testing for deltoid ligament| (+) TEST: if excessive adduction or abduction occurs and/or pain is noted

Exacerbation of emphysema is treated with

Bactrim doxy or ceftin BID x10day (mild) augmentin or resp quinolones (Avelox, levaquin) (severe) medal dose pack (mild and severe)

Which of the following is often found with rotator cuff tendonitis? A. osteoarthritis B. tendon rupture C. bursitis D. joint effusion

C. bursitis

Hepatitis B - What does it mean if: (+) HBsAg (-) anti-HBs (-) anti-HBc IgM (+) anti-HBc IgG

Chronic Hepatitis B

No controlled study in humans available • Studies in animals have revealed adverse effects on the fetus. - Embryocidal - Teratogenic - Other Approximately 2/3 of all medications • Select antibiotics - Clarithromycin (C=C) - Fluoroquinolones (ciprofloxacin=C) • -floxacin suffix • Many drugs used to treat serious mental and physical health problems - Most SSRIs, atypical antidepressants what category?

Category C= Caution

all of the following pharmacological interventions are used in the txment of pts with bulimia nervosa except: a. fluoxetine (prozac) b. desipramine (norpramin) c. bupropion (wellbutrin) d. paroxetine (paxil)

c. bupropion (wellbutrin)

Coombs test

Coombs test: detects presence of Rh antibodies in the mother (indirect Coombs test) and the infant (direct Coombs test). This test is done as part of the labs done early in pregnancy. ■ RhoGAM 300 mcg IM fi rst dose is at 28 weeks. ■ Give second dose within 72 hours (or sooner) after delivery. ■ RhoGAM decreases the risk of isoimmunization of the maternal immune system by destroying fetal Rh-positive RBCs that have crossed the placenta.

CLASSIC CASE: Complains of acute onset of severe eye pain and keeps the affected eye shut. Reports feeling something in the eye. Has increased tearing of affected eye. Depending on the cause (i.e contact) there is a high risk of bacterial infection *Use fluorescein dye with black lamp in dark room* -linear lines

Corneal abrasion R/O penetrating trauma,vision loss, soil/dirt, check vision Flush eye with NS to attempt to remove foreign body - if unable to remove, refer. Corneal abrasion- ophthalmic abx (erythromycin/Polytrim) for 3 to 5 days. Avoid patching the eye. F/u in 24hrs, refer if not improved.

a typical description of sciatica is: a. deep and aching b. worse at nighttime c. burning and sharp d. worse with coughing

c. burning and sharp

most episodes of low back pain are caused by: a. an acute precipitating event b. disk herniation c. muscle or ligamentous strain d. nerve impingement

c. muscle or ligamentous strain

Undescended testicles that remain in the abdominal cavity. Can affect both or only one testicle. Markedly increases the risk of testicular cancer. Usually corrected during infancy.

Cryptoorchidism

Hyperextension or backward curvature of the knees

Genu recurvatum

Potent H1 inhibitor Weight gain common Major side effect is sedation that is worse in lower doses. Little sexual dysfunction or gastrointestinal side effect Effect likely due to increase in central noradrenergic and serotoninergic activity Selectively stimulates 5HT1A while blocking 5HT2 and 5HT3 Higher doses more receptor siteselective and associated with fewer side effects

Mirtazapine (Remeron) [Tetracyclic antidepressant]

FIRST LINE Action: Decreases gluconeogenesis and decreases peripheral insulin resistance RARE to cause hypoglycemia Preferred for obese pts - aids in weight loss Contraindications: Renal disease, hepatic disease acidosis, ETOHs, hypoxia Labs: Monitor renal fx & LFTs Increased risk for lactic acidosis = During hypoxia, hypoperfusion and renal insufficiency IV Contrast: Stop metformin x1 day before - continue 48 hrs after dye ONLY if ceratinine is normal - otherwise hold until it becomes normalized

Metformin (glucophage)

the preferred method of preventing NSAID-induced gastric ulcer?

Misoprostol (cytotec)

A patient with type 2 diabetes comes to the clinic after reading about Metformin. Which of the following conditions that the patient also has would be a contraindication to taking Metformin? a. ketoacidosis b. cirrhosis or alcoholic patient c. hypoglycemic episodes d. all of the aboved.

all of the above

Nausea, anorexia, mild fever, malaise, abdominal pain, (most common) and jaundice, clay colored stools, dark colored urine, and joint pain, aversion to cigarette smoke, arthritis-like s/s & skin rash

Nausea, anorexia, mild fever, malaise, abdominal pain, (most common) and jaundice, clay colored stools, dark colored urine, and joint pain, aversion to cigarette smoke, arthritis-like s/s & skin rash

which antithyroid medication would you use in a pregnant women and why?

PTU, because it is less likely to cross the placenta pg 843

Reduce gastric secretion. Rx 4-8 weeks if typical symptoms. If symptoms return, then endoscopy. o Omeprazole (Prilosec®) o Lansoprazole (Prevacid®) o Rabeprazole (Aciphex®) o Esomeprazole (Nexium®) o Pantoprazole (Protonix®)

PPIs

what are the two antithyroid medications used today?

PTU propylthiouracil MMI methimazole pg 843

Psoas sign evaluates for

Pain elicited by extending the hip with the knee in full extension, seen with appendicitis and psoas inflammation

S/S of epididymitis?

Pain, dysuria, urgency, frequency, low back or perineal pain, fever, chills, malaise *unilateral scrotal edema

antiprostaglandin drugs cause stomach mucosal injury primarily by: a. a direct irritative effect. b. altering the thickness of the protective mucosal layer c. decreasing peristalsis d. modifying stomach pH level

b. altering the thickness of the protective mucosal layer

Hoshimoto's thyroiditis is often linked with other autoimmune disorders such as

SLE pernicous anemia RA DM Sjogren's syndrome

What DM Medication? insulin secretagogue (enhance release) -adjust dose w/renal impairment -intended for those you are non-obese/mild obese -can cause weight gain -possible drug allergy -can cause severe hyperglycemia, especially in older adults -can cause weight gain -A1C reduction of 1-2% with intensified use. -Typically less effective after 5 yrs of use. (b/c they require functioning beta cells)

SULFONYLUREAS glipizide (Glucotrol) glyburide (Diabeta) glimepride (Amary) best one ^most potent *sulfa allergy

Lateral curvature of the spine that is idiopathic and is most common in adolescence How do you evaluate What signals for referral surgery is indicated when... thoracic? lumbar?

Scoliosis Xray to determine degree curve Adams's Forward Bend Test Scoliometry Refer -if any pain at all -degree greater than 25 degree thoracic region 40 degrees lumbar region 50 degrees

Hypoglycemia treatment:

[preferred treatment:] 15-20 g glucose if conscious. [other options:] 4 oz OG, reg soft drink, hard candy *Recheck glucose in 15mins post tx. When glucose normalized have pt eat meal/snack of complex carbs and proteins ****glucagon prescribed for patients with significant risk for severe hypoglycemia***

(alpha blockers) including tamsulosin (Flomax) can be helpful in improving the symptoms of BPH. The use of alpha blockers as a solo or first-line antihypertensive agent has been associated with higher than expected rates of ______ and _______

Stroke and heart failure

The nurse is examining a patient who has possible cardiac enlargement. Which statement about percussion of the heart is true?

Studies shows that percussed cardiac borders do not correlate well with the true cardiac border.

What is Estrogen's mechanism of action in oral contraceptives?

Suppress LH/FSH=Inhibits ovulation Alters endometrium=inhibits implantation Ovum transport is accelerated Luteolysis may occur as estrogen causes progesterone levels to fall

Female - breast buds w/ areolar enlargement Male - enlargement of scrotum and testes; scrotum roughens and reddens

Tanner II

Female - Areola and nipple project as secondary mound Male - penis enlarges in breadth and development of glans; rugae appear

Tanner IV

14 yr old with sparse pubic hair, scrotum reddened, no penis length. Tanner stage? Normal?

Tanner stage 2

-Jaw claudication (w/chewing), -fever -visual disturbance or transient monocular loss of vision or partial visual field defect in affected eye (amaurosis fugax) -pain in temple area *visual loss occurs in 15-20% of cases despite treatment *mean age is 72 diagnosis? mgmt?

Temporal arteritis (Giant Cell arteritis, GCA) ESR and CRP markedly increased ******Refer to ophthalmologist or the ED STAT***** ***need biopsy of temporal artery***** ***high dose steroids 40-60 of prednisone daily for several weeks*******

Which of the following should be the goal measurement in treating a person w/ DM and HTN? a. blood pressure less than 140/90 b. hemoglobin A1c equal to or > than 7% c. triglyceride 200-300mg/dL (11.1 to 16.6 mmol/L) d.HDL 35-40 mg/dl (0.9 to 1.03 mmol/L)

a. blood pressure less than 140/90

Triple Screen Test

The Triple Test combines the AFP, beta hCG, and estriol serum level values. The hormone level results are used in a formula to figure out the risk for a Down's syndrome infant. Diagnostic test for genetic anomalies is chromosome testing.

the drug buspirone (buspar) has: a. low abuse potential; b. significant antidepressant action c. withdrawal syndrome when dc, similar to benzodiazepine d. rapid onset of action

a. low abuse potential;

which of the following characteristics applies to type 2 diabetes? a. major risk factors are heredity and obesity b. pear-shaped body type is commonly found c. exogenous insulin is needed for control of disease d. physical activity enhances insulin resistance

a. major risk factors are heredity and obesity

hyperthyroidism peaks at what ages? a. 20 to 40 yrs b. 40-60 yrs c. 15 to 20 yrs d. no peaks, can affect at any age

a. 20 to 40 yrs pg 837

In contrast to most forms of arthritis, bursitis typically presents with an ______ (slow or abrupt) onset with focal tenderness and swelling.

abrupt

Tx for PMS and PMDD

aerobic exercise stress reduction balanced diet reduction in alcohol, caffeine, sugar, sodium adequate sleep stop smoking SSRIs (for PMDD) May also use continous BCPs x 3 mo, NSAIDs, and vitamin supplements (B6, E, Mg, Ca) R/O thyroid, depression, anxiety, PCOS, sleep disorders, endometriosis, substance abuse

In Crohn's disease and UC, oral ________________, including sulfasalazine (Azulfidine®) and mesalamine (Apriso®), are usually the first-line therapy and are equally effective.

aminosalicylates mesalamine (Apriso®), is usually better tolerated and can be used in the presence of sulfa allergy.

Hyperthyroidism, Hypothyroidism or Both -atypical presentation in elderly

both

NPH insulin last from? (in reference to eating)

breakfast to dinner

Signs of deficient progestin in BCP

breakthrough bleeding (not enough endometrium stabilization) androgenic effects (hirsutism, acne, oily skin, edema, incr. libido)

colonic diverticulosis is

bulging pockets in the intestinal wall, commonly the sigmoid

which of the following best represents the peak ages for occurrence of acute appendicitis? a. 1 to 20 yrs b. 20 to 40 yrs c. 10 to 30 yrs d. 30 to 50 yrs

c. 10 to 30 yrs

which HPV types are most likely to cause anorectal carcinoma? a. 1 and 3 b. 6 and 11 c. 16 and 18 d. 72 and 81

c. 16 and 18

in a healthy person, what percentage of the body's total daily physiological insulin secretion is released as basally? a. 10% b. 25% c. 50% d. 75%

c. 50%

which of the following SSRIs is most likely to significantly interact with warfarin? a. citalopram b. paroxetine c. fluoxetine d. sertraline

c. fluoxetine

in order to determine how much T4 replacement a pt needs to re-establish a euthyroid state, the NP considers: a. the TSH value b. the patients's T4 c. the pt's body weight d. the pt's gender

c. the pt's body weight

Balanitis (inflammation of glans of penis)Occurs in about 1 /4 of all male sex partners of women with _______vaginitis. Can also occur in presence of immunosuppression systemic antimicrobial use, or DM.

candida Candida (40%), other causes: Group B streptococcus, Gardnerella

Which medications reduce efficacy of oral contraceptives?

carbamazepine/Tegretol phenytoin/Dilantin rifampin warfarin insulin oral hypoglycemics

contemplation

considering change and looking at the positives and negatives of the change

in the report of a thyroid scan done on a 48 y/o woman with a thyroid mass, a "cold spot" is reported. This finding is most consistent with: a. autonomously functioning adenoma b, Graves' disease c. Hashimoto's disease d. thyroid cyst

d. thyroid cyst

Management of carpal Tunnel

elevation, occupational splinting, NSAIDS, inject with corticosteroids, refer for surgical intervention

How do you describe a prostate with BPH

firm smooth symmetrically enlarged prostate

____________ is no longer recommended for gonorrheal infection.

fluoroquinolone use cephalosporin i.e Rocephin

Genetic syndrome in males or females [males] large forehead, ears, prominent jaw, avoid eye contact, large testicles, large body habitus, learning and behavioral differences [females] less common, fewer prominent findings most common known cause of autism in either gender

fragile X syndrome

Ortolani Test

good for up to age 1 diagnostic test involves abducting the thighs to test for hip subluxation or dislocation, if the femoral head can be felt to slip forward into the acetabulum on pressure from behind it is dislocated and positive evaluates hip dyplasia

In BPH Prolonged obstruction can lead to _________ and compromised renal function; this is the etiology of _______, a potentially life-threatening condition.

hydronephrosis postrenal azotemia

Hyperthyroidism, Hypothyroidism or Both -frequent, low-volume, loose stools

hyper

identify the following findings as associated with hyperthyroidism, hypothyroidism, or both: heat intolerance, smooth, silky skin, amenorrhea, or oligomenorrhea, hyperreflexia with a characteristic "quick out-quick back" action at the patellar reflex, proximal muscle weakness, tachycardia with hypertension

hyperthyroidism

Celiac disease treatment

life long gluten free diet is the ONLY treatment -NO wheat, barely, rye -Refer to knowledgeable dietician -avoid dairy permanently or temporarily until intestinal symptoms subside - dietary supplements w/ intestinal flares

monosaturated fats (4) examples decreases risk for

olive oil, canola oil, almonds/walnuts, sunflower oil/seeds (Mediterranean diet- high in monounsaturated fats) heart disease

Skin Lesion: Example: acne, nevi (moles) -palpable solid lesion up to 0.5cm

papule

Increased tactile remits in a pt makes you suspect

pneumonia

Progesterone challenge test: What findings constitutes sufficient estrogen?

pt bleeding after receiving

Term describing patterns of skin lesions lichenification=

skin thickening

microcephaly, abnormal head shape flat nose, appearance of hypertelorism (wide-set eyes) protruding tongue d/t hypotonia inner epicanthial folds upward-slanting eyes short broad hands/fingers; single palmar crease delayed growth & development hypotonia Brushfield spots

trisomy 21 (Down Syndrome)

Gluten foods

wheat, spelt, kamut, rye, barely, oats, bread, cereals, pasta, cookies and cakes

how is tetany, dyrhythmias, or seizures tx with hypocalcemia?

with calcium gluconate (94 mg of elemental calcium) in bolus and maintenance infusion pg 831

Secondary gout can be caused by all of the following conditions except: A. psoriasis. B. hemolytic anemia. C. bacterial cellulitis. D. renal failure.

C. bacterial cellulitis.

a characteristic of rheumatoid arthritis not typical in osteoarthritis is: a. wt loss b. morning stiffness c. symmetrical joint involvement d. the presence of Bouchard's nodes

c. symmetrical joint involvement

Contraindication for use of 5-Ht-1 agonist: sumatriptan (Imitrex)

-1st rule out CVD -Don't use if hx or signs of ischemic heart disease, CVA, TIA, uncontrolled HTN or hemiplegic migraine -Dont combine with ergots within 24hrs (ergot very potent vasoconstrictor) -Dont start within 2 weeks of MAOIs

osteoporosis t score =

-2.5 or less

GRAVE'S DISEASE TREATMENT

-Propylthiouracil (PTU) - shrinks gland & decreases hormone production -Methimazole (Tapazole) - shrinks gland & decreases hormone production -Betablockers - propranolol / atenolol 25mg / day SE= skin rash, granulocytopenia, hepatic necrosis (monitor CBC and LFTs)

REMEMBER AT THE BEGINNING OF THE TEST QUESTIONS ARE HARDER....BREATHE AND READ THEM THOROUGHLY "BACKWARD READING" -read last sentence first (stem) of a long question and case scenarios so that you know ahead of time what the questions is asking for then go back and read from the beginning.

...

Which of the following agents offers an intervention for the control of tremor and tachycardia associated with alcohol withdrawal?

...

an adolescent female has had normal menses for almost 2 yrs. She has not had menses in 3 months. She is dx with polycystic ovarian syndrome. What else is a common finding? a. obesity b. elevated insulin levels c. positive pregnancy test d. elevated blood pressure

...

prostaglandin analogue thats specifically designed for gastric protection w/ NSAID. This medication is possibly helpful in minimizing renal injury secondary to NSAID use.

...

NPH (Novolin N) (Humulin N) *intermittent acting* onset: peak: duration:

1-2 6-14 16-24

NPH (Novolin N) (Humulin N) *intermittent acting* onset:

1-2hrs

Absolute contraindications with IUD placement?

1. Active, recent or recurrent pelvic infection, such as GC/Chlamydia, 2. Pregnancy

When is stiffness with OA? Is it worse with activity or rest?

1. Better in the am, worse as the day progresses, 2. Worse with activity, relieved by rest

The mechanism of action for "triptans" in patients who have migraine headache is: 1. cerebral artery constriction. 2. cerebral artery dilation. 3. prevention of arterial spasm. 4. reduction of intraarterial inflammation

1. cerebral artery constriction.

How much daily calcium is recommended for women older than 50 years of age?

1200mg

RA involves at least _______ joint groups, and can recur after a period of inactivity

3

AANP Frail eldery are those aged

85+

Kleinfelters

A genetic disorder in males caused by having an extra x chromosome. (XXY) Males with this disorder may have: ****gynecomastia***l-lack facial/body hair -rounded body type -small testicles -Public hair grow similar to female -IQ lower than avg (15) -They may learn to speak much later than other children and may have difficulty learning to read and write. Increases the risk of developing testicular cancer.

mulder test

A test for Morton's neuroma. Done by grasping the first and fifth metatarsals and squeezing the forefoot. Positive test is hearing a click along with a patient report of pain during compression. Pain is relieved when the compression is stopped.

The mother of a 10-year-old boy asks the nurse to discuss the recognition of puberty. The nurse should reply by saying:

ANS: "The first sign of puberty is enlargement of the testes." Puberty begins sometime between ages 9 1/2 and 13 1/2 years. The first sign is enlargement of the testes. Next, pubic hair appears and then penis size increases.

What do these labs indicate? HbsAg-positive IgM& Anti-HBc-positive Anti-HBc-postive Anti-HBs-negative

Acutely infected with Hep B

Bartholin Gland Cyst and Abscesses

Age 20-29 Usually mixed flora: Gonorrhea, Chlamydia, E.coli Very painful vulvar mass at 4 and 8 o'clock of introitus Tx: sitz baths, I/D, Word catheter placement Rocephin + clindamycin x 7 Refer to surgeon for excision to r/o carcinoma if >40 If rapidly growing (over 24 hr), likely MRSA

Which of the following statements about tendonitis is false? A. Tendonitis is typically the result of overuse. B. Tendonitis is the result of a macroscopic or partial tear of the tendon. C. Acute pain results when firm pressure is applied to the tendon. D. Signs of tendonitis include reduced ROM caused by stiffness and discomfort.

B. Tendonitis is the result of a macroscopic or partial tear of the tendon.

-reduces hepatic glucose production and intestinal absorption **insulin sensitizer** via increased peripheral glucose uptake and utilization A1C reduction = 1-2% with intensified use *requires monitoring of Cr lvl **CONTRAINDICATIONS** -IMPAIRED RENAL FX -HEART FAILURE -PPL >80yo ***Safety considerations*** There is a risk for lactic acidosis with this medication, so if needs to be held prior to receiving contrast and 48hr after. Cr MUST be checked BEFORE restarting

BIGUANIDE glucophage (Metformin) only drug in it's class

Candidal infection of the glans penis; more common in uncircumcised men, diabetics, and/or immunocompromised males. Treated with topical OTC azole creams. If partner has candidiasis, treat at the same time.

Balanitis

good for 6 months Flex infants hips and knees to 90 degrees, place thumb along medial thigh, apply pressure in a posterior direction while adducting the femur. This maneuver will force the head out of the socket. evaluates hip dysplasia

Barlow Test

Loss of posterior tibial reflex often indicates a lesion at: A. L3. B. L4. C. L5. D. S1.

C. L5.

(+) HBsAg, anti-HBc, anti-HBe, IgM, IgG

CHRONIC Hep B

Which supplement has the highest amount of elemental calcium?

Calcium carbonate

chronic, genetically linked, autoimmune disorder - alleles code for HLD-DQ2 or DQ8 proteins

Celiac Disease

Sequele of DVT and or leg trauma.. 6 P's Pain Parasthesia Pallor Pulselessness, Poikilothermy (temp vary) paralysis

Chronic venous insufficiency

What do these labs indicate? HbsAg-(+) IgM &Anti-HBc-(-) Anti-HBc-(+) Anti-HBs-(-)

Chronically infected

Hale's Lactation Risk Category • L1 - Safest

Controlled study=Fail to demonstrate risk • Acetaminophen • The penicillins • Medroxyprogesterone acetate (Depo-Provera®) given at recommended time postpartum

Name 4 DMARDS given with RA?

Corticosteroids Methotrexate Antimalarials Gold Salts

Dietary sources of calcium?

Dairy, sardines, salmone, green leafy, tofu

What is Cause of urge incontinence?

Detrusor overactivity CNS abnormalities (stroke) Infections of the GU tract Urinary stones Neoplasm

Pseudocyst in pancreatic cancer is bothersome and growing, what is the intervention?

Draining it via endoscopic ultrasound guided fine needle aspiration. if large, must be removed surgically

AST > ALT hepatitis or ETOH/drugs?

ETOH/drugs Acetaminophen, Statins, Tequila

CLASSIC CASE: Classic lesion is an expanding red rash with central clearing that resembles a target. The "bull-eye" usually appears within 7-14days after a deer tick bite, but can range from 3-30 days. The rash feel hot to touch and has a rough texture. Common locations are the belt line, axillary area, behind the knees and in the groin area. It is accompanied by flu-like symptoms. The lesion spontaneously resolve within a few weeks. Pt education would involve use of DEET repellent on clothes and skin.

Erytheme Migrans (Early Lyme Disease)

First line of treatment for osteoarthritis:

Exercise

True or False: CA-MRSA is most commonly spread from one person to another via airborne pathogen transmission

False

True or False: If a skin and soft tissue infection does not improve in 48 to 72 hours with antimicrobial therapy, infection with a resistant pathogen is virtually the only cause

False

Who is most likely to get Crohns?

Female>Male Cauacasian 15-25 y.o 55-65 y.o Jewish

Alpha-glucosidase inhibitors are acarbose (precose) and miglitol (glycet) and are taken with ________.

First bite of meal

Homan's sign significance.

Forced dorsiflexion of affected foot causes pain in calf, indicative of PE

retrosternal burning, bitter taste in mouth, belching, excessive salivation, at night, relieved by sitting up, antacids,

GERD consider referral for EGD. R/O cancer, Barrett's esophagus , PUD, ect.

Major risk factors for ___________ formation include age older than 50 years, female gender, obesity, hyperlipidemia, rapid weight loss (including patients who have undergone bariatric surgery), pregnancy, genetic factors, European or Native American ancestry, and ingestion of a diet with a high glycemic index.

Gallstone

H2RAs don't offer protection against? Duodenal or Gastric ulcers

Gastric ulcer PPIs provide better protection

Knock-knees " together

Genu valgum To remember valgum, think of "gum stuck between the knees"

what is the most common cause of spontaneous hyperthyroidism in the U.S.

Grave's disease pg 837

duodenal ulcer is mainly caused by

H. Pylori cork-screw organism gramm (-)

Unvaccinated persons should receive ____ and hepatitis B vaccine as soon as possible, preferably within 24 hours, after exposure.

HBIG

Only individuals with ___________________ are at risk for Hepatitis D virus

HBV

Hashimoto thyroiditis is a form of ___________thyroidism (hypo or hyper), that results from loss of thyroid gland function due to anti-thyroglobulin and anti-________________ antibodies against the TSH receptor

HYPOthyroidism, anti-PEROXIDASE

Anti-HAV (+) means

Had HepA in the past or is immunized against it *Cant get HepA again*

• HBsAg (surface antigen) = infection ***Antigen = infection; they both end in "n"*** • Anti-HBs (surface antibody) = immunity *** Antibody = immunity; they both end in "y"***

Hepatitis B Markers

S/S of excessive androgenic effects

Hirsutism Acne Oily skin Edema Increased libido

CLASSIC CASE: Complains of an itchy eyelid and an acute onset of a pustule on either upper or lower eyelid that eventually becomes painful. Initial tx =warm compress BID/TID until pustule drains Infection=ABC

Hordeolum (Stye)

SGLT2 (Sodium glucose cotransporter 2) I.e invokana, Farxiga, and jardiance have a risk of _____________

Hypoglycemia

which of the following statements is most consistent with IBD, IBS, or both conditions? involvement can be limited to intestinal mucosa only, or the full thickness of the intestinal wall can be involved

IBD

which of the following statements is most consistent with IBD, IBS, or both conditions? potential complications include fistula formation and perineal disease

IBD

which of the following statements is most consistent with IBD, IBS, or both conditions? potential complications include increased risk for colonic malignancy

IBD

there are four typical bowel patterns with irritable bowel syndrome: predominantly diarrhea - small volumes of loose stools, volumes < than 200ml without nocturnal diarrhea, often aggravated by emotional stress or eating with passage of lg amts of mucus

IBS-D

What is cause of stress incontinence?

Impaired muscle support Sphincter deficiency

In assessing a 70-year-old man, the nurse finds the following: blood pressure 140/100 mmHg; heart rate 104 and slightly irregular, split S2. Which of these finding can be explained by expected hemodynamic changes related to age?

Increase in systolic blood pressure

morton's neuroma

Inflammation of the digital nerve of the foot between the third and fourth metatarsals. Increased risk with high-heeled shoes, tight shoes, obesity, dancers, runners.

ACL or PCL tear, assess with

Lachman's test - ACL/PCL tears (Lachman=Ligament-LL) -Easy to perform on painful knee -With knee in 20-30 degree flexion, grasp leg with one hand with the anterior force to proximal tibia while the opposite hand stabilizes the thigh -aka 'Drawer sign

When the nurse is auscultation the carotid artery for bruits, which of these statements reflects correct technique?

Lightly apply the bell of the stethoscope over the carotid artery, and while listening, have the patient take a breath, exhale, and hold it briefly.

What are the most common knee tears?

Medial meniscus McMurray Test 1. Patient is supine-NP with hand on joint, bends knee to 90˚, applies pressure to knee both medially/laterally while straightening the leg. 2. An audible/palpable click or pain when the knee is raised = positive test

Normal frequency Bleeding is heavy and prolonged

Menorrhagia

knees are abnormally close and ankle space is increased typically by age 7, normal alignment evolves common (preschool) Has full ROM No pain distance between medial malleoli is >3 inches Xray after 7 or if unilateral

Menu Valgum (knock-knee) think "knees stuck together like gum"

Pharmacologic management of Crohns

Mesalamine suppositories Antibiotics as needed (Flagyl) Folate if taking sulfasalazine Antispasmotics/antidiarrheals

Diabetic Rx that cause weight loss

Metformin, GLP-1, SGLT-2

Anemia is a common problem in IBD; its etiology is often from multiple causes. Iron-deficiency anemia, manifesting as a ____________ ____________ anemia, occurs as a result of chronic blood loss.

Microcytic hypochromic

A 30-year-old woman with a history of mitral valve problems states that she has been "very tired". She has started waking up at night and feels like her "heart is pounding." During the assessment, the nurse palpates a thrill and lift at the fifth left intercostal space midclavicular line. In the same area the nurse also auscultates a blowing, swishing sounds right after S1. These findings would be most consistent with:

Mitral regurgitation

____________ agents give 2 hrs apart from other meds Bismuth subsalicylate (pepto): has direct antibacterial action against H. Pylori, Promotes prostaglandin production/stimulates gastric bicarbonate

Mucosal Protective Agents:

Which of these finding would the nurse expect to notice during a cardiac assessment on a 4-year-old child?

Murmur at second left intercostal space when supine

Altered mental status, hypothermia, hyponatremia, and hypoventilation are often seen in ________________, which is severe hypothyroidism.

Myxedema Coma

Major adverse effect of GLP-1 agonist (incretin mimetics I.e exenatide/ Byetta , liraglutide/victoza include _____ and ________ and are contraindicated in

N/V Gastroparesis

Fracture Type? Wrist pain on palpation of the anatomic snuffbox. Pain on axial loading of the thumb. History of falling forward with outstretched hand (hyperextension of the wrist) to break the fall. associated with high risk of avascular necrosis and nonunion. ****Key terms**** -pain on side of thumb -spica splint -avascular necrosis & nonunion.

Navicular Fracture (Scaphoid Bone Fracture) Initial x-ray of the wrist may be normal, but.... ***repeat x-ray in 2 weeks will show the scaphoid fracture (due to callus bone formation). High risk of avascular necrosis and nonunion. Splint wrist (thumb spica splint) & refer to hand surgeon.

Fasting glucose criteria

No caloric intake for a period of at least 8 hours -can have water and non-caloric beverages

What do these labs indicate? HbsAg-negative Anti-HBc-negative Anti-HBs-negative

No infection, No immunity

Elevation in Gamma Glutamyl Transaminase (GGT) with elevation of AST/ALT is a possible sign of?

Occult alcoholism Leik p274

Specific Headache Clues: Visual field defect

Optic Pathway Lesion (pituitary tumor)

Murphy's sign: deep pain on inspiration while fingers placed under right rib cage RUQ tenderness, rebound pain

Physical findings of cholecystitis

inflammation of unknown origin. that affects the muscles and the joints. Typically affects ppl >50. presentation usually starts with pain in the shoulder first then neck/upper arms/lower back/hips/thighs Symptoms can appear over days or weeks pain is worse in the am and improves during the day No specific test to diagnose, but lab work will reveal elevated CRP and ESR

Polymyalgia rheumatica (PMR) Treated with low dose corticosteroids (10-15mg/day) until symptoms relieved (avg 2-3 wks) followed by tapering the dose to find the lowest dose necessary to suppress symptoms Treatment can continue up to 2-3 years

CLASSIC CASE: Most commonly seen in elderly patients, especially those of African background or diabetics. Usually asymptomatic. Gradual changes in peripheral vision (lost 1st) and then central vision. Cupping on fundoscopic. Blindness is a complication. Requires referral

Primary open angle glaucoma. IOP high Normal 10-22mmHg Need BB eyedrops- timolol (Betimol)

5-alpha reductase inhibitor. prostate shrinks by 50% while on it category X drug. Teratogenic. Should not be touched with bare hands if reproductive-aged female (adversely affects male fetus).

Proscar 5-alpha reductase inhibitor.

erythropoietin is needed for ______production and is produced by the _____

RBC kidneys when in renal failure, thats why so many patients have anemia and take procreate

What skin problems may arise in the early stages of acute hepatitis B?

Rashes and arthritis

Diagnostics with epididymitis?

STD testing Urine culture Scrotal Ultrasound

an inflammatory condition that results in the production of noncaseating granulomas in various sites of the body, predominantly in the lungs, lymph nodes, eyes, and skin.

Sarcoidosis

Sarcoidosis manifests in what parts of the body Mainstay treatment is

Sarcoidosis manifests in what parts of the body corticosteroids

Which autoimmune diseases have been linked to primary biliary cirrhosis?(6)

Scleroderma, Raynaud's syndrome, autoimmune thyroiditis, celiac disease, Sjogren's syndrome

CLASSIC CASE: Report of daily symptoms of dry eyes/mouth for several months (>3m). Complains of chronic dry eyes and that both eyes have sandy or gritty sensation (keratoconjunctivitis sicca). Dry mouth, on examination the salivary glands or swollen and inflamed. Labs and Referral to follow

Sjogren's Syndrome OTC tear substitute eye gtts TID. Refer to ophthalmologist (keratoconjunctivitis sicca) Labs: CBC, ESR,CRP, and autoimmune panel. Refer to rheumatologist for management.

What do you do if three pills is missed?

Start over Use back up Buy a crib!

Action: Stimulates beta cells to secrete insulin Adverse: Hypoglycemia, Photosensitivity Blood dyscrasias (disordered state) - monitor CBC Avoid w/ impaired renal or hepatic fx Can cause weight gain D/c when insulin started

Sulfonylureas Second generation: Glipizide (Glucotrol) max 40mg daily, glyburide (Diabeta) max 20mg daily, glimepiride (Amaryl) max 8mg daily

(T/F) As few as 3 days of malnutrition in the form of inadequate protein-calorie intake can impair normal wound -healing mechanisms.

T

(T/F) During the first 6 weeks of the psotpartum period, the childbearing woman is at increased risk for venous thrombus formation.

T

What is LAM method?

The idea that breastfeeding may delay ovulation/menstruation for six months.

self-limiting inflammation of the hip, most likely due to a viral or immune cause occur in children age 1-15 (common in children between age 2-6) Painful limo unilateral involvement internal rotation = hip spasm

Toxic synovitis

S/S of BPH? Physical exam with BPH? How does the prostate palpate?

Urgency, frequency, dribbling, retention Bladder distension Non-tender prostate Smooth rubbery prostate

swelling of the uvea, the middle layer of the eye that supplies blood to the retina (refer to ophthalmologist). Patient treated with high-dose steroids for several weeks.

Uveitis:

Long-term PPI use is linked to

Vitamin B12, Ca, Mg, and Fe malabsorption as well as fracture. There is no recommendation to supplement with vitamins.

Hypo pigmentation patches of skin with irregular shapes. Located on any part of body, seen better on darker skins

Vitiligo

Normal Lab Changes in Pregnancy

WBC elevates up to 16,000 with neutrophilia Hgb and Hct decrease Alk phos incr. due to fetal bone growth ESR incr Total T3 and T4 incr. due to incr. binding globulin (TSH and free levels unchanged) Incr. lipids Incr. GFR

Per the ADA, when do you do ANNUAL screening for T2DM?

When BMI is >25kg/m2 and has 1 or more RFs for DM

When is pain exacerbated with carpal tunnel syndrome?

With Dorsiflexion (up)

• Isotretinoin (Accutane®) - ~33% increase • Thalidomide - ~25% increase • The statins - 3- to 4-fold increase in congenital anomalies, "congenital statin syndrome" not yet identified category?

X • Animal or human studies have demonstrated fetal abnormalities • Evidence of fetal risk based on human study • No therapeutic indication in pregnancy

SULFONYLUREAS glipizide (Glucotrol) glyburide (Diabeta) glimepride (Amary) best one ^most potent What is their action? A1C reduction? Effectiveness? Are the affordable? Safety issues? Contraindications? Elimination?

[ACTION] insulin secretagogue (enhances release) A1C reduction of 1-2% with w/ intensified use. [EFFECTIVENESS] -typically less effective after 5 years of use because they require functioning pancreatic beta cells to be effective [COST] - on the $4 list (cheapest) [SAFETY] -can cause severe hyperglycemia, especially in older adults -adjust dose w/renal impairment -dc when started on insulin [CONTRAINDICATION] -none just use w/ caution is known sulfa allergy [DRUG ELIMINATION] -renally eliminated, hence the the need adjust dose based on renal impairment.

An inflammatory and localized disorder of the penis that results in fibrotic plaques on the tunica albuginea. Results in penile pain that primarily occurs during erection, palpable nodules, and penile deformity (crooked penile erections). May resolve spontaneously or worsen over time.

[Peyronie's Disease] Labs ■ None. Clinical diagnosis. Plan ■ Refer to urologist

H/A... -Bilateral -Pressure -tightness band-like -waxes and wanes -Varies ***band-like*** *someone squeezing my head* ***can last for days***

[Tension] Treatment: NSAIDS, Acetaminophin, OTC (ASA+CAFFEINE) STRESS REDUCTION -YOGA -TAI-CHI EXERCISE REG EATING THERAPY

What should be avoided with calcium carbonate supplement?

^Fiber foods Aluminum containing antacids (Calcium+aluminum=bind)

**LEIK** [Bell's Palsy] -abrupt unilateral facial paralysis due to dysfunction of the motor branch of CN7. -linked to herpes, Epstein Barr, and zoster -paralysis can progress rapidly w/in 24hrs -skin sensation intact *tear production may stop **unable to close eyelid** [Etiology] viral infection, autoimmune, pressure from tumor/b. vessel [Mgmt] *R/O CVA/TIA mastoid infection, bone fracture, Lyme disease and tumor [Meds] HD corticosteroids x10d (wean) acyclovir (if suspect herpes) [Safety] *****protect cornea from drying and ulcerating with eye lubricant (am/pm)****** **use eye patch if pt is unable to fully close the eyelid** [Complication] corneal ulceration permanent facial weakness 10%

`

a pt presents with consistently elevated blood glucose before his evening meal. What choice below represents an insulin change that would improve his evening glucose? Current regimen: AM: 22 u intermediate-acting insulin, 12 u short-acting insulin PM: 10 u intermediate-acting insulin, 8 u short-acting insulin a. 24 units intermediate-acting insulin in AM b. 14 units short-acting insulin in AM c. 12 units intermediate-acting insulin in PM d. 10 units intermediate-acting insulin in PM

a. 24 units intermediate-acting insulin in AM

a diabetic pt with proteinuria (approx 1 g/d) has been placed on an ACE inhibitor. How soon can the anti-proteinuric effect of the ACE inhibitor be realized in this pt? a. 6-8 wks b. 3 months c. 6 months d. 3-5 yrs

a. 6-8 wks

what is the normal serum calcium values in adults? a. 9-11 mg/dL b. 6-8 mg/dL c. 13-14 mg/dl d. 0.2-.09 mg/dl

a. 9-11 mg/dL pg 830

a pt who is taking long acting insulin basal insulin has elevated blood sugars. Which blood sugars are important to review in order to increase the dose of insulin? a. AM fasting b. 2 hr post prandial c. pre-prandial d. bedtime

a. AM fasting long acting insulin mimics the amt of insulin the pancreas produces at a steady rate throughout the day/night. Adjustments in doses of long-acting insulin are typically based ont he AM fasting glucose values. the other blood sugars reflect blood sugars in relation to meals

which of the following is most likely to be reported in a pt on long-term use of a HMG-CoA reductase inhibitor (statin)? a. AST, 22 U/L; ALT, 28 U/L b. AST, 320 U/L; ALT, 190 U/L c. AST, 32 U/L; ALT 120 U/L d. AST, 440 U/L ALT, 670 U/L

a. AST, 22 U/L; ALT, 28 U/L

a female pt has the following characteristics. Which one represents the greatest risk factor for development of type 2 diabetes? a. BMI 26 b. lack of exercise c. Mediterranean decent d. lack of regular health care

a. BMI 26

a female pt has the following characteristics. Which one represents the greatest risk factor for development of type 2 diabetes? a. BMI 31 b. osteopenia c. Mediterranean decent d. hypothyroidism

a. BMI 31

T., age 4, has an apparent hypertrophy of the calf muscles, which seem doughy on palpation. his mother is concerned because T. is unable to raise himself from the floor without bracing his knees with his hands. what do you suspect? a. Duchenne muscular dystrophy b. cerebral palsy c. Legg-Calve-Perthes disease d. multiple sclerosis

a. Duchenne muscular dystrophy

you see an obese 25 y/o male with acanthosis nigricans and consider ordering: a. FBS b. LFT c. RPR d. ESR

a. FBS

a serological marker for acute hepatitis A virus (HAV) infection is: a. HAV IgM b. HAV viral RNA c. TNF-alpha d. IL-10

a. HAV IgM

you see a 27 y/o man who says he ate at a restaurant last week that was later reported to have a worker identified with Hepatitis A. He is healthy and shows no sign of infection but is concerned about contracting HAV infection. You recommend: a. HAV vaccine b. HAV immune globulin c. HAV vaccine plus immune globulin d. no intervention at this time and wait until sxs manifest

a. HAV vaccine

when discussing the use of immunoglobulin (IG) with a 60 y/o woman who was recently exposed to the hepatitis A virus, you consider that: a. IG is derived from pooled donated blood b. the product must be used within 1 week of exposure to provide protection c. its use in this situation constitutes an example of active immunization d. a short, intense flu- like illness often occurs after its use

a. IG is derived from pooled donated blood

Intervention in microalbuminuria for a person with DM includes: (choose all that apply) a. improved glycemic control b. strict dylipidemia control c. use of an optimized dose of an ACEI or ARB d. the use of an ACEI with an ARB

a. Improved glycemic control b. strict dylipidemia control c. use of an optimized dose of an ACEI or ARB -glycemic control -management of HTN SBP <140; DBP <90 -Treat dyslipidemia with statins *never use an ACEIw/ARB*

A newly diagnosed client with diabetes who has an HbA1c of 7.5 is started on therapeutic lifestyle changes (TLC) and medical nutritional therapy (MNT). Which oral antidiabetic agent is recommended as monotherapy? a. Metformin (Glucophage). b. Glipizide. c. Sitagliptin (Januvia). d. Exenatide (Byetta).

a. Metformin (Glucophage).

a 16 y/o c/o that his knees hurt. his mother states that he has c/o knee pain for the past 2 wks. he has a prominent tibial tubercle. what should be part of the differential dx? a. Osgood-Schlatte disease b. growing pains c. acute lymphocytic leukemia (ALL) d. psychogenic pain

a. Osgood-Schlatter disease

Alice, age 48, has a benign thyroid nodule. The most common treatment involves: a. Watchful waiting with an annual follow-up. b. Surgery. c. Administration of levothyroxine therapy. d. Radioactive iodine therapy.

a. Watchful waiting with an annual follow-up. quiz 624

The mechanism of action of exenatide (Byetta) is as: a. a drug that stimulates insulin production in response to increase in plasma glucose b. a product virtually identical in action to sulfonylureas c. a drug that increases insulin action in the peripheral tissues and reduces hepatic glucose production d. a facilitator of renal glucose excretion

a. a drug that stimulates insulin production in response to increase in plasma glucosee GLUCAGON-LIKE PEPTIDE 1 AGONIST (GLP) [incretin mimetics] exenatide (Byetta) (Bydureon) liraglutide (Victoza) [ACTION] 1.) Stimulates insulin production in response to increase in plasma glucose. 2.) Inhibits postprandial glucagon release. 3.) Slows gastric emptying, often leading to appetite suppression and weight loss [EFFECTIVENESS] -A1C reduction of 1-2% w/ intensified use. ***indicated to improve glycemic control in combo w/glucophage and or sulfonylureas*** [Common SE] -N&V [COST] -EXPENSIVE $300 mo "injection only" [SAFETY] **FDA advisory to monitor patient carefully for the development of pancreatitis.*** FDA advises clinicians to teach signs of acute pancreatitis and the need for emergent attention (persistent abdominal pain, usually accompanied w/vomiting) -Use cautiously in pts with mild-moderate renal impairment (CrCl 30-50) [CONTRAINDICATIONS] *Hx of pancreatitis *CrCl<30ml/min *Gastroparesis [ELIMINATION OF DRUG} -renally eliminated

lower GI hemorrhage associated with diverticular disease usually manifests as: a. a painless event b. a condition noted to be found with a marked febrile response c. a condition accompanied by severe cramp-like abdominal pain d. a common chronic condition

a. a painless event

clinical findings most consistent with appendiceal rupture includes all of the following EXCEPT: a. abdominal discomfort less than 48 hrs in duration b. fever > 102 F (38C) c. palpable abdominal mass d. marked leukocytosis with total WBC > than 20,000 mm3

a. abdominal discomfort less than 48 hrs in duration

clinical findings in pts with acute hepatitis B likely include all of the following EXCEPT: a. abdominal rebound tenderness b. scleral icterus c. a smooth, tender, palpable hepatic border d. report of myalgia

a. abdominal rebound tenderness

principles of management of genital herpes include which of the following? a. antiviral chemotherapy can control the signs and symptoms b. antiviral chemotherapy, if prescribed early in a first clinical episode is curative c. antiviral chemotherapy does not control recurrent episodes d. antiviral topical tx offers minimal clinical benefit

a. antiviral chemotherapy can control the signs and symptoms

the most appropriate time to begin screening for renal nephropathy in a pt with type 2 diabetes is: a. at diagnosis b. one yr after dx c. 2-3 yrs after dx d. 5 yrs after dx

a. at diagnosis

drug txment options for a pt with bipolar d/o often include all of the following EXCEPT: a. atomoxetine (strattera) b. lithium carbonate c. risperidone (risperdal) d. valproic acid (depakote)

a. atomoxetine (strattera)

the findings of a painless thyroid mass and TSH level of less than o.1 IU/mL in a 35 y/o woman is most consistent with: a. autonomously functioning adenoma b. Graves' disease c. Hashimoto's disease d. thyroid malignancy

a. autonomously functioning adenoma

a 35 y/o woman c/o of a 6 month hx of periodic heartburn primarily after eating tomato based sauces. her wt is unchanged and exam reveals a single alterned finding of epigastric tenderness without rebound. as first-line tx, you advise. a. avoiding the trigger foods b. the use of a prokinetic agent c. the addition of sucralfate with meals d. increased fluid intake with food intake

a. avoiding the trigger foods

a young adult female presents with vaginal discharge and itching. Besides trichomoniasis and yeast, what else whould be included in the differential? a. bacterial vaginosis b. chlamydia c. herpes genitalis d. syphilis

a. bacterial vaginosis

J. age 64 comes in for a visit. she has a cast on her rt arm and tells you that she has a comminuted fx of her radius. when she asks what that means, you tell her that in a comminuted fx the: a. bony fragments are in many pieces b. broken ends of the bone protrude through the soft tissues and skin c. bone breaks cleanly but does not penetrate the skin d. bone is crushed

a. bony fragments are in many pieces

a 75 y/o pt has osteoarthritis and pain. which of the following medications increases the risk of a GI related ulceration? a. celecoxib b. warfarin c. tramadol d. amitriptyline

a. celecoxib

the histamine2-receptor antagonist most likely to cause drug interactions with phenytoin and theophylline is: a. cimetidine b. famotidine c. nizatidine d. rantidine

a. cimetidine

QT prolongation is a concern with higher doses of: a. citalopram (celexa) b. sertraline c. venlafaxine d. fluoxetine

a. citalopram (celexa)

which of the following is most consistent with the presentation of a pt with ACUTE colonic diverticulosis? a. cramping, diarrhea and leukocytosis b. constipation and fever c. rt sided abdominal pain d. frank blood in the stool with reduced stool caliber

a. cramping, diarrhea and leukocytosis

which of the following results in a clinically insignificant increase in the prostate specific antigen (PSA)? a. digital rectal exam b. ejaculation c. prostatitis d. prostate biopsy

a. digital rectal exam

Mr. KY presents in the clinic with pain, tenderness, erythema, and swelling of his left great toe. The healthcare provider suspects acute gout. Which of the following should the healthcare provider suspect in the initial test results for this patient? a. elevated uric acid level b. elevated blood urea nitrogen (BUN) c. decreased urine pH d. decreased C-reactive protein (CRP)

a. elevated uric acid level pg 899 elevated wbc count, elevated temperature, elevated serum uric acid or normourecemia (acute phase, pg 900)

you consider prescribing insulin glargine (Lantus) because of its: a. extended duration of action b. rapid onset of action c. ability to prevent diabetic end-organ damage d. ability to preserve pancreatic function

a. extended duration of action

a first line approach to tx Cushing's syndrome in a 56 y/o woman who has been taking oral corticosteroids to tx rheumatoid arthritis for the past 2 yrs is: a. gradually tapering corticosteroid use b. referral for surgery c. consider radiation tx d. prescribe mifepristone

a. gradually tapering corticosteroid use

a common presentation of an inguinal hernia is: a. groin or abdominal pain with a scrotal mass b. an abdominal mass without pain c. scrotal and abdominal masses d. abdominal pain and scrotal erythema

a. groin or abdominal pain with a scrotal mass

v a 56 y/o man with a 60 pack-year cigarette smoking hx recent 5lb uninteded wt loss, and a 3 month hx of new-onset sxs of peptic disease presents for care. He is taking no meds on a regular basis and reports drinking approximately six 12oz beets per week with no more than 3 beers per day. PE is unremarkable except for mild pharyngeal erythema and moderate epigastric tenderness without rebound. the most helpful diagnostic test at this point in his evaluation is a: a. upper endoscopy b. barium swallow c. evaluation of H. pylori status d. esophageal pH monitoring

a. upper endoscopy

M. age 16 states that he feels like he has a bag of worms on his left scrotum. even before examining him, what do you suspect? a. varicocele b. hydrocele c. cystic nodules d. spermatocele

a. varicocele

which of the following best describes the presentation of a pt with OA? a. worst sxs in wt bearing joints later in the day b. symmetrical early morning stiffness c. sausage-shaped digits with associated skin lesions d. back pain with rest and anterior uveitis

a. worst sxs in wt bearing joints later in the day

which of the following is not an "alarm" finding in the person with GERD sxs? a. wt gain b. dysphagia c. odynophagia d. iron-deficiency anemia

a. wt gain

irritable bowel disease is a functional bowel disorder characterized by

abdominal pain or discomfort and altered bowel habits in the absence of detectable structural abnormalities

COPD medication order

anticholinergic beta agonist steroid

for pts with documented coronary heart disease, the american Heart Association advises intake of approximately ____ of eicosapentaenoic acid (EPA) and docosahexaenoic acid (DHA) per day, preferably from oily fish. a. 500mg b. 1 g c. 2 g d. 4 g

b. 1 g

sxs of lymphogranuloma venereum typically occur how long after contact with an infected host? a. 5-7 days b. 1-4 weeks c. 4-6 weeks d. 2-3 months

b. 1-4 weeks

in a person with obesity, weight loss of ___% or more yields an immediate reduction in death rates from cardiovascular and cerebrovascular disease: a. 5 b. 10 c. 15 d. 20

b. 10

approximately what percentage of sexually active adults has serological evidence of human herpesvirus 2? a. 5 b. 15 c. 25 d. 40

b. 15

in teaching a pt with type 2 diabetes about using rapid-acting insulin to help with the management of post-prandial hyperglycemia, the NP advises that the usual dose is____unit per 15 grams of carbohydrate a. 1 b. 2 c. 3 d. 4

b. 2

which of the following drugs is likely to be the most dangerous when taken in an overdose? a. 4 wk supply of fluoxetine b. 2 wk supply of nortriptyline c. 3 wk supply of venlafaxine d. 3 day supply of diazepam

b. 2 wk supply of nortriptyline

during asymptomatic HSV-2 infection, genital shedding of the virus occurs during approximately ____ of days a. 10% b. 25% c. 50% d. 100%

b. 25%

according to the American Cancer Society data, colorectal cancer is the number ___ cause of cancer death in men and women a. 1 b. 3 c. 5 d. 7

b. 3

generally, testing for type 2 diabetes in asymptomatic, undx individuals older than 45 yrs should be conducted every: a. year b. 3 yrs c. 5 yrs d. 10 yrs

b. 3 yrs

consideration should be given to setting A1c goal in a 22 y/o man with a 8 yr hx of type 1 diabetes who has no comorbid conditions equal to or at less than: a. 5.5% b. 6% c. 6.5% d. 7%

b. 6%

which of the following is at greatest risk of esophageal cancer? a. 34 y/o male who eats a high-fat diet b. 76 y/o male who stopped smoking 15 yrs ago c. 45 y/o woman with a hx of 6 full-term pregnancies d. 58 y/o female vegetarian

b. 76 y/o male who stopped smoking 15 yrs ago

which of the following is least likely to be found in a pt with gastric ulcer? a. hx of long-term naproxen use b. age younger than 50 yrs c. previous use of histamine2-receptor antagonist d. cigarette smoking

b. age younger than 50 yrs

high doses or prolonged use of kava kava has been associated with cases of: a. renal impairment b. hepatotoxicity c. iron-deficiency anemia d. hyperthyroidism

b. hepatotoxicity

a 66 y/o African American female has multiple risk factors for osteoporosis. which choice listed below is NOT a risk factor for osteoporosis? a. her age b. her race c. glucocorticoid intake d. excessive alcohol intake

b. her race

a female pt has the following characteristics. Which one represents a risk factor for development type 2 diabetes? a. dyslipidemia b. hx of gestational diabetes c. hypertension d. exposure to cigarette smoke

b. hx of gestational diabetes

when assessing a person with acute opioid WITHDRAWAL, you expect to find: a. constipation b. hypertension c. hypothermia d. somnolence

b. hypertension

rebound tenderness is best described as abdominal pain that worsens with: a. light palpation at the site of the discomfort b. release of deep palpation at the site of the discomfort c. palpation on the contralateral side of the abdomen d. deep palpation at the site of the discomfort

b. release of deep palpation at the site of the discomfort

a 54 y/o white man with no obvious risk for prostate cancer opted to undergo PSA screening and DRE testing. the DRE findings are normal and his PSA is 3.7 ng/mL. you recommend: a. repeating the PSA immediately b. repeat screening in 1 yr c. repeat screening in 2 yrs d. repeat screening in 5 yrs

b. repeat screening in 1 yr

testicular torsion can produce: a. penile erythema b. scrotal edema c. scrotal erythema d. penile edema

b. scrotal edema

an inguinal hernia is palpated on a male pt by an examiner. which word below best describes what the hernia feels like when touched by the examiner? a. nodular b. silky c. firm d. bumpy

b. silky

long term, recurrent high-dose oral use of mineral oil can lead to deficiency in: a. iron b. vitamin A c. vitamin C d. vitamin B12

b. vitamin A

Side effects of Meglitinides repagalinide and nateglinde

bloating, abd cramps, diarrhea, flatulence

Hyperthyroidism, Hypothyroidism or Both -change in mental status

both

Hyperthyroidism, Hypothyroidism or Both -goiter

both

which of the following statements is most consistent with IBD, IBS, or both conditions? onset of sxs is before age 30-40 yrs in most cases

both

a pound of fat contains approximately____ stored calories: a. 2500 b. 3000 c. 3500 d. 4000

c. 3500

recommended A1c goal in a 79 y/o woman with a 20 yr hx of type 2 diabetes who has difficulty ambulating, sues a walker, and has a cardiac ejection fraction of 35% and a hx of heart failure should be equal to or less than: a. 7% b. 7.5% c. 8% d. 8.5%

c. 8%

which of the following should be periodically monitored with the use of a thiazolidinedione? a. CK b. ALP c. ALT d. Cr

c. ALT remember their is a rare risk of hepatic toxicity

skip lesions are usually reported during colonoscopy in: a. IBS b. ulcerative colitis c. Crohn's disease d. C. difficile colitis

c. Crohn's disease

IBD is a term usually used to describe: a. UC and IBS b. C. difficile colitis Crohn's disease c. Crohn's disease and UC d. inflammatory colitis and ileitis

c. Crohn's disease and UC

a 63 y/o male has been your pt for several years. He is a former smoker who takes simvastatin, ramipril, and an aspirin daily. His BP and lipids are well controlled. he presents to your clinic with c/o fatigue and "just not feeling well" for the last few days. His VS and exam are normal, but his liver enzymes are elevated. His hepatitis panel is negative for infectious hepatitis. What is the most likely cause of his elevated liver enzymes? a. he has received a generic version of simvastatin b. he is an alcoholic in denial c. daily grapefruit consumption for the past 10 days d. rare liver toxicity from a usual dose of simvastatin

c. daily grapefruit consumption for the past 10 days

M. is c/o ED. he also has a condition that has reduced arterial blood flow to his penis. the most common cause of this condition is: a. epilepsy b. multiple sclerosis c. diabetes mellitus d. parkinson's disease

c. diabetes mellitus

what is the recommendation of American Cancer Society for initial screening of an African-American male male for prostate cancer? He should have: a. digital rectal exam starting at age 40 years b. PSA starting at age 45 years c. discussions starting at age 40-45 years d. he should be screened starting at age 50 years

c. discussions starting at age 40-45 years

a 78 y/o has been dx with diabetes about 10 yrs ago. an older adult with a hypoglycemic episode is more likely to exhibit: a. tremors b. sweating c. dizziness and weakness d. sxs of hyperglycemia

c. dizziness and weakness

an ulcer that is noted to be located int he region below the lower esophageal spincter and before the pylorus is usually referred to as a(n) _______ulcer a. duodenal b. esophageal c. gastric d. stomach

c. gastric

which of the following best describes the presentation of a pt with complete medial meniscus tear? a. joint effusion b. heat over the knee c. inability to kneel d. loss of smooth joint movement

c. inability to kneel

a Baker's cyst is: a. an inflammation of the bursa b. a form of tendinitis c. the buildup of synovial fluid behind the knee d. the result of a swollen ligament

c. the buildup of synovial fluid behind the knee

a 54 y/o female presents with a small to moderate amt of vaginal bleeding of recent onset. She has been postmenopausal for approximately 2 years. What dx is LEAST likely? a. endometrial carcinoma b. ovarian cancer c. endometrial hyperplasia d. uterine polyps

carcinoma b. ovarian cancer

Erosive gastritis causative agents

corticosteroids NSAIDs

a female pt is 35 y/o. She has never had an abnormal PAP smear and has had regular screening since age 18. If she has a normal PAP smear with HPV testing today, when should she have the next cervical cancer screening? a. one year b. 2-3 years c. 3 years d. 5 years

d. 5 years

the frequency for cervical screening depends on the pt and her age. What is the longest recommended time interval between cervical screens for pts who are 21-65 years of age? a. 1 yr b. 2 yrs c. 3 yrs d. 5 yrs

d. 5 yrs

hemoglobin A1c best provides information on glucose control over the past: a. 1 - 29 days b. 21 - 47 days c. 48- 63 days d. 64 to 90 days

d. 64 to 90 days

suppressive tx reduces the frequency of genital herpes recurrences by: a. 5-10% b. 20-25% c. 40-50% d. 70-80%

d. 70-80%

which of the following is an expected lab result in a pt with acute hepatitis A infection?normal values: AST, 0-31 U/L ALT, 0-40 U/L a. AST, 55 U/L; ALT, 50 U/L b. AST, 320 U/L; ALT, 190 U/L c. AST, 320 U/L; ALT 300 U/L d. AST, 640 U/L ALT, 87 U/L

d. AST, 640 U/L ALT, 87 U/L

in assessing an infant for developmental dysplasia of the hip (DDH), the NP places the infant supine, flexes the knees by holding the thumbs on the inner midthighs, with fingers outside on the hips touching the greater trochanters, stabilizes one hip, and abducts and gently pulls anteriorly on the other thigh. if this external rotation feels smooth with no sound present, there is no hip dislocation. this is: a. the Allis test b. Lasegue's sign c. the McMurray test d. Ortolani maneuver

d. Ortolani maneuver

which of the following pts has impaired glucose tolerance? a. a 70 y/o man with a fasting glucose of 109mg/dL b. an 84 y/o woman with a 1 hr post-prandial glucose of 98 mg/dL c. a 33 y/o man with a hemoglobin A1c of 5.4% d. a 58 y/o woman with a 2 hr post-prandial glucose of 152 mg/dL

d. a 58 y/o woman with a 2 hr post-prandial glucose of 152 mg/dL

How often should the healthcare provider examine the feet of a person with diabetes? a. annually b. every 6 months c. every 3 months d. every visit

d. every visit pg 875

all of the following are typically noted in a young adult with the dx of acute appendicitis EXCEPT: a. epigastric pain b. positive oburator sign c. rebound tenderness d. marked febrile response

d. marked febrile response

What are symptoms of LUTS?

decreased stream hesitancy post-void dribbling incomplete bladder emptying urinary retention painless hematuria firm smooth symmetrically enlarged prostate incontinence

Magnesium decreases does what to the BP and blood vessels Sources (examples)

decreases, vasodilates almond, peanuts, cashews, some beans, whole wheat

Medications for alcohol abuse (2)

disulfiram (Antabuse) -causes severe N&V, H/A etc when alcohol is consumed naltrexone (Vivitrol) -decreased cravings

What is a reasonable treatment option for a 30-year-old man with acute epididymitis who presents without gastrointestinal upset and will be treated as an outpatient?

doxycycline with ceftriaxone

Factors affecting PSA; increase

ejaculation cycling prostate infection massage *not DRE or exercise

The measurement of ______ is the most helpful test to confirm an abnormal TSH level

free T4 *TSH will be elevated

Cultural..... susto means

fright

What are some causes of adrenal gland damage (primary) What is the most likely/common cause?

from an autoimmune response, infection (HIV, TB, fungus), blood loss, tumor or use of anticoagulants autoimmune

medial epicondylitis

golfer's elbow- involving tight grip

During a cardiovascular assessment, the nurse knows that an S4 heart sound is:

heard at the end of ventricular diastole

UV A light exposure in psoriasis....effectiveness? risk?

highly effective increased risk of skin cancer and photo aging.

Second highest mortality of adolescence

homicide #1 MVA

Describe lesion of impetigo after they have ruptured.

honey-colored crust

Hyperthyroidism, Hypothyroidism or Both -heat intolerance

hyper

Hyperthyroidism, Hypothyroidism or Both -proximal muscle weakness

hyper

Hyperthyroidism, Hypothyroidism or Both -coarse dry hair

hypo

Hyperthyroidism, Hypothyroidism or Both -coarse dry hair, breaks easily

hypo

Hyperthyroidism, Hypothyroidism or Both -constipation

hypo

Hyperthyroidism, Hypothyroidism or Both -hyporeflexia, with a characteristic slow relaxation phase, the "hung-up" reflex

hypo

Hyperthyroidism, Hypothyroidism or Both -secondary hypertriglyceridemia

hypo

Term describing patterns of skin lesions linear= example=

in streaks (poison ivy)

PDE-5 inhibitor

include sildenafil. inhibit cGMP PDE5 and prolong vasodilatory effect of nitric oxide. Also used to treat erectile dysfuntion.

adenovirus? (males <2 yo) - healthy infant > acute colicky pain; bilious, *currant jelly* (late sign), *sausage*, *dance sign* w/ empty RLQ KUB or US --> confirm w/barium

intussusception

Most common type, location, and age at diagnosis for breast cancer

invasive ductal carcinoma age 45-65 (70% dx >50) most commonly in upper outer quadrant

___________ injuries of the ankle cause about 85% of all sprains and are the most common injury sustained while jumping or running.

inversion

inflammatory bowel disease (IBD) is a disease of unclear etiology, but likely

involves an autoimmune response to the GI tract

what does carpopedal spasm indicate?

it is one of the neuromuscular signs indicating hypocalcemia and is a significant sign of tetany pg 830

an infected host, consists of a vesicular or ulcerative lesion on the external genitalia, often not noted by the patient, which progresses to cause inguinal lymphadenitis or buboes. These can fuse and then drain, forming multiple sinus tracts with resultant scarring.

lymphogranuloma venereum

in relationship to pancreatitis...in the acute setting, alcohol is the culprit, but what is known about alcohol and pancreatitis?

most alcohol-related acute pancreatitis occurs in ppl with a min of 5-7 years of heavy drinking. Binge drinkers have a much lower risk

immature neutrophil forms that are typically found in only the granulopoiesis pool, the presence of these cells is an ominous marker of life-threatening infections and occasionally found in the presence of appendiceal rupture

myelocyts and metamyeloctes

Term describing patterns of skin lesions fissure=

narrow linear crack in the epidermis (split lip, athletes foot)

Reactive arthritis treatment

nsaids, dmards, antibiotics for infection, glucocorticoids, cytotoxiz

Lantus is dosed?

once daily

Evaluation for disease progression is a patient with sarcoidosis can involve:

pulmonary function test and carbon monoxide capacity test.

After 55, mammogram how often?

q2 unless....

_________-strengthening exercises should be performed in OA of the knee

quadriceps

Cultural..... -listening without questioning is a sign of ________, ______are held in high esteem

respect, high esteem

Management of hepatitis

rest, increase fluids, avoid alcohol, low protein, oxazepam, it K for PT>15, lactulose 30ml orally for elevated ammonia

what is the recommendation from American Cancer Society for assessment of the prostate gland in a man who is 45 y/o and of average risk for development of prostate cancer? He should have: a. screening starting at 50 yrs of age b. prostate specific antigen now c. PSA and digital rectal exam now d. digitial rectal exam only

screening starting at 50 yrs of age

bandemia is the elevation of the # of bands or young neutrophils in circulation. Typically there is less than 4% in normal circulation So when more than 4% of bands are circulating, a ___________ infection is indicated

serious

PMS/PDD

somatic and affective symptoms which occur 7-10 days prior to menstruation and end at onset of menses PMS peaks during 30s, decreases at 41

What can cause hypothalmic dysfunction?

stress, excessive exercise, severe drop in body fat (anorexia), malnutrition, severe systemic illness, stress

match the condition with the lab results: TSH=8.9 mIU/L; free T4=1.5 pmol/L hypothyroidism, hyperthyroidism, subclinical hypothyroidism

subclinical hypothyroidism

Thyroid storm=

thyrotoxicosis- acute worsening symptoms due to infection or stress. PRESENTATION: decreased LOC,fever, abd pain, LIFE-THREATENING, immediate hospitalization

Nausea, edema, breast tenderness caused by? estrogen or progestin?

too much estrogen

T/F: A systolic cardiac murmur is often benign.

true

T/F: All household members of the newborn should receive seasonal influenza vaccine and be up-to-date with pertussis immunization (Tdap in adult household members). Mother also needs to receive influenza vaccine during pregnancy. Tdap for mother with each pregnancy as previously mentioned

true

A webbed neck may be associated with?

turner syndrome

antibiotic use in ______ is discouraged because of the increased risk of C. difficileinfection.

ulcerative colitis

inflammatory condition characterized by diffuse mucosal inflammation of the colon. involves rectum and may involve whole colon. bloody diarrhea

ulcerative colitis

44 yr old male taking SSRI c/o sexual dysfunction and inability to achieve orgasm should be advised that what drug do you recommend?

venlafaxine may be a good alternative SSNRI SSRI-like effect only in low doses, with norepinephrine uptake blockade at medium to high doses, similar to TCA effect, but with fewer adverse effects. Withdrawal syndrome similar to SSRIs

-burning, swelling, throbbing, cramping, aching, &heaviness in the legs -restless legs and leg fatigue -telangietasias

venous insufficiency

What are the possible complications of Protracted PPI use?

vitamin B12 Calcium Magnesium Iron malabsorption Increased fracture C diff risk

Sickle cell anemia presentation

sudden excuriating pain due to vaso-occlusive crisis usually in the back or chest low grade fever chronically ill jaundice retinopathy delayed puberty enlarged heart fatigue systolic murmur diagnosed through hgb electrophoresis

Turning the forearm with palm up is:

supination

insulin daily dose

0.1 to 0.2 u/kg.

What are risk factors for pancreatic cancer? (3)

1] hx of chronic pancreatitis 2] tobacco use 3/ DM

Per the ADA, what is the BMI cut point for general population?

25 kg/m2

Max dose of Metformin?

2550 mg/dl

How far before and after ovulation can conception occur?

6 days before, 1 day after

Lyme disease is caused by the bacterium: A. Borrelia burgdorferi. B. Bacillus anthracis. C. Corynebacterium striatum. D. Treponema pallidum

A. Borrelia burgdorferi

ADA suggest A1C goals based on Age for T2DM: for T1DM pts_________

A1C<6%

ADA suggest A1C goals based on Age for T2DM: for most pregnant patients _________

A1C<6%

ADA suggest A1C goals based on Age for T2DM: for most adults with T2DM_________

A1C<7%

First-line treatment of impetigo with less than 5 lesions of 1-2 centimeters in diameter on the legs in a 9-year-old girl is: A. topical mupirocin. B. topical neomycin. C. oral cefixime. D. oral doxycycline

A. topical mupirocin.

A possible adverse effect with the use of a first-generation antihistamine such as diphenhydramine in an 80-year-old man is: A. urinary retention. B. hypertension. C. tachycardia. D. urticaria

A. urinary retention.

(+) HBsAg, HBeAg, anti-HBc, IgM

ACTIVE Hep B

The administration of a short acting insulin with each meal that requires frequent BG checks is what type of dosing in T2DM?

Basal bolus

A 2-year-old boy has been diagnosed with "physiologic cryptorchidism." Given this diagnosis, during assessment the nurse will most likely observe:

ANS: an absence of the testis in the scrotum, but the testis can be milked down. Migratory testes (physiologic cryptorchidism) are common because of the strength of the cremasteric reflex and the small mass of the prepubertal testes. The affected side has a normally developed scrotum and the testis can be milked down. The other responses are not correct.

When performing a genitourinary assessment, the nurse notices that the urethral meatus is positioned ventrally. This finding is:

ANS: called hypospadias. Normally the urethral meatus is positioned just about centrally. Hypospadias is the ventral location of the urethral meatus. The position of the meatus does not change with aging. Phimosis is the inability to retract the foreskin. A stricture is a narrow opening of the meatus.

The nurse is performing a genital examination on a male patient and notices urethral drainage. When collecting urethral discharge for microscopic examination and culture, the nurse should:

ANS: compress the glans between the examiner's thumb and forefinger and collect any discharge. If urethral discharge is noticed, then the examiner should collect a smear for microscopic examination and culture by compressing the glans anteroposteriorly between the thumb and forefinger. The other options are not correct actions.

When performing a genital examination on a 25-year-old man, the nurse notices deeply pigmented, wrinkled scrotal skin with large sebaceous follicles. On the basis of this information the nurse would:

ANS: consider this a normal finding and proceed with the examination. After adolescence, the scrotal skin is deeply pigmented and has large sebaceous follicles. The scrotal skin looks corrugated

Suspicion of appendicitis, order

Abd CT

A laboratory finding commonly observed in patients with sarcoidosis is: A. hyponatremia. B. hypercalcemia. C. hypokalemia. D. hyperkalemia.

B. hypercalcemia.

Cilostazol (Pletal) should be used with great caution in the presence of which of the following diagnoses? A. diabetes mellitus B. heart failure C. hypertension D. dyslipidemia

B. heart failure

Which of the following is not a recommended option to make cosmetic improvements for phymatous rosacea? A. laser peel B. ablative laser surgery C. surgical shave technique D. mechanical dermabrasion B. ablative laser surgery

B. ablative laser surgery

Finasteride (Proscar, Propecia) and dutasteride (Avodart) are helpful in the treatment of BPH because of their effect on: A. bladder contractility. B. prostate size. C. activity at select bladder receptor sites. D. bladder pressure.

B. prostate size.

The bisphosphonate therapy given as an annual infusion is: A. risedronate. B. zoledronic acid. C. ibandronate. D. denosumab. SE of bisphosphonate therapy >5yr=

B. zoledronic acid. atypical fractures

administration of a short acting insulin that is given before biggest meal is what type of insulin administration for T2DM.

Basal plus

S/S of excess progesterone?

Breaset tenderness HTN Depression Fatigue Decreased libido Decreased duration of bleeding Increased appetite

Treatment options for uncomplicated Chlamydia trachomatis infection in pregnancy include A. Clarithromycin. B. Doxycycline. C. Azithromycin. D. Ofloxacin

C. Azithromycin.

The most likely causative pathogens in a 26-year-old man with acute epididymitis include: A. Escherichia coli. B. Enterobacteriaceae. The most likely causative pathogens in a 26-year-old man with acute epididymitis include: C. C. trachomatis. D. Pseudomonas species.

C. C. trachomatis. .

A common cause of angular cheilitis is infection by: A. Escherichia coli B. Streptococcus pneumoniae C. Candida species D. Aspergillus species

C. Candida species

Which of the following would be ineffective in treating acute gouty arthritis? A. Intraarticular corticosteroid injection B. Naproxen Sodium (Aleve) C. Febuxostat (Uloric) D. Colchicine (Colcrys)

C. Febuxostat (Uloric)

Which of the following would not be recommended to prevent Lyme disease when visiting a Lyme-endemic area?A. Wear long pants and long-sleeved shirts. B. Use insect repellent. C. If a tick bite occurs, wait until after consulting a healthcare provider before removing the insect. D. If a tick bite occurs and the tick is engorged, administer a single 200-mg dose of doxycycline

C. If a tick bite occurs, wait until after consulting a healthcare provider before removing the insect.

Type I hypersensitivity reaction is mediated through: A. TNF-binding to T cells. B. IgG antibodies binding to T cells. C. IgE antibodies binding to mast cells. D. IL-10 binding to basophils.

C. IgE antibodies binding to mast cells

The most common sites for lumbar disk herniation are: A. L1 to L2 and L2 to L3. B. L2 to L3 and L4 to L5. C. L4 to L5 and L5 to S1. D. L5 to S1 and S1 to S2.

C. L4 to L5 and L5 to S1.

Loss of Achilles tendon reflex most likely indicates a lesion at: A. L1 to L2. B. L3 to L4. C. L5 to S1. D. S2 to S3.

C. L5 to S1.

Which of the following is the most common presentation in a patient with Raynaud phenomenon? A. digital ulceration B. worsening of symptoms in warm weather C. a period of intense itchiness after blanching D. unilateral symptoms

C. a period of intense itchiness after blanching

A 29-year-old woman has a sudden onset of right-sided facial asymmetry. She is unable to close her right eyelid tightly or frown or smile on the affected side. Her examination is otherwise unremarkable.This likely represents paralysis of cranial nerve: A. III B. IV C. VII D. VIII

C. VII

Psoriasis vulgaris is a chronic skin disease caused by: A. bacterial colonization. B. absence of melanin. C. accelerated mitosis. D. type I hypersensitivity reaction

C. accelerated mitosis

Tamsulosin (Flomax) is helpful in the treatment of BPH because of its effect on: A. bladder contractility. B. prostate size. C. activity at select bladder receptor sites. D. bladder pressure.

C. activity at select bladder receptor sites

Seborrheic dermatitis is likely caused by: A. accelerated mitosis of skin cells. B. colonization of skin by Staphylococcus aureus. C. an inflammatory reaction to Malassezia species on skin. D. exposure to excessive UV radiation.

C. an inflammatory reaction to Malassezia species on skin

The most common causative organisms in cellulitis are: A. Escherichia coli and Haemophilus influenzae. B. Bacteroides species and other anaerobes. C. group A beta-hemolytic streptococci and S. aureus. D. pathogenic viruses

C. group A beta-hemolytic streptococci and S. aureus.

You see a 28-year-old man who was involved in a fight approximately 1 hour ago with another person. The patient states, "He bit me in the arm." Examination of the left forearm reveals an open wound consistent with this history. Your next best action is to: A. obtain a culture and sensitivity of the wound site B. refer for rabies prophylaxis C. irrigate the wound and débride as needed D. close the wound with adhesive strips

C. irrigate the wound and débride as needed

When advising a woman with varicose veins about the use of support stockings, you consider that the preferred type: A. can be purchased in the hosiery section of a department store. B. is a lightweight pair and available over-the-counter. C. is a medium weight to heavy-weight prescription product D. is used in the form of panty hose.

C. is a medium weight to heavy-weight prescription product

Symptoms in chronic bacterial prostatitis often include: A. fever. B. gastrointestinal upset. C. low back pain. D. penile discharge.

C. low back pain.

Common triggers for anaphylaxis include exposure to certain types of all of the following except: A. medications B. food C. pet dander D. insect bites

C. pet dander

Clinical findings of the knee in a patient with OA include all of the following except: A. coarse crepitus. B. joint effusion. C. warm joint. D. knee often locks or a pop is heard.

C. warm joint.

Biological agents to treat psoriasis, such as infliximab and etanercept, work by blocking the action of: A.IL-9 B.CD4 C.TNF-a D.IgG

C.TNF-a

Gram-negative bacteria that commonly cause burn wound infections include all of the following except: A. P. aeruginosa B. E. coli C. K. pneumoniae D. H. influenzae

D. H. influenzae

154. When counseling a patient taking a bisphosphonate such as alendronate (Fosamax), you advise that the medication should be taken with: A. a bedtime snack. B. a meal. C. other medications. D. a large glass of water.

D. a large glass of water.

Who is most likely to have new-onset primary Raynaud phenomenon? A. a 68 year old man B. a 65 year old woman C. A 25 year old man D. an 18 year old woman

D. an 18 year old woman

Which part of the prostate is readily palpable during a DRE? A. anterior lobe B. median lobe C. lateral lobes D. posterior lobe

D. posterior lobe

Patients with lateral epicondylitis typically present with:

Decreased hand grip strength

Igm anti-HBc Igm anti-HBc(-) means

Did not acquire recently

-Failure rate 10% -Must insert 6 hour prior and remain 6 hours after -May provide protection from G&C -Increased risk for candidiasis (never leave in >30 hours)

Disposable Barriers (Sponge)

-A1C >6.5 -Fasting plasma glucose >126 -S/s hyperglycemia - polyuria, polydipsia, polyphagia + random blood glucose >200 -2 hr plasma glucose >200 during GTT w/ 75g of glucose

Dx DM

What is gold standard diagnostic test with cholecystitis?

First Ultrasound Then HIDA scan

How do you know local resistance rates?

Get a copy of local antibiogram which local microbiology lab will have. Or app "Bugs and Drugs"

What should you do before starting treatment for pyelonephritis?

Get urine and blood cultures.

How does heat help in an acute injury?

Helps to reabsorb bruising

What is the strongest risk factor for acquiring Hepatitis C? The major risk factor for Hepatitis D is ___________

IV drug use

Hepatitis B - What does it mean if: (-) HBsAg (+) anti-HBs (+) anti-HBc IgG

Immunity due to infection

Treatment for H. Pylori (-) ulcer: -Quadruple therapy is indicated and include:

Lifestyle changes w/ 4-8 weeks PPIs: omeprazole (Prilosec) 20mg daily esomeprazole (Nexium) 40mg daily

Diabetic Rx that are weight neutral

Meglitanides (Starlix), DDP-4

Associated with pregnancy (past/present) or use of oral contraceptives that cause brown to tan colored stains on the upper cheeks and forehead in some women. More commonly darker-skinned women. Stains are usually permanent but can lighten over time. Cause=estrogen

Melasma (Mask of pregnancy)

Cultural..... [Curandera] is...

Mexican healer

Heberden's nodes: distal interphalangeal joints Bouchards nodes: proximal interphalangeal joints genetic, obese, crepitus, LIMITED ROM, increased age

OA

narrowing of the joint space , bone spurs are present

OA

caused by H. pylori, NSAIDS, glucocorticoids, more common in men, duodenal age 30-55, gastric 55-65, smokers, not caused by alcohol

Peptic Ulcer Disease

Specific Headache Clues: Sweating, tachycardia

Pheochromocytoma

Foreskin cannot be pushed back from the glans penis because of edema; usually seen in neonates.

Phimosis

Acute or recurrent pain on the bottom of the feet that is aggravated by walking. Caused by microtears in the plantar fascia due to tightness of the Achilles tendon. Higher risk with obesity (body mass index [BMI] greater than 30), diabetics, aerobic exercise, flat feet, prolonged standing.

Plantar Fasciitis

*PIH + proteinuria + edema* May occur from 20wks gestation to 4 wks postpartum Sx: sudden wt. gain, peripheral edema (can't put on shoes or take rings off), HA, visual disturbances, oliguria, N/V may indicate encephalopathy Dx: check urine at every visit, BPs, labs, US, nonstress test, kick counts Tx: referral, bedrest, weekly steroid injections if <34 weks for lung development

Preeclampsia

Specific Headache Clues: Transient visual changes, intracranial noise

Pseudotumor cerebrii (idiopathic intracranial hypertension)

Koebner phenomenon is r/t to what inherited skin disorder

Psoriasis -new psoriatic plaques forms over areas of skin trauma

Auspitz sign is r/t to what inherited skin disorder

Psoriasis -pinpoint areas of bleeding remain in the skin when a plaque lesion is removed

Need Xray, often require surgery to pin fixate joint no ambulation to prevent irreparable joint damage more common in african american males

Slipped Capital Femoral Epiphysis

Which 2 drugs are used to treat Bells Palsy?

Steroids Antiviral

What joints are effected in pseudo gout

The knees (most commonly) wrists and ankles also can be involved, with symptoms of swollen, warm, and severely painful joints

Labs of hepatitis

UA: proteinuria, biliurinia, elevated AST, ALT (500-2000)

Diagnostics for acute bacterial prostatitis?

Urine culture

What antibiotic use to treat purulent cellulitis? 1. clindamycin 2. TMPS (Bactrim) 3. doxycycline 4. ceftriaxone (Rocephin)

When there is pus about, let it out! I&D it 1st then start ABX. 1. clindamycin 2. TMPS (Bactrim) 3. doxycycline

which of the following is the least helpful test for the assessment of thyroid disease? a. total T4 b. TSH c. free T4 d. antihyroid antibodies

a. total T4

how often should a diabetic have his/her eyes checked?

annually

Adam's Forward Bend Test is used to evaluate what?

asymmetry of shoulder, ribs, hips and waistline related to scoliosis

which of the following best describes ethnicity and insulin sensitivity? a. little variation exists in insulin sensitivity among different ethnic groups b. African Americans are typically less sensitive to the effects of insulin when compared to people of European descent c. Mexican Americans are likely the most insulin sensitive ethnic group residing in North America d. the degree of insulin sensitivity has little influence on insulin production

b. African Americans are typically less sensitive to the effects of insulin when compared to people of European descent

deformity of the proximal interphalangeal joints found in an elderly pt with OA is knwn as: a. Heberden nodes b. Bouchard nodes c. hallus valgus d. Dupuytren contracture

b. Bouchard nodes

transillumination of fluid in the scrotum may be seen with: a. a varicocele b. a hydrocele c. testicular torsion d. testicular cancer

b. a hydrocele

with the use of ezetimibe (zetia), the NP expects to see: a. a marked increase in HDL cholesterol b. a reduction in LDL cholesterol c. a significant reduction in triglyceride levels d. increased rhabdomyolysis when the drug is used in conjunction with HMG-CoA reductase inhibitor

b. a reduction in LDL cholesterol

a pt has been dx today with type 2 diabetes. A criterion for dx is: a. an abnormal random blood glucose b. proteinuria c. a fasting glucose => 126 and confirmed on a previous day d. an abnormal post-prandial glucose

c. a fasting glucose => 126 and confirmed on a previous day

a NP has decided to initiate insulin in a pt who takes oral diabetic medication. How much long acting insulin should be initiated in a pt who weight 100 kg? a. about 5 units b. 10 units c. 15 units d. 20 units

d. 20 units formula = 0.2 units x kg

risk factors for Barrett esophagus include all of the following EXCEPT: a hx of cigarette smoking b. older than 50 yrs of age c. male gender d. African American ethnicity

d. African American ethnicity

the bladder tumor antigen test may also be positive with: a. testicular torsion b. the use of steroids for bodybuilding c. scrotal trauma d. symptomatic sexually transmitted disease

d. symptomatic sexually transmitted disease

Cultural..... [mal ojo] means

evil eye

BMD between -1 and -2.5

osteopenia

BMD-2.5 is indicative of

osteoporosis

Vitamin D deficiency may ultimately lead to

ricketts

Safety Assessments: Dementia

• Driving • Financial capacity • Wandering • Living alone

Essential Tremor (ET)

• Most common tremor (5% population affected) • Sometimes familial (termed familial tremor) • Sporadic Essential tremor (termed benign essential tremor) • More common with aging • Bilateral action tremor of hands, forearms, head, voice, chin, lip tremor • Tremor in legs is unusual

A 45-year-old man is in the clinic for a routine physical. During the history the patient states he's been having difficulty sleeping. "I'll be sleeping great and then I wake up and feel like I can't get my great." The nurse's best response to this would be:

"Do you have any history of problems with your heart?"

Celiac disease symptoms

- most common "silent" sx: iron deficient anemia, osteoporosis - GI: N/V/D, IBS, flatulence, ab pain - nutrition: fatigue, irritability, alopecia, failure to thrive - skeletal: osteoporosis, arthritis, bone pain, tooth enamel

Hypothyroidism labs: Causes: (3) Most common cause: (1)

-High TSH with low free T4 (NOT TOTAL) -Causes: Hashimoto's, postpartum, thyroid ablation -Hasimotos is most common cause in US

***LEIK*** Malignant Neuroleptic Syndrome Rare & life-threatening IDIOPATHIC reaction to Typical/Atypical antipsychotics (usually after dose increase) Affect dopaminergic system SUDDEN high fever LOC change urinary incontinence BP fluctuation muscular rigidity

...

**LEIK** [Exercise and Injuries] For the 1st 48 hours.... NO exercising (all forms) NO HEAT(shower,pack) NO ACTIVE ROM [R]est - Don't use injured limb/joint [I]ce -1st 48h, c.packs (20on/10off) [C]ompression -use ace wrap, decrease swelling & provide support [E]levation - prevents/decrease swelling avoid wt bearing

...

Stage _________ Lyme disease findings include mild flu-like symptoms single annular lesion rarely pruritic S/S usually resolve in 3-4 weeks

1

What is the usual regimen for PID treatment in terms of days of treatment?

1. Oflaxacin + levofloxacin with/without Metronidazole 2. Cefoxitin + probenicid + doxycycline with or w/oMetro 3. Ceftriaxone + doxycycline + with or w/o Metronidazole 14 days

Who is likely to get Ulcerative Colitis?

15-40 yo 50-70 yo Male=female Jewish

The Pap test is recommended for women between ________ years of age.

21 and 65

Regular insulin (Humulin R, Novolin R) *short acting* What is the onset of action is?

30min-1 hr

What GFR should metformin be discontinued?

< 45 renally excreted by (90%)

Placenta Previa

A multipara who is in the late second to third trimester complains of new onset of painless vaginal bleeding that is worsened by intercourse. Blood is bright red in color. Uterus is soft and nontender. If cervix is not dilated, treatment is strict bed rest. Intravenous magnesium sulfate if there is uterine cramping. Uterus will usually reimplant itself if mild. Any vaginal or rectal insertion or stimulation is an absolute contraindication (can precipitate hemorrhage). If cervix is dilated or if hemorrhaging, fetus is delivered by C-section. Severe cases cause hemorrhage; fetus must be delivered to save mother's life.

When taken concomitantly with warfarin, which of the following causes a possibly decreased anticoagulant effect? A. cholestyramine B. allopurinol C. cefpodoxime D. zolpidem

A. cholestyramine

Which of the following is a treatment option for a 30-year-old woman with PID and a history of severe hive-form reaction when taking a penicillin or cephalosporin? A. ofloxacin with metronidazole B. amoxicillin with gentamicin C. cefixime with vancomycin D. clindamycin with azithromycin

A. ofloxacin with metronidazole

Signs that bed bugs are present in a home include all of the following except: A. small drops of fresh blood on floorboards. B. blood smears on bed sheets. C. presence of light brown exoskeletons. D. dark specks found along mattress seams.

A. small drops of fresh blood on floorboards.

Which of these statements is most appropriate when the nurse is obtaining a genitourinary history from an elderly man?

ANS: "Do you need to get up at night to urinate?" The elderly male patient should be asked about the presence of nocturia. This may be due to diuretic medication, fluid retention from mild heart failure or varicose veins, or fluid ingestion 3 hours before bedtime, especially coffee and alcohol. The other questions are more appropriate for younger males.

The nurse is describing how to perform a testicular self-examination to a patient. Which of these statements is most appropriate?

ANS: "If you notice an enlarged testicle or a painless lump, call your health care provider." If the patient notices a firm painless lump, a hard area, or an overall enlarged testicle, he should call his health care provider for further evaluation. The testicle normally feels rubbery with a smooth surface. A good time to examine the testicles is during the shower or bath, when one's hands are warm and soapy, and the scrotum is warm. It should be performed once a month.

Hepatitis B - What does it mean if: (+) HBsAg (-) anti-HBs (+) anti-HBc IgM (-) anti-HBc IgG

Acute Hepatitis B

Infection ascends into urinary tract. Most common nonsexually transmitted cause is Enterobacter. If condition occurs in a male under 35 years old, it is treated like gonococcal or chlamydial urethritis

Acute Prostatitis Objective ■ Gently examine prostate. Prostate will be extremely tender and warm. ■ Warning: Vigorous palpation and massage of an infected prostate can cause septicemia. Labs ■ CBC: Leukocytosis with shift to the left (presence of band cells). ■ UA: Large amount of white blood cells (pyuria), hematuria. ■ Urine C&S (if possible, also obtain urine after gentle prostatic massage).

Amniocentesis and Chorionic Villus Sampling (CVS) Chorionic villus sampling can be done earlier ( weeks) than the amniocentesis (weeks). Specimens contain fetal cells. Fetal chromosomes/DNA are tested for ( )

Amniocentesis and Chorionic Villus Sampling (CVS) Chorionic villus sampling can be done earlier (10-12 weeks) than the amniocentesis (15-18 weeks). Specimens contain fetal cells. Fetal chromosomes/DNA are tested for abnormalities.

Pt presents with lanugo, amenorrhea for >3 months, marked wt loss (>15%) and peripheral edema....what do you suspect?

Anorexia nervosa additionally: -usually in adolescence -irrational preoccupation with an intense fear of gaining wt. -secretive, perfectionistic and self-absorbed. -At HIGHER RISK for osteopenia/osteoporosis

What syndrome can an IUD prevent?

Ashermans (adhesions)

HowHow many grams of a topical cream or ointment are needed for a single application to the hands? A. 1 B. 2 C. 3 D. 4

B. 2

What is the international normalized ratio (INR) range recommended during warfarin therapy as part of the management of a patient with a DVT? A. 1.5 to 2.0 B. 2.0 to 3.0 C. 2.5 to 3.5 D. 3.0 to 4.0

B. 2.0 to 3.0

When prescribing fluconazole, the NP considers that it is a cytochrome P-450: A. 3A4 inhibitor B. 2CP inhibitor C. 2D6 inducer D. 1A2 inducer

B. 2CP inhibitor interacts with carbamazepine, some benzos, and CCBS

A positive Homans sign is presented in approximately what percentage of patients with DVT? A. 25% B. 33% C. 50% D. 75%

B. 33%

A 48-year-old woman complains of increased frequency and severity of hot flashes. Her last menses occurred 6 months ago. You would expect all of the following laboratory findings except: A. increased levels of LH. B. elevated levels of testosterone. C. reduced levels of estradiol. D. reduced levels of progesterone.

B. elevated levels of testosterone.

Risk factors for medial epicondylitis include playing: A. tennis. B. golf. C. baseball. D. volleyball.

B. golf

Which of the following best describes seborrheic dermatitis lesions? A. flaking lesions in the antecubital and popliteal spaces B. greasy, scaling lesions in the nasolabial folds C. intensely itchy lesions in the groin folds D. silvery lesions on the elbows and knees

B. greasy, scaling lesions in the nasolabial folds found in area w/ high concentration of sebaceous glands: scalp, eyelid margins, nasolabial folds, ears and upper trunk

The likely causative organisms of nonbullous impetigo in a 6-year-old child include: A. H. influenzae and S. pneumoniae. B. group A streptococcus and S. aureus. C. M. catarrhalis and select viruses. D. P. aeruginosa and select fungi

B. group A streptococcus and S. aureus.

A 54-year-old white man with no obvious risk for prostate cancer opted to undergo PSA screening and DRE testing. The DRE findings are normal and his PSA is 3.7 ng/mL. You recommend: A. repeating the PSA test immediately. B. repeat screening in 1 year. C. repeat screening in 2 years. D. repeat screening in 5 years.

B. repeat screening in 1 year.

Risk factors for mitral regurgitation include a prior diagnosis of all of the following except: A. rheumatic heart disease. B. scarlet fever. C. endocarditis. D. calcific annulus.

B. scarlet fever.

A 56-year-old truck driver presents with a new nodular, opaque lesion with non distinct borders on his left forearm. This most likely represents a(n): A. actinic keratosis B. squamous cell carcinoma C. basal cell carcinoma D. malignant melanoma

B. squamous cell carcinoma "NO SUN" N=odular O=paque S=un-exposed areas U=clerating N=ondistinct boarders

The primary treatment option for sarcoidosis is the use of: A. parenteral methotrexate. B. systemic corticosteroids. C. oral acetaminophen D. oral hydroxychloroquine.

B. systemic corticosteroids.

type of bursitis that is located behind the knee (popliteal fossa). Sometimes when a joint is damaged and/or infl amed, synovial fluid production increases, causing the bursa to enlarge. Bursae are the protective synovial sacs that are located on certain on joints.

Baker's Cyst

Explain ACTH stimulation test.

Baseline cortisol level Administration of synthetic ACTH Repeat cortisol level Damage to the adrenal gland will show limited or no response to the synthetic ACTH.

What DM medication? -reduces hepatic glucose production and intestinal absorption via increased peripheral glucose uptake and utilization. -insulin sensitizer Major Complaint= GI upset (diarrhea) A1C reduction = 1-2% with intensified use *requires monitoring of Cr lvl **CONTRAINDICATIONS** -IMPAIRED RENAL FX -HEPATICFAILURE -HEART FAILURE -ALCOHOLICS -HYPOXIA -PPL >80yo

Biguanide metformin (Glucophage)

Decrease gluconeogeneis is an action of which oral anti diabetic drug

Biguanindes glucophage (Metformin)

What criteria should be met to determine that a woman does not need additional contraceptives while breastfeeding?

Breastfeeding with <15% supplemented <6mo postpartum amennorhea

Management of PMS

Bromocriptine, xanax, buspirone, SSRIs, progesterone restrict caffeine

Presentation of superficial venous thrombophlebitis usually includes: A. positive Homans sign B. diminished dorsalis pedis pulse C. a dilated vessel D. dependent pallor

C. a dilated vessel

You write a prescription for a topical agent and anticipate the greatest rate of absorption when it is applied to the: A. palms of the hands B. soles of the feet C. face D. abdomen

C. face

Which of the following is not representative of the presentation of primary syphilis? A. painless ulcer B. localized lymphadenopathy C. flu-like symptoms D. spontaneously healing lesion

C. flu-like symptoms

Which of the following is likely to be noted with short-term (less than 1 to 2 years) HT use in a postmenopausal woman? A. reduction in dementia risk B. significant increase in breast cancer risk C. minimized hot flashes. D. increase in cardiovascular risk

C. minimized hot flashes.

94. A 22-year-old woman complains of pelvic pain. Physical examination reveals cervical motion tenderness and uterine tenderness. Which of the following would further support a diagnosis of PID? A. temperature less than 100ºF (37.8ºC) B. absence of white blood cells in vaginal fluid C. mucopurulent vaginal discharge D. laboratory documentation of cervical infection w/E. coli

C. mucopurulent vaginal discharge

To prevent a recurrence of testicular torsion, which of the following is recommended? A. use of a scrotal support B. avoidance of testicular trauma C. orchiopexy D. limiting the number of sexual partners

C. orchiopexy A procedure in which both testes are brought down and tacked lower in the scrotum, is usually performed to avoid subsequent torsion.

(Toxic Shock Syndrome) TSS

Cause by Staph aureaus RF: tampon use, skin/resp infections, post-surgery, postpartum Sx: 2-3 prodrome of malaise, myalgia, fever >102, sunburn-like rash on palms and soles, vomiting, watery diarrhea, abd pain followed by organ failure and shock Tx: hospitalize immediately!

Management with epididymitis for <35 y.o.

Ceftriaxone and doxyclycline OR Azithromycin

Failure rate 18% -May provide some protection against STIs when use with spermicidal gel -Have refitted if >20lb wt change -Do not use with oil based lube -MUST REMAIN WITHIN VAGINA FOR 6 HOURS POST-SEX -DO NOT REMOVE FOR REPEAT SEXUAL ACTIVITY, SIMPLY REAPPLY SPERMICIDE

Cervical Cap (Diaphram)

H/A type? -Always unilateral -Pain peaks within minutes; excruciating, explosive -Varies -30-90 minutes usually (up to 180 minutes) -can resolve spontaneously - EtOH can trigger ***ICE-PICK**** *begins around temple or eye* *rhinitis/orrhea, tearing,ptosis*

Cluster Headache Treatment -HD O2 (100% 12L by mask) 15m ***not in COPD** -sumatriptan (Imitrex) inje/nasal Prophylaxis: -verapamil PO daily

________ Fracture Fracture of the distal radius (with or without ulnar fracture) of the forearm along with dorsal displacement of wrist. History of falling forward with outstretched hand "dinner fork" fracture due to the appearance of arm and wrist after the fracture. The most common type of wrist fracture. ***pain on side of pinky***

Colles Fracture

inflammation of the cartilage that connects a rib to the sternum from injury, blow to chest, physical strain, URI, infection, fibromyalgia pain and tenderness where the ribs attach to the breastbone, pain when taking deep breath

Costochondritis

Crohn's or UC vitamin B12 deficieny macrocytic normochromic parenteral replacement renal calculi lactose intolerance Gut rest tobacco use plays factor small bowel cancer Use of cipro and flag

Crohn's

Patients with __________________ disease involving the small intestine frequently have abdominal pain, involuntary weight loss, diarrhea, and occasionally they have symptoms of intestinal obstruction. "Skip lesions," areas of affected mucosal tissue alternating with normal tissue, are common; the rectum is often spared with the terminal ileum and right colon involved

Crohn's also, A cobblestone mucosal pattern is often identified on endoscopy or contrast radiography

In ___________ syndrome there is compression that can lead to rectal or perineal pain and disturbance in bowel and bladder function. Signs of lumbosacral strain are present, and the straight-leg-raising maneuver reproduces pain.

Cuada equina

Leonard is an 18-year-old man who has been taking isotretinoin (Accutane) for the treatment of acne for the past 2 months. Which of the following is the most important question for the clinician to ask at his follow-up office visit? A. Are you having any problems remembering to take your medication? B. Have you noticed any dry skin around your mouth since you started using Accutane? C. Do you notice any improvement in your skin? D. Have you noticed any recent changes in your mood?

D. Have you noticed any recent changes in your mood?

A mother brings to the clinic her 3-year-old daughter,who presents with dry red patches on her face around the eyes. The mother has observed her daughter constantly rubbing the area, which has caused swelling around the eyes. Physical examination is consistent with atopic dermatitis. The NP considers that this is adiagnosis that: A. requires a skin culture to confirm contributing bacterial organisms. B. should be supported by a biopsy of the affected area. C. necessitates obtaining peripheral blood eosinophil level D. Is usually made by clinical assessment alone

D. Is usually made by clinical assessment alone

A common site for atopic dermatitis in an adult is on the: A. dorsum of the hand. B. face. C. neck. D. flexor surfaces

D. flexor surfaces

In a 22-year-old woman using isotretinoin (Accutane) therapy, the NP ensures follow-up to monitor for all ofthe following tests except: A. hepatic enzymes B. triglyceride measurements C. pregnancy test D. platelet count

D. platelet count

Shingles most commonly involve the dermatomes of the: A. legs and pubic area. B. face. C. upper arms and shoulders. D. thorax

D. thorax

-Normal physiologic event -Elevation in fasting blood glucose early in the am -Due to increase in insulin resistance b/w 4-8 am -R/t spike in growth hormone (a. pituitary), glucagon (pancreatic b cells), epi & cortisol (adrenals)

Dawn Phenomenon

Finkelstein's Test

De Quervain's tenosynovitis w/ Finkelstein's Test A young mother carrying baby with flexed wrist and extended thumb to stabilize the baby's head. She is having pain on the thumb side of her wrist. What do you suspect and how to you test?

How does the prostate palpate with acute bacterial prostatitis?

Edematous/tender/boggy prostate

Treatment for Epididymoorchitis Age > 35 or insertive partner in anal intercourse

Enterobacteriaceae (coliforms) Ciprofloxacin 500mg QD or levofloxacin 750 mg QD x 10-14 days

S/S with acute bacterial prostatitis?

Fever/chills, low back pain, dysuria, urgency, frequency, nocturia

Exnatide (Byetta) BID or Victoza subq Daily Stimulates glucagon like peptide that increases insulin production and inhibits postprandial glucagon release - decreases postprandial hyperglycemia Causes weight loss May cause pancreatitis / pancreatic duct metaplasia - monitor amylase and lipase

GLP-1

DIAGNOSIS? -TSH <0.01 w/ elevated T3 & T4 - +thyroid stimulating immunoglobulin -Thyroid ultrasound if mass / nodule -24 hr RAIU shows homogenous uptake

GRAVE'S DISEASE

Post exposure Hep A

Healthy 12 mo- 40 yo vaccine at age appropriate dose preferred to IG bc offers long term protection, those > 40 yo should receive IG (effective if given within 2 weeks of exposure) Self-limiting so just treating s/s, no medications to tx, will clear on its own & liver should heal itself after a month or two, advise patient to rest, eat small high calorie snacks to combat nausea & weight loss, any medications processed by liver (such as tylenol) should be stopped or changed, alcohol should be avoided while s/s persist

mild right upper quadrant abdominal tenderness w/o rebound is found in 50% of patients, with splenomegaly in 15%

Hepatomegaly

Specific lumbar nerve root findings... Pain and numbness radiates to medial aspect of knee into medial malleolus quadriceps weakness, knee pain, how do you assess?

L3-L4: Pain and numbness radiates to medial aspect of knee into medial malleolus quadriceps weakness, knee pain, **(have pt squat and rise )**

Patient complains of pain over medial epicondyle or inner aspect of lower humerus. Pain worsens with wrist flexion and pronation activities. Local epicondylar tenderness, elbow pain, forearm weakness, pain aggravated by wrist flexion, and pronation activities with decreased grip strength and full ROM occur.

Medial epicondylitis

Bleeding between menstrual periods

Metrorrhagia

Specific Headache Clues: Unilateral vision loss

Optic neuritis

<1-2% failure rate -Estrogen + Progesterone -Apply once per week -Easily reversible -Reduced effectiveness if >90kg -Increased risk for MI, CVA, PE -Contraindicated in >35, smokers, uncontrolled HTN

OrthoEvra (patch)

good for up to age 1 diagnostic test involves abducting the thighs to test for hip subluxation or dislocation, if the femoral head can be felt to slip forward into the acetabulum on pressure from behind it is dislocated and positive evaluates hip dyplasia

Ortolani's Test

rupture of growth plate at the tibial tuberosity from stress on patellar tendon painful limp below the knee exacerbated by running, jumping Swelling

Osgood Schlatter RICE, NSAID

GERD there is an increased risk of fracture of the hip, spine, wrist and forearm with the long term use of ___________

PPI

Without sufficient amounts of vitamin D, intestinal calcium absorption is inadequate. The resulting calcium deficiency prompts an increase in production and secretion of _______ hormone.

Parathyroid

ecchymosis and swelling in the lower abdomen, the hips, groin, and/or scrotum. May have bladder and/or fecal incontinence, vaginal or rectal bleeding, hematuria, numbness, etc. May cause internal hemorrhage, which can be life threatening.

Pelvic fracture

What does the IgG antibody due during Hepatitis A virus?

Persists for > 10 years and provides immunity

new onset low back pain, ED/rectal/perineal pain accompanied by obstructive voiding symptoms such as weaker stream and nocturne.

Prostate cancer

Dx of arthritis which manifests within 2 month of a bacterial gastroenteritis or nongonococcal urethritis/cervicitis (most commonly C. trachomatis) with a unilateral presentation.

Reactive arthritis (Reiter's syndrome) -"unilateral finding" _^ESR and CRP -urethritis -unilateral conjunctivitis -mucocutaneous lesions (mouth ulcers) enthesitis (Achilles tendon pain) *get urine PCR* **can be linked genetically (HLA-B27 gene)

Which arthritic condition is typically seen many days to weeks after an episode of acute bacterial diarrhea caused by Shigella species, Salmonella species, Campylobacter species, or a sexually transmitted infection such as Chlamydia trachomatis or Ureaplasma urealyticum.

Reactive arthritis aka Reiter syndrome

What is cervical mucus/billings test?

Record changes in cervical mucus over 3-4 months, increase in spinnbarkeit (egg white like) during ovulation, Abstain from sex at mucus change til 4 days after until is resumes thickness again.

What is the calendar method of family planning?

Record serial cycles, determine fertile phase by subtracting 18 days from shortest cycle, 11 days from longest cycle

What do these labs indicate? Anti-HAV, IgG

Recovered Hep A

What happens if ortho evra is left off for >24 hours? Where can the patch be applied? Where should it NOT be placed?

Restart whole pack and use back up arm, buttocks, torso, abdomen *Not breast!

What medications must be monitored while taking with OCPs

St. Johns Wort, antibiotics, anticonvulsants, warfarin, insulin, hypoglycemics

aching abdominal pain in LLQ, constipation or diarrhea, n/v, low grade fever, LLQ tenderness

Sx of diverticulitis

T/F: Migraine with aura, any age, OC risk unacceptable

True

COCs decrease the effectiveness of which medications?

Tylenol, anticoagulants, benzos, oral hypoglycemics, methlydopa

Crohn's disease is associated with increased risk of malignancy involving the: a. small bowel b. large intestine c. duodenum d. stomach

a. small bowel

the clinical indication for the use of lubiprostone (Amitiza) is for: a. the tx of constipation that is not amenable to standard therapies b. intervention in intractable diarrhea c. control of intestinal inflammation d. the relief of intestinal spasms

a. the tx of constipation that is not amenable to standard therapies

pertaining to the use of sliding scale insulin in response to elevated blood glucose, which of the following best describes current best practice? a. the use of this type of sliding scale insulin tx is discouraged as this method tx hyperglycemia after it has already occurred b. sliding scale insulin in response to elevated glucose is a safe and helpful method of txing hyperglycemia c. delivering insulin in this manner is acceptable within the acute care hospital setting only d. the use of the sliding insulin scale is appropriate in the tx of type 1 diabetes only

a. the use of this type of sliding scale insulin tx is discouraged as this method tx hyperglycemia after it has already occurred

Characteristics of pain in pancreatic cancer?

abrupt in onset steady boring severe worse with walking and lying supine improved with sitting and leaning forward

Jobs linked to epididymitis

accountant programmer truck driver ----sitting for a long time-----

Side effects of febuxostat (Uloric)

associated with rash, nausea, and reduced liver function.

the NP is advising a 58 y/o woman about the benefits of a high fiber diet. which of the following foods provides the highest fiber content? a. a small banana b. 1 cup of cooked oatmeal c. a 1/2 cup serving of brown rice d. a medium-sized blueberry muffin

b. 1 cup of cooked oatmeal

the average incubation time for HAV is approximately: a. 10 days b. 28 days c. 60 days d. 6 months

b. 28 days

a 73 y/o pt is thought to have benign prostatic hyperplasia. what would be part of the initial workup? a. digital rectal exam (DRE) only b. DRE, urinalysis, PSA c. PSA only d. PSA, DRUE, BUN, Cr

b. DRE, urinalysis, PSA

a 45 y/o female pt has a screening TSH performed. Her TSH value is 13 mU/L. It was repeated in one wk and found to be 15 mU/L. What explains this finding? a. subclinical hypothyroidism b. hypothyroidism c. transiety hypothyroidism d. hyperthyroidism

b. hypothyroidism

a pt with a past hx of treatment for hyperthyroidism is most likely to exhibit: a. a euthyroid state b. hypothyroidism c. hyperthyroidism d. subclinical hypothyroidism

b. hypothyroidism

which of the following medications is representative of high-intensity statin therapy? a. pravastain 40mg b. rosuvastin 20mg c. simvastatin 40 mg d. lovastain 20mg

b. rosuvastin 20mg

"Hot nodules" such as thyrotoxic adenomas are always ________ (benign or malignant)

benign

what is the AM fasting glucose goal for a 75 y/o pt who has diabetes? a. 80-100 mg/dL b. 100-120 mg/dL c. 80-120 mg/dL d. 120-140 mg/dL

c. 80-120 mg/dL

the most common source of hepatitis A infection is: a. sharing intravenous drug equipment b. cooked seafood c. contaminated water supplies d. sexual contact

c. contaminated water supplies

the greatest risk of transmitting HIV is during: a. the acute phase. b. the time that detectable antibody is present c. high viral load periods d. late infection phase

c. high viral load periods

which of the following tx agents is commonly used to help with sleep difficulties such as insomnia associated with PTSD? a. duloxetine b. bupropion c. mirtazapine (remeron) d. zolpidem

c. mirtazapine (remeron)

in caring for a pt with OA of the knee, you advise that: a. straight leg raising should be avoided b. heat should be applied to painful joints after exercise c. quadriceps- strengthening exercises should be performed d. physical activity should be avoided

c. quadriceps- strengthening exercises should be performed

a sexually active male pt presents with epididymitis. what finding is likely? a. abnormal urinalysis b. dysuria c. recent hx of heavy physical exercise d. scrotal edema

c. recent hx of heavy physical exercise

Out-patient management PUD: H2 receptor antagonists

climetidine (tagamet) , ranitidine, famotidine, nizatidine

What are pancreatic pseudocysts?

consist of benign pockets of fluid lined with scar or inflammatory tissue

gouty crystals that fill tophi precipitate more easily in _____ areas of the body.

cooler

the use of which of the following medications can induce thyroid dysfunction? a. sertraline b. venlafaxine c. bupropion d. lithium

d. lithium

the American Academy of Orthopaedic Surgeons strongly recommends all of the following therapeutic agents for the management of symptomatic OA of the knee except: a. oral NSAIDs b. topical NSAIDs c. tramadol d. opioids

d. opioids

Classic findings: (COPD or emphysema) increased A-P diameter pursed-lip breathing hyperressonance decreased tactile fremitus and egophony flattended diaphragm with hyperinflation decreased breath and heart sound

emphysema

which gender is more often dx with IBS? male or female?

female

Regular insulin last from? (in reference to eating)

from meal to meal

__________ulcer, pain worsens when eating

gastric

Type of strain: involves a complete tear of a ligament. There is joint instability, severe pain, edema, tenderness, and ecchymosis; usually unable to bear weight due to pain.

grade III sprain

Hyperthyroidism, Hypothyroidism or Both -smooth silky skin

hyper

Hashimoto's thyroiditis is the most common etiology of

hypothyroidism in the U.S., more common in women and the average age at onset from 30-60 yrs pg 846-847

Regular insulin (Humulin R, Novolin R) *short acting* When does it peak?

in 2-3 hours

pain in Slipped Capital Femoral Epiphysis (SFCE) vs pain Legg-Calve Perthes disease (LCPD)

in SCFE, there is "acute onset" "severe pain" with "inability to ambulate or move hip". Pain "goes down the thigh to the knee", is more serious, requires "immediate referral to ortho"and can present with asymmetric leg length. in LCPD, insidious onset, pain in goes up the thigh and migrates to the groin/lateral hip

In CAP... tactile fremitus and egophony is (decreased? increased?)

increased + has abnormal pectoriloquy (whispered sounds are louder)

Causes of secondary gout

increased catabolism and purine turnover: -psoriasis -myeloproliferative and lymphoproliferative diseases -chronic hemolytic anemia conditions with decreased renal uric acid clearance - intrinsic kidney disease and renal failure.

Sickle Cell disease mgmt and goal of treatment

keep hydrated ensure adequate oxygenation analgesics for pain control aBx in presence of infection may need transfusions hydroxyurea 35mg/kg/day --stimulate fetal hemoglobin genetic counseling confirm hepatitis immunity

What low back injury present with Stiffness, spasm, and decreased ROM usually the erector spinae muscle. Sitting usually aggravates the pain, maybe some relief lying supine on a firm surface. In addition, poor posture, scoliosis, and spinal stenosis can be predisposing factors.

lumbosacral strain

Term describing patterns of skin lesions reticular=

netlike cluster

Anti-HBc (Hep B ) Anti-HBc(+) means

ongoing HB infection

doxazosin is associated with a risk of:

orthostatic hypotension

During an Addisonian crisis or acute adrenal failure (crisis), symptoms include: (5)

pain in lower back, abdomen, and or legs severe vomiting and diarrhea low bp Loss of LOC hyperkalemia

Anal fissure goal therapy? First line treatment (2) When unresponsive to 1st line therapy, what are three other options, least to greatest

prevent constipation Increases dietary fiber/supplementation & stool softner (mineral oil useful, long-term use related to malabsorption of A,D,E,K and fatty acids) 1.] Intraanal NTG 0.4% (relax sphincter,^ blood flow) 2.] Injection of botulinum toxin (Botox) into the sphincter, relieves spasms that are believed to cause the pain 3.] surgical sphincterotomy

Weight gain, increased appetite, fatigue, depression, acne, hirsutism caused by too much of which hormone? estrogen or progestin?

progestin

Infant with PERRLA, unilateral white reflex noted.....

retinoblastoma referral to ophalamology

Name some gluten free things

rice, corn, potatoes, peanuts, soy, beans, meat, dairy, all fruits/veggies

Term describing patterns of skin lesions annular=

ring Lyme disease

Torsion of the appendix testis

school aged boy c/o blue colored round mass located on testicular surface. Caused by infarction and necrosis of the appendix testis d/t torsion. REFER TO ED

Diagnostics of ulcerative colitis Management of ulcerative colitis

sigmoidoscopy establishes diagnosis mesalamine suppositories or enemas for 3-12 weeks hydrocortisone suppositories

psychomotor agitation

sxs present in both anxiety and depression

hypospadias epispadias

urethral meatus on ventral aspect of penis urethral meatus on dorsal (top) of penis

***LEIK*** DJD/OA Non-pharmacologic Mgmt.

■ Exercise (with care) at least three times a week. Lose weight. Stop smoking. ■ Isometric exercises to strengthen quadriceps muscles (knee OA). ■ Exercise (swimming, walking, biking); resistance-band exercises. ■ Avoid aggravating activities. Cold or warm packs. Ultrasound treatment. ■ Walking aids. Patellar taping (reduces load on knees) by physical therapist. ■ Alternative medicine: Glucosamine supplements, SAM-e, Tai-Chi exercises, acupuncture. ■ Join Arthritis Self-Management Program (ASMP) with physical therapy.

***LEIK*** DJD/OA Pharmacologic Mgmt.

■ First-line medication: is acetaminophen 325 mg to 650 mg every 4 to 6 hours (maximum 4 g per day) PRN. OR ■ Tylenol Extra Strength 500 mg to 1,000 mg every 6 hours (Max =6 tablet/dayPR) ■ If no relief with high-dosed acetaminophen, switch to a short-acting NSAID. ■ Start with NSAIDs such as ibuprofen (Advil) 1 to 2 tablets every 4 to 6 hours or naproxen (Aleve) BID or Anaprox DS 1 tablet every 12 hours PRN. ■ For added gastrointestinal (GI) protection, add a PPI (omeprazole) or misoprostol(Cytotec). ■ If patient is at high risk for both GI bleeding and cardiovascular (CV) side effects, avoid NSAIDs. ■ Age greater than 75 years: Use topical NSAIDs for treatment (versus oral form). ■ Rule out osteoporosis. Order bone density testing. - GI bleed risk factors: History of uncomplicated ulcer, aspirin, Coumadin, PUD, platelet disorder. - Opioid analgesics: Avoid if possible (especially if patient is a recovering addict/alcoholic). [Topical Medicine] ■ NSAID: Diclofenac gel (Voltaren Gel); apply to painful area and massage well into skin BID. ■ Capsaicin cream: Apply to painful area QID. Avoid contact with eyes/mucous membranes. ■ Do not use on wounds/abraded skin. Avoid bathing/showering afterward (so that it is not washed off). ■ Capsaicin is from chili peppers. Also used to treat neuropathic pain (e.g., postshingles). [NSAIDs] ■ Highest risk for GI bleeding: Ketorolac (Toradol) and piroxicam (Feldene) ■ Lowest risk for GI bleeding: Ibuprofen, celecoxib (Celebrex) ■ Highest risk for CV events: Diclofenac and celecoxib at higher doses ■ Lowest CV risk: Naproxen

Gravida or "G"

■ Gravida or "G": The number of pregnancies of the mother (no matter what the outcome).

Use of Antihypertensives With Pregnant Women Used for women with preexisting hypertension or for moderate-to-severe preeclampsia or eclampsia.

■ Methyldopa (Aldomet) ■ Hydralazine (Apresoline) ■ Labetalol (Normodyne)

Effect of exercise on insulin resistance (IR)... approximately _____ of the body's insulin-mediated glucose uptake occurs in ___________ with regular aerobic exercise, insulin resistance is reduced by approximately ________ with reducing effects of exercise persisting for _______ after the activity. Ppl with DM should perform at least ____ min/week of ______ intensity aerobic activity that is ___________ of their max HR. Exercise should be spread out over at least _____ days per week without going no more than _____ without exercise. In addition, some form of ___________ at least twice per week. NOTED: _____ aerobic or _____ activity is potentially CI in the presence of proliferative or severe non-proliferative retinopathy due to the possible risk of ________ or __________.

80% skeletal muscle 40% 48 hrs 150min/wk moderate 50-70% 5 48 hours resistance exercise aerobic resistance vitreous hemorrhage retinal detachment

What is the weight constraint for the patch?

<180 lbs

at what level should you immediately treat hypocalcemia?

<6.5mg/dL pg 830

Illegal blood alcohol level

>8

Screening for Genetic Disorders ***Whites/Caucasians**,Northern European or Ashkenazi Jewish heritage. Less common with African ancestry, uncommon with Asian descent Test Prior to or in early pregnancy

(cystic fibrosis)

Hale's Lactation Risk Category • L2 - Safer

- Limited number of women studied without risk • Macrolides (azithromycin, clarithromycin, erythromycin) • Nitrofurantoin (Macrodantin®, Macrobid®) • Cephalosporins • 2nd-generation antihistamines (loratadine [Claritin®], et al.) - Limited number of women studied without risk • Prednisone • SSRIs

Hale's Lactation Risk Category • L3 - Moderately safe

- No controlled study or controlled study shows minimal, non life-threatening risk • TMP-SMX (Bactrim®) • FQ antibiotics (-floxacin suffix, ciprofloxacin, levofloxacin) • 1st-generation antihistamines (diphenhydramine [Benadryl®], et al.) • Doxycycline

Pathophysiology of celiac disease and gluten

- gluten passes through lamina propria when intestinal permeability is increased (stress, infection) - interacts with antigen presenting cells - releases inflammatory mediators (antibodies, cytokines) - chronic inflammation = villous atrophy = malabsorption

How do you diagnose Celiac disease

- serology testing "celiac panel": after on gluten diet for 4-6 wks prior; finds specific IgA antibodies and is very accurate - intestinal biopsy - resolution of sx on gluten free diet (genetic testing not routine)

Normal thyroid levels

-0.4-4.0 (goal of tx 1.2) -Monitor every 6-8 weeks until normal (no early than 6)

Baker Acts = days? Is? For who?

-72h/3d -Involuntary detention -intended to eval/tx individuals who are at high risk for suicide or homicide

Biguanide metformin (Glucophage) A1C reduction __________ Rare Risk of use _____________ Monitor what lab?____________ Common SE? CI (4)______________

-A1C reduction = 1-2% w/ intensified use -lactic acidosis -Creatinine - GI upset (diarrhea) **CONTRAINDICATIONS** -IMPAIRED RENAL FX -HEPATIC FAILURE -HEART FAILURE -PPL >80yo for the purpose of practice, not the test, to combat, GI upset, use once daily extended use instead of BID dosing.

Danger Signals Skin& Integumentary (6)

-Angioedema/anaphylaxis, Steven-Johnson, Meningococcemia, RMSF, and Lyme disease

Describe the prostate with cancer

-Asymmetry -Adenopathy -Distension -Hard prostate w/obscure boundaries -Nodules (often)

A1C certification & standards

-Certified by the National Glycohemoglobin -Standardization Program (NGSP) -Standardized to the Diabetes C-ontrol and Complications Trial (DCCT) assay

Hot vs Cold thyroid nodules

-Cold - not active - more likely to be cancerous - need bx -Hot - active - more likely to be benign

Danger Signals of Women's Health (2)

-Dominant Breast mass that is attached to surrounding tissue, ruptured tubal ectopic pregnancy

Subclinical hypothyroidism lab findings When do you treat

-Elevated TSH with normal T4 -above 10

DM preventative Care

-Flu shot annually -Prevnar -Asa 81 if high risk MI, CVA (not for <30) -Eye doc - early dilated exam - if type II eye exam on dx, if type I than eye exam 5 years after dx -Podiatrist - once to twice a year, esp with older DM -Dental care - associated with heart disease

Hashimotos Tx plan

-Start low dose levothyroxine 25 to 50 mcg / day -Start with 25 for elderly w/ heart disease - (watch for angina, MI, a-fib) -Increase dose by 25mcg every 8 weeks until TSH is normal -Report palpitations, nervousness, tremors - too high dose

Concerning Legg Calve Perthes Disease, when do you refer to orthopedic?

-older than 6, -more than 1/2 of femoral head is involved very fragile

In what (4) instances would metformin be considered for IFG?

1. A1C 5.7-6.4% 2. Ages <60 3. BMI >35 4. Women with hx of gestational diabetes

Additional complaints with RA besides joint complaints?

1. Swelling, edema, redness at joint 2. Malaise, fatigue 3. Weakness, anorexia, weight loss

Which antibiotic below could be used to treat bacterial folliculitis? 1. cephalexin (Keflex) 2. cefadroxil (Duricef) 3. amoxicillin (Amoxil) 4. cefdinir (Omincef)

1. cephalexin (Keflex) 2. cefadroxil (Duricef) 4. cefdinir (Omincef) *DONT USE amoxicillin for staph skin infection

"Shift to Left" *LEUKOCYTOSIS: increase in total WBC *NEUTROPHILIA is the elevation of neutrophils in circulation. [neutrophilia]: absolute neutrophil count (ANC) greater than 7000 n/mm3 *BANDEMIA: elevation in # of bands or young neutrophils -usually <4%, when elevated, the absolute band count (ABC) is >500mm3 [ABC]= %of neutrophils x by total WBC Ex: which signifies shift to left? 1.) total WBCs 16,500 mm3; neutrophils 66%; bands 8%; lymphocytes 22% 2.) total WBCs 18,100 mm3; neutrophils 55%; bands 3%; lymphocytes 28%

1.) total WBCs 16,500 mm3; neutrophils 66%; bands 8%; lymphocytes 22%

S/S of perforation?

1.] Severe epigastric pain, 2.] board like abdomen *Rigidity indicates peritonitis* 3.] *quiet bowel sounds- omninous* 4.] Xray shows air in diaphragm

There are 4 types of rosacea forms (describe/treatment) 1.] erythematotelangiectatic type 2.] papulopustular type 3.] phymatous type 4.} ocular type

1.] erythematotelangiectatic type- facial flushing and burning or stinging 2.] papulopustular type usually affects middle-age women, present with red central portion of their face that contains erythematous papillose surmounted by pinpoint pustules - (Respond well to ABX) 3.] phymatous type marked skin thickenings and irregular surface modularities of the nose, chin, forehead, one or both ears and or the eyelids.- (respond to mechanical dermabrasion, laser peel, and surgical shave) 4.} ocular type various manifestations: conjunctivitis, blepharitis, inflammation of the eyelids, conjunctival telangiectasis.

Between what ages are appendicitis most common in?

10 and 30 years (Rare in infants and adults)

Normal Free T4 is ___________ to __________.

10 to 27

According to the latest recommendations from the National Osteoporosis Foundation (NOF), the calcium intake goal should be the equivalent of __________ mg/d for men between 50 to 70 years of age, and the dose should be _________ mg/day for women 51 years and older and men older than age 70 years.

1000; 2000

By definition, a woman is in menopause when she has had no naturally occurring menstrual period for ______ months.

12

What is the average incubation period of Hepatitis B?

12-14 weeks but can be anywhere from 6 weeks to 6 months

gynecomastia associated with puberty begins at

12-14 yrs, duration is six months and then regression is expected

clinical presentation in patients with OA includes an insidious onset of symptoms, including use-related joint pain that is relieved by rest and joint stiffness that occurs with rest but resolves with less than ______ minutes of activity.

15

Short acting rapid onset insulin (Lispro aka Humalog) onset in ___________minutes and peak @ ___________ hours and last for ___________ hours

15-30 minutes 30 mins - 2.5 hours 3 -6.5 hours

anterior fontanel close

18 months- 24 as well

How much should the fundus grow per week?

1cm/wk

Treatment for Bullous impetigo caused by MSSA?

1st gen cephalosporin -cephalexin (Keflex) 2nd gen cephalosporin- cefuroxime axetil (Ceftin) cefprozil (Cefzil) cefaclor (Ceclor)

Erectile dysfunction: treatment

1st line: phosphodiesterase type 5 inhibitors Take viagra and levity on empty stomach. Take one dose 30-60 min before sex.

Levothyroxine should NOT be taken within ______ hour(s) of cations such as calcium, aluminum, magnesium, or prenatal vitamins etc.

2

Stage __________ presents months later. COMMON SIGNS: Classic rash may reappear w/ multiple lesions accompanied by HA, myalgia, arthralgias, and fatigue. LESS COMMON SIGNS: acute facial nerve paralysis (Bell's Palsy), AVB, aseptic meningitis

2

The presence of an umbilical hernia is considered normal until what age?

2

Is there a Hep A vaccine?

2 doses 6-12 mo apart OR alternative hep A/hep B accelerated dosing schedule, not a live virus, well tolerated w/o systemic reaction, lasts 15-25 yrs

Which medication may worsen symptoms of GERD? 1. Levothyroxine 2. Amlodipine (Norvasc) 3. Fluticasone (Flovent) 4. Atorvastatin (Lipitor

2. Amlodipine (Norvasc) Ca makes muscles contract, LES opens by muscle, if block it then muscles dont close/open as forcefully

A 30 year old NP who is a recent graduate has a job interview. She feels very anxious and has a noticeable tremor at the time of her interview. What type tremor is this? 1. Not a real tremor 2. Physiologic tremor 3. Essential tremor 4. Parkinsonian tremor

2. Physiologic tremor

Mrs. Jones is a newly diagnosed T2DM. In which items(s) below would metformin be a safe choice for her? Choose all that apply. 1. Active Hepatitis C 2. Serum creatinine: 1.2 3. Heart failure 4. Binge drinking

2. Serum creatinine: 1.2

Pap when and how often?

21 and then every 3 years unless abnormal.

Per the ADA, what is the BMI cut point for Asians?

23 kg/m2

When is blood glucose tested in low-risk mothers?

24-28 weeks

Diagnostic option for evaluating appendicitis in a pregnant women: 1] CT 2] MRI 3] abd/pelvic US

2] MRI abd/pelvis US is 1st option normally, but in pregnancy it does not provide much use and may not be able to visualize appendix due to pregnant state, next step in CT. Pregnant can't have CT, but can have MRI (lack of ionizing radiation)

with an 8am dose of the following insulin forms, followed by an inadequate dietary intake and/or excessive energy use, at approximately what time would hypoglycemia be most likely to occur with NPH insulin?

2p-10p NPH (Novolin N) (Humulin N) *intermittent acting* onset: 1-2 hr peak: 6-14 hr duration: 16-25 hr go by peak to determine risk of hypoglycemia

Stage __________ presents approximately 1 yr later after initial infection COMMON SIGNS: Musculoskeletal s/s usually persist (joint pain, frank arthritis, and joint damage). Neuropsychiatric symptoms (memory problems, depression, neuropathy) LESS COMMON SIGNS: acute facial nerve paralysis (Bell's Palsy), AVB, aseptic meningitis

3

Because chlamydia reinfection is common, all women with chlamydial infection should be rescreened __________ after completing antimicrobial treatment.

3-4 months

What is the most common presenting symptom of pseudotumor cerebrii? 1. Tremor 2. Vertigo 3. Headache 4. Blurred vision

3. Headache

What disease/condition has the potential to produce a drug-disease interaction secondary to triptan use? 1. Diabetes 2. Hypothyroidism 3. Hypertension 4. Menstruation

3. Hypertension

Which antibiotic below should not be used to treat plain cellulitis? 1. cephalexin (Keflex) 2. cefadroxil (Duricef) 3. amoxicillin (Amoxil) 4. cefdinir (Omincef)

3. amoxicillin (Amoxil)

Short acting insulin like regular insulin aka Humalin R or Novolin R onset in ___________minutes and peak @ ___________ hours and last for ___________ hours

30 min - 1 hour 2-3 hours 4-6 hours

Regular insulin (Humulin R, Novolin R) *short acting* onset of action ______ peak_______ duration_________

30min-1 hr 2-3 hr 4-6 hr

A 72 year old female describes an intermittent facial pain along the right side of her face and head. It is sharp, piercing and started last night. Her neurologic exam is WNL. Which statement is correct? 1. She should have a CT of the brain. 2. This has classic shingles. 3. She should be given prednisone for 10 days. 4. This is trigeminal neuralgia involving the 5th cranial nerve.

4. This is trigeminal neuralgia involving the 5th cranial nerve.

AACE Guidelines reccomend treatment of patients with TSH > ____ mIU/L if the patient has a goiter or if thyroid antibodies are present. The presence of symptoms compatible with hypothyroidism: infertility, pregnancy, or imminent pregnancy, would also favor treatment.

5

When should IUD be placed when being used for Emergency contraception?

5-6 days after intercourse

For long-term prevention, patients should be given _________ IU of vitamin D3 once or twice per month plus 1000 to 2000 IU of vitamin D3 daily.

50,000

How long does diaphragm get left in the vagina after intercourse?

6 hours

When does the sponge get inserted prior to intercourse? Left in place afterward? How long does the sponge provide protection? When does the sponge NEED to be taken out?

6 hours: 6 hours 12 hours 24 hours--increase risk of TSS after 24 hours

If the TSH is 0.5 to 2.0 mIU/mL and symptoms are resolved, continue the dose and measure TSH in __________ months. Then every __________ year(s) or when symptomatic.

6,1

Herpes Zoster (Shingles) is managed with antiviral if patient presents w/in ____ hours of symptoms.

72 hours

What is the therapy of choice for symptom and relief and healing of erosive esophagitis

8-week course of PPIs

Knee and hip joint replacement should be considered when pain cannot be adequately controlled, when function is severely compromised, or when more than _____% of the articular cartilage is worn away.

80

Systemic Lupus Erythematosus (SLE)

A multi system autoimmune disease that is more common in women (9:1 ratio). Remissions and exacerbations More common in African American and Hispanic women. Organ systems affected are the skin, kidneys, heart, and blood vessels. Milder form of lupus is called cutaneous lupus erythematosus. Management Refer to rheumatologist (topical and oral steroids, Plaquenil, methotrexate, biologics). Patient Education ■ Avoid sun 10 a.m. to 4 p.m. (causes rashes to break out). ■ Cover skin with high SPF (UVA and UVB) sunblock. ■ Sun-protective clothing such as hats with wide brims, long-sleeved shirts. ■ More sensitive to indoor fl uorescent lighting. Use non-fl uorescent light bulbs

Severe Preeclampsia

A primigravida who is in the late third trimester of pregnancy (more than 34 weeks) complains of a sudden onset of severe recurrent headaches, visual abnormalities (blurred vision, scotomas), and pitting edema. Edema easily seen on the face/eyes and fingers. Sudden rapid weight gain within 1 to 2 days (greater than 2 lbs/week). New onset of right upper quadrant abdominal pain. Blood pressure more than 140/90 mmHg. Urine protein 1+ or higher. Sudden decrease in urine output (oliguria). Nausea and vomiting is a worrisome sign (encephalopathy). If seizures occur, it is reclassified as eclampsia. Earliest time period that preeclampsia/eclampsia can occur is at 20 weeks of gestation (and up to 4 weeks postpartum). Only known "cure" is delivery of fetus/baby.

The most common human papillomavirus types associated with cutaneous, nongenital warts include: A. 1, 2, and 4. B. 6 and 11. C. 16 and 18. D. 32 and 36.

A. 1, 2, and 4

You see a kindergartner with impetigo and advise that she can return ________ hours after initiating effective antimicrobial therapy. A. 24 B. 48 C. 72 D. 96

A. 24

When a pregnant woman takes a teratogenic drug, the fetal effects are usually seen in: A. Certain target organs in a predictable manner. B. Random body systems without a predictable pattern. C. Select organ systems with random outcomes. D. A pattern of systemic injury.

A. Certain target organs in a predictable manner. B. Random body

A likely causative pathogens in a 25-year-old man with acute epididymitis who has sex with men is: A. Escherichia coli. B. Mycoplasma spp. C. Chlamydia trachomatis. D. Acinetobacter baumannii.

A. Escherichia coli.

A 25-year-old woman presents with sinus arrhythmia. Which of the following statements is correct concerning sports participation? A. Full activity should be encouraged. B. Weight lifting is contraindicated. C. An exercise tolerance test is advisable. D. A CCB should be prescribed.

A. Full activity should be encouraged.

Genital Candida albicans infection in men typically presents with all of the following except: A. Penile discharge B. Balanitis C. Groid-fold involvement D. Scrotal excoriation

A. Penile Discharge (relatively uncommon in men with GU infections with the following infections) most common

When prescribing pulse dosing with itraconazole for the treatment of fingernail fungus, the clinician realizes that: A. a transient increase in hepatic enzymes is commonly seen with its use. B. drug-induced leukopenia is a common problem. C. the patient needs to be warned about excessive bleeding because of the drug's antiplatelet effect. D. its use is contraindicated in the presence of iron- deficiency anemia

A. a transient increase in hepatic enzymes is commonly seen with its use

In counseling a postmenopausal woman, you advise her that systemic estrogen therapy users can possibly experience: A. an increase in breast cancer rates with long-term use. B. reduction in high-density lipoprotein cholesterol. C. a 10% increase in bone mass. D. no change in the occurrence of osteoporosis.

A. an increase in breast cancer rates with long-term use.

During a preparticipation sports examination, you hear a grade 2/6 early-to mid-systolic ejection murmur, heard best at the second intercostal space of the left sternal border, in an asymptomatic young adult. The murmur disappears with position change from supine to stand position change. This most likely represents: A. an innocent flow murmur. B. mitral valve incompetency. C. aortic regurgitation. D. mitral valve prolapse (MVP).

A. an innocent flow murmur.

More common etiologies of acute lower-extremity atherosclerotic arterial disease include: A. arterial embolism withunderlying afib B. chronic venous insufficiency C. extension of venous thrombosis D. vessel trauma

A. arterial embolism with underlying afib

When taken with warfarin, which of the following causes a possible increased anticoagulant effect? A. clarithromycin B. carbamazepine C. pravastatin D. sucralfate

A. clarithromycin

Treatment of superficial venous thrombophlebitis in a low-risk, stable patient includes use of: A. compression stockings B. acetaminophen C. warfarin D. heparin

A. compression stockings

You have initiated therapy for an 18-year-old man with acne vulgaris and have prescribed doxycycline. He returns in 3 weeks, complaining that his skin is "no better." Your next action is to: A. counsel him that 6 to 8 weeks of treatment is often needed before significant improvement is achieved. B. discontinue the doxycycline and initiate minocycline therapy. C. advise him that antibiotics are likely not an effective treatment for him and should not be continued. D. add a second antimicrobial agent such trimethoprim- sulfamethoxazole

A. counsel him that 6 to 8 weeks of treatment is often needed before significant improvement is achieved.

A 24-year-old woman presents with hive-form linear lesions that develop over areas where she has scratched. These resolve within a few minutes. This most likely represents: A. dermographism B. contact dermatitis C. angioedema D. allergic reaction

A. dermographism (Development of urticaria with scratching, slapping, stroking or rubbing)

Which of the following is the best treatment option for cellulitis when risk of infection with a methicillin-resistant pathogen is considered low? A. dicloxacillin B. amoxicillin C. metronidazole D. trimethoprim-sulfamethoxazole

A. dicloxacillin

Lifestyle modification for patient with Raynaud phenomenon includes: A. discontinuing cigarette smoking B. increasing fluid intake C. avoiding placing hands in warm water. D. discontinuing aspirin use.

A. discontinuing cigarette smoking

Typically, the earliest sign of lower-extremity venous insufficiency is: A. edema B. altered pigmentation C. skin atrophy D. shiny skin

A. edema -usually unilateral is best true + finding

Which of the following do you expect to find in the assessment of the person with urticaria? A. eosinophilia B. low erythrocyte sedimentation rate C. elevated thyroid-stimulating hormone level D. leukopenia

A. eosinophilia

A 78-year-old resident of a long-term care facility complains of generalized itchiness at night that disturbs her sleep. Her examination is consistent with scabies. Which of the following do you expect to find on examination? A. excoriated papules on the interdigital area B. annular lesions over the buttocks C .vesicular lesions in a linear pattern D. honey-colored crusted lesions that began as vesicles

A. excoriated papules on the interdigital area

One of the most common trigger agents for contact dermatitis is: A. exposure to nickel B. use of fabric softener C. bathing with liquid body wash D. eating spicy foods

A. exposure to nickel metals (i.e nickel), rubber additives on shoes/gloves an] and topical medications

An 88-year-old, community-dwelling man who lives alone has limited mobility because of osteoarthritis.Since his last office visit 2 months ago, he has lost 5%of his body weight and has developed angular cheilitis. You expect to find the following on examination: A. fissuring and cracking at the corners of the mouth B. marked erythema of the hard and soft palates C. white plaques on the lateral borders of the buccal mucosa D. raised, painless lesions on the gingival

A. fissuring and cracking at the corners of the mouth

Which of the following is representative of the presentation of secondary syphilis? A. generalized rash B. chancre C. pupillary alterations D. aortic regurgitation

A. generalized rash

A Tzanck smear that is positive for giant multinucleated cells was taken from a lesion caused by: A. herpes virus B. S. aureus C. streptococci D. allergic reaction

A. herpes virus

A 38-year-old woman with advanced human immunodeficiency virus (HIV) disease presents with a chief complaint of a painful, itchy rash over her trunk. Examination reveals linear vesicular lesions that do not cross the midline and are distributed over the posterior thorax. This presentation is most consistent with: A. herpes zoster B. dermatitis herpetiformis C. molluscum contagiosum D. impetigo

A. herpes zoster

The mechanism of action of pimecrolimus (Elidel) in the treatment of atopic dermatitis is as a/an: A. immunomodulator B. antimitotic C. mast cell activator D. exfoliant

A. immunomodulator

Characteristics of onychomycosis include all of the following except: A. it is readily diagnosed by clinical examination B. nail hypertrophy C. brittle nails D. fingernails respond more readily to therapy than toenails

A. it is readily diagnosed by clinical examination

Which of the following is not a potential adverse effect with long-term high-potency topical corticosteroid use? A. lichenification B. telangiectasia C. skin atrophy D. adrenal suppression

A. lichenification

You see a 34-year-old man with atopic dermatitis localized primarily on the arms who complains of severe itching. The condition becomes worse at night and interferes with his sleep. You recommend: A. taking a bedtime dose of antihistamine B. taking a bedtime dose of acetaminophen C. taking a hot shower prior to bedtime D. applying a warm compress to the affected areas 30 minutes prior to bedtime

A. taking a bedtime dose of antihistamine A bedtime dose of antihistamine can yield tremendous relief from itching and improve sleep. hydroxyzine (Atarax) seems to provide better relief of itching than other antihistamines. cetirizine (Zyrtec) is a less sedating antihistamine thatt is a metabolite of hydroxyzine.

The most common type of injury causing a sport related meniscal tear involves: A. twisting of the knee. B. hyperextension of the knee. C. repetitive hard impact on the knee (i.e., running on hard surface). D. an unknown origin in most cases.

A. twisting of the knee. This twist-type injury to the knee. This condition can also be found in older, sedentary adults; in this case, the injury is usually because of degenerative changes. Larger tears: - knee locks, popping sound - "gives out." Effusion = knee tightness and stiffness. With certain positions, there is often sudden onset, sharp, localized pain, on the medial aspect of the knee.

In counseling a patient with seborrheic dermatitis on the scalp about efforts to clear lesions, you advise her to: A. use ketoconazole shampoo. B. apply petroleum jelly nightly to the affected area. C. coat the area with high-potency corticosteroid cream three times a week. D. expose the lesions periodically to heat by carefully using a hair dryer.

A. use ketoconazole shampoo

All of the following organisms have been implicated in the development of rosacea except: A. viruses B. bacteria C. yeast D. mites

A. viruses

ADA suggest A1C goals based on Age for T2DM: for older patients _________

A1C<8%

When the nurse is conducting sexual history from a male adolescent, which statement would be most appropriate to use at the beginning of the interview?

ANS: "Often boys your age have questions about sexual activity." Start the interview with a permission statement. This conveys that it is normal and all right to think or feel a certain way. Avoid sounding judgmental.

A 55-year-old man is in the clinic for a yearly check-up. He is worried because his father died of prostate cancer. The nurse knows that which tests should be done at this time? Select all that apply.

ANS: Blood test for prostate-specific antigen Digital rectal examination Prostate cancer is typically detected by testing the blood for prostate-specific antigen (PSA) or by a digital rectal exam (DRE). It is recommended that both PSA and DRE be offered to men yearly, beginning at age 50 years. If the PSA is elevated, then further lab work or a transrectal ultrasound (TRUS) and biopsy may be recommended.

A 55-year-old man is experiencing severe pain of sudden onset in the scrotal area. It is somewhat relieved by elevation. On examination the nurse notices an enlarged, red scrotum that is very tender to palpation. It is difficult to distinguish the epididymis from the testis, and the scrotal skin is thick and edematous. This description is consistent with which of these?

ANS: Epididymitis Epididymitis presents as severe pain of sudden onset in the scrotum that is somewhat relieved by elevation. On examination, the scrotum is enlarged, reddened, and exquisitely tender. The epididymis is enlarged and indurated and may be hard to distinguish from the testis. The overlying scrotal skin may be thick and edematous. See Table 24-6 for more information and for descriptions of the other terms.

When the nurse is performing a genital examination on a male patient, which of these actions is correct?

ANS: Have the patient shift his weight onto the left (unexamined) leg when palpating for a hernia on the right side. When palpating for the presence of a hernia on the right side, ask the male patient to shift his weight onto the left (unexamined) leg. It is not appropriate to auscultate for a bruit over the scrotum. When palpating for lymph notes, palpate the horizontal chain. The inguinal canal should be palpated whether or not a bulge is present.

The nurse is aware that which of these statements is true regarding the incidence of testicular cancer?

ANS: Men with a history of cryptorchidism are at greatest risk for development of testicular cancer. Men with undescended testicles (cryptorchidism) are at greatest risk for development of testicular cancer. The overall incidence of testicular cancer is rare. Testicular cancer has no early symptoms. When detected early and treated before metastasis, the cure rate is almost 100%.

The nurse is examining the glans and knows that which of these is a normal finding for this area?

ANS: Smegma may be present under the foreskin of an uncircumcised male. The glans looks smooth and without lesions and does not have hair. The meatus should not have any discharge when the glans is compressed. Some cheesy smegma may have collected under the foreskin of an uncircumcised male.

A 16-year-old boy is brought to the clinic for a problem that he refused to let his mother see. The nurse examines him, and finds that he has scrotal swelling on the left side. He had the mumps the previous week, and the nurse suspects that he has orchitis. Which of the assessment findings below support this diagnosis? Select all that apply.

ANS: Swollen testis Mass does not transilluminate Tender upon palpation Scrotal skin is reddened With orchitis, the testis is swollen, with a feeling of weight, and is tender or painful. The mass does not transilluminate, and the scrotal skin is reddened. Transillumination of a mass occurs with a hydrocele, not orchitis.

Which of these statements about the testes is true?

ANS: The cremaster muscle contracts in response to cold and draws the testicles closer to the body. When it is cold, the cremaster muscle contracts, which raises the scrotal sac and brings the testes closer to the body to absorb heat necessary for sperm viability. The lymphatics of the testes drain into the inguinal lymph nodes. The vas deferens is located along the upper portion of each testis. The left testis is lower than the right because the left spermatic cord is longer.

The nurse is inspecting the scrotum and testes of a 43-year-old man. Which finding would require additional follow-up and evaluation?

ANS: The skin on the scrotum is taut. Scrotal swelling may cause the skin to be taut and to display pitting edema. Normal scrotal skin is rugae, and asymmetry is normal with the left scrotal half usually lower than the right. The testes may move closer to the body in response to cold temperatures.

During an examination, the nurse notices that a male patient has a red, round, superficial ulcer with a yellowish serous discharge on his penis. On palpation, the nurse finds a nontender base that feels like a small button between the thumb and fingers. At this point the nurse suspects that this patient has:

ANS: a syphilitic chancre. This lesion indicates syphilitic chancre, which begins within 2 to 4 weeks of infection. See Table 24-4 for descriptions of the other options.

During an examination of an aging male, the nurse recognizes that normal changes to expect would be:

ANS: decreased penis size. In the aging male the amount of pubic hair decreases, the penis size decreases, and there is a decrease in the rugae over the scrotal sac. The scrotal sac does not enlarge.

A 62-year-old man states that his doctor told him that he has an "inguinal hernia." He asks the nurse to explain what a hernia is. The nurse should:

ANS: explain that a hernia is a loop of bowel protruding through a weak spot in the abdominal muscles. A hernia is a loop of bowel protruding through a weak spot in the musculature. The other options are not correct responses to the patient's question.

During a genital examination, the nurse notices that a male patient has clusters of small vesicles on the glans, surrounded by erythema. The nurse recognizes that these lesions are:

ANS: genital herpes. Genital herpes, or HSV-2, infections are indicated with clusters of small vesicles with surrounding erythema, which are often painful and erupt on the glans or foreskin. See Table 24-4 for descriptions of the other options.

When performing a genital assessment on a middle-aged man, the nurse notices multiple soft, moist, painless papules in the shape of cauliflower-like patches scattered across the shaft of the penis. These lesions are characteristic of:

ANS: genital warts. The lesions of genital warts are soft, pointed, moist, fleshy, painless papules that may be single or multiple in a cauliflower-like patch. They occur on the shaft of the penis, behind the corona, or around the anus, where they may grow into large grape-like clusters. See Table 24-4 for more information and for descriptions of the other options.

When performing a genitourinary assessment on a 16-year-old boy, the nurse notices a swelling in the scrotum that increases with increased intra-abdominal pressure and decreases when he is lying down. The patient complains of pain when straining. The nurse knows that this description is most consistent with a(n) _____ hernia.

ANS: indirect inguinal With indirect inguinal hernias there is pain with straining and a soft swelling that increases with increased intra-abdominal pressure, which may decrease when the patient lies down. These findings do not describe the other hernias. See Table 24-7 for descriptions of femoral and direct inguinal hernias.

The nurse is providing patient teaching about an erectile dysfunction drug. One of the drug's potential side effects is prolonged, painful erection of the penis without sexual stimulation, which is known as:

ANS: priapism. Priapism is prolonged, painful erection of the penis without sexual desire. Orchitis is inflammation of the testes. Stricture is a narrowing of the opening of the urethral meatus. Phimosis is the inability to retract the foreskin.

When the nurse is performing a genital examination on a male patient, the patient has an erection. The nurse's most appropriate action or response is to:

ANS: reassure the patient that this is a normal response and continue with the examination. When the male patient has an erection, the nurse should reassure the patient that this is a normal physiologic response to touch and proceed with the rest of the examination. The other responses are not correct and may be perceived as judgmental.

An older man is concerned about his sexual performance. The nurse knows that in the absence of disease, a withdrawal from sexual activity later in life may be due to:

ANS: side effects of medications. In the absence of disease, a withdrawal from sexual activity may be due to side effects of medications such as antihypertensives, antidepressants, or sedatives. The other options are not correct.

A 45-year-old mother of two children is seen at the clinic for complaints of "losing my urine when I sneeze." The nurse documents that she is experiencing:

ANS: stress incontinence. Stress incontinence is involuntary urine loss with physical strain, sneezing, or coughing that occurs due to weakness of the pelvic floor. Urinary frequency is urinating more times than usual (more than 5 to 6 times per day). Enuresis is involuntary passage of urine at night after age 5 to 6 years (bed wetting). Urge incontinence is involuntary urine loss from overactive detrusor muscle in the bladder. It contracts, causing an urgent need to void.

When performing a scrotal assessment, the nurse notices that the scrotal contents transilluminate and show a red glow. On the basis of this finding the nurse would:

ANS: suspect the presence of serous fluid in the scrotum. Normal scrotal contents do not transilluminate. Serous fluid does transilluminate and shows as a red glow. Neither a mass nor a hernia would transilluminate.

A 15-year-old boy is seen in the clinic for complaints of "dull pain and pulling" in the scrotal area. On examination the nurse palpates a soft, irregular mass posterior to and above the testis on the left. This mass collapses when the patient is supine and refills when he is upright. This description is consistent with:

ANS: varicocele. A varicocele consists of dilated, tortuous varicose veins in the spermatic cord caused by incompetent valves within the vein. Symptoms include dull pain or constant pulling or dragging feeling, or the individual may be asymptomatic. When palpating the mass, the examiner will feel a soft, irregular mass posterior to and above the testis that collapses when the individual is supine and refills when the individual is upright. See Table 24-6 for more information and for descriptions of the other options.

American Recovery and Reinvestment Act of 2009

ARRA- Law with provisions concerning the standards for the electronic transmission of health care data; intended to create jobs and promote investment and consumer spending during the recession that followed the financial collapse in 2008. No Republicans in the House and only 3 Republicans in the Senate voted for this bill, arguing against the massive growth in federal spending Obama shifted gov spending. much larger than first stimulus 168 billion vs. 787 billion

Adult to older male complains of acute onset of a swollen red scrotum that hurts. Accompanied by unilateral testicular tenderness with urethral discharge. Scrotum is swollen and erythematous with induration of the posterior epididymis. Sometimes may be accompanied by a hydrocele, and signs and symptoms of UTI. Some will have systemic symptoms such as fever. discharge ■ Positive Prehn's sign: Relief of pain with scrotal elevation.

Acute Bacterial Epididymitis edications ■ Age less than 35 years, give doxycycline PO BID × 10 days plus ceftriaxone 250 mg IM × one dose. Do not forget to treat sex partner. ■ Older male (greater than 35 years), give ofl oxacin (Floxin) 300 mg PO × 10 days or levofloxacin (Levaquin) 500 mg PO 7 to 10 days. ■ Treat pain with NSAIDs (ibuprofen, naproxen) or acetaminophen with codeine (severe pain). ■ Scrotal elevation and scrotal ice packs. Bed rest for few days. ■ Stool softeners (i.e., docusate sodium or Colace) if constipated. ■ Refer to ED if septic, severe intractable pain, abscessed, others.

HBsAg: positive o Anti-HBs: negative o Anti-HBc IgM: positive

Acute Hepatitis B:

Classic Case Adult to older male complains of sudden onset of high fever and chills with suprapubic and/or perineal pain/discomfort. Pain sometimes radiates to back or rectum. Acompanied by UTI symptoms such as dysuria, frequency, nocturia with cloudy urine. DRE reveals extremely tender prostate that is warm and boggy.

Acute Prostatitis Medications ■ Based on age and presumptive organism: - Age < 35, treat with ceftriaxone 250 mg IM and doxycycline 100 mg BID x 10 days. - Age > 35 or unlikely sexual transmission, treatment with ciprofl oxacin or ofl oxacin PO twice a day (BID) or levofl oxacin (Levaquin) PO daily for 4 to 6 weeks. ■ Others: Levofloxacin (Levaquin) PO daily, Bactrim PO BID. ■ Antipyretics, stool softener without laxative (Colace), sitz baths, hydration. ■ Refer if septic or toxic.

This disorder symptoms can be wide and varied but often develop slowly, often over several months...diarrhea, N&V, loss of appetite (followed by weight loss), paleness or darkening of skin (may be patchy). May also have muscle weakness, fatigue, slow sluggish movement, fainting, hypoglycemia, low BP and craving for salt....I am a result of inadequate hormone production. What gland and name of disorder

Adrenal glands; Addison's disease

CLASSIC CASE: Elderly smoker complains of gradual or sudden and painless central vision loss During the early phase of visual loss, they may not notice central vision loss until a large area is involved. Reports that straight lines appear distorted. Peripheral vision is usually preserved.

Age-related macular degeneration Referral to ophthalmologist and give Amsler Grid so they can check their vision daily to weekly. Ophthalmologist may authorize ocular vitamins.

which of the following best describes the physical activity recommendations such as brisk walking for a 55 y/o woman with newly dx type 2 diabetes (more than one can apply) a. goal should be for a total increased physical activity of 150 min/week or more b. increased physical activity for at least 30 min per day, at least 3x per week with no more than 48 hrs w/o exercise c. some form of resistance exercise such as lifting such as dumbbells or suing an exercise band should be included at least three times per week d. vigorous aerobic or resistance activity is potentially CI in the presence of proliferative or severe non- proliferative retinopathy due to the possible risk of vitreous hemorrhage or retinal detachment

All

CLASSIC CASE: Complains of chronic nasal congestion with clear mucus discharge or postnasal drip. Accompanied by nasal itch and, at times, frequent sneezing. Coughing worsens when supine due to postnasal drip. May be allergic to dust mites, mold, pollen, others.

Allergic rhinitis Complications: acute sinusitis, AOM, rhinitis medicamentosa. -prolonged use of topical nasal decongestants >3days. Tx: nasal steroid (Flonase), Decongestants (pseduophedrine) Zyrtec 10mg daily, avoid triggers

RhoGAM

Also known as anti-D immune globulin. It is made from pooled IgG antibodies against Rh (rhesus) factor. It is an immunoglobulin that helps prevent maternal isoimmunization (self-immunization) or alloimmunization (immunity against another individual of the same species). If RhoGAM is not given to Rh-negative pregnant women, it will result in fetal hemolysis and fetal anemia in her future pregnancies.

Treatment options for actinic keratoses include topical: A. vitamin D derivative cream B. 5-fluorouracil C. acyclovir D. doxepin

B. 5-fluorouracil

A 64-year-old man with seborrhea mentions that his skin condition is "better in the summer when I get outside more and much worse in the winter." You respond: A. Sun exposure is a recommended therapy for the treatment of this condition. B. Although sun exposure is noted to improve the skin lesions associated with seborrhea, its use as a therapy is potentially associated w/an increased rate of skin cancer. C. The lower humidity in the summer months noted in many areas of North America contributes to the improvement in seborrheic lesions. D. Use high-potency topical corticosteroids during the winter months, tapering these off for the summer months

B. Although sun exposure is noted to improve the skin lesions associated with seborrhea, its use as a therapy is potentially associated with an increased rate of skin cancer

The most common causative organism of lymphogranuloma venereum is: A. Ureaplasma genitalium. B. C. trachomatis types 1 to 3. C. Neisseria gonorrhoeae. D. H. ducreyi.

B. C. trachomatis types 1 to 3.

In men with reactive arthritis and associated urethritis, a common finding is: A. ANA positive. B. HLA-B27 positive. C. RF positive. D. ACPA positive.

B. HLA-B27 positive.

Which of the following statements is false regarding sunlight exposure and vitamin D production? A. In the continental United States, summertime exposure to sunlight can produce the majority of the body's requirement for vitamin D. B. One glass of fortified milk has an equivalent amount of vitamin D as what is produced after 10 minutes of exposure to summer sunlight in a healthy young individual. C. Use of sunscreen can block the majority of solarinduced vitamin D production. D. A person with a darker skin tone produces less vitamin D with sun exposure compared with a person with a lighter skin tone.

B. One glass of fortified milk has an equivalent amount of vitamin D as what is produced after 10 minutes of exposure to summer sunlight in a healthy young individual.

Who is the best candidate for isotretinoin (Accutane) therapy? A. a 17-year-old patient with pustular lesions and poor response to benzoyl peroxide B. a 20-year-old patient with cystic lesions who has tried various therapies with minimal effect C. a 14-year-old patient with open and closed comedones and a family history of "ice pick" scars D. an 18-year-old patient with inflammatory lesions and improvement with tretinoin (Retin-A)

B. a 20-year-old patient with cystic lesions who has tried various therapies with minimal effect

Which of the following patients would be an appropriate candidate for treatment with teriparatide (Forteo)? A. a 54-year-old woman with osteopenia B. a 64-year-old woman with BMD T-score of -2.5 and prior hip fracture C. a 67-year-old man with a BMD T-score of -1 D. a 72-year-old woman who has a stable BMD T-score of -1.5 with bisphosphonate treatment for the past 3 years

B. a 64-year-old woman with BMD T-score of -2.5 and prior hip fracture

A 72-year-old woman presents with a newly formed,painless, pearly, ulcerated nodule with an overlying telangiectasis on the upper lip. This most likely represents: A. an actinic keratosis B. a basal cell carcinoma C. a squamous cell carcinoma D. molluscum contagiosum

B. a basal cell carcinoma "PUTON" sunscreen P=early papule U=ulcerating T=elangiectasia O=n the face and scalp N=nodules, slow growing

Preferred antimicrobials for the treatment of adultswith Lyme disease include all of the following except: A. a tetracycline B. an aminoglycoside C. a cephalosporin D. a penicillin

B. an aminoglycoside

To enhance the potency of a topical corticosteroid, the prescriber recommends that the patient apply the preparation: A. to dry skin by gentle rubbing B. and cover with an occlusive dressing C. before bathing D. with an emollient

B. and cover with an occlusive dressing

In counseling a patient with atopic dermatitis, you suggest all of the following can be used to alleviate symptoms of a flare except: A. the use of oral antihistamines B. applying a heating pad on the affected region for 30 minutes C. the use of topical corticosteroids D. applying cool, wet dressings made from a clean cloth and water to the affected area

B. applying a heating pad on the affected region for 30 minutes

Which of the following findings is often found in a person with stage 2 Lyme disease? A. peripheral neuropathic symptoms B. atrioventricular heart block C. conductive hearing loss D. macrocytic anemia

B. atrioventricular heart block

Symptoms commonly associated with reactive arthritis include all of the following except: A. dactylitis. B. bursitis. C. enthesitis. D. cervicitis.

B. bursitis.

When advising the patient about scabies contagion, you inform her that: A. mites can live for many weeks away from the host B. close personal contact with an infected person is usually needed to contract this disease C. casual contact with an infected person is likely to result in infestation D. bedding used by an infected person must be destroyed

B. close personal contact with an infected person is usually needed to contract this disease

The clinical presentation of acute gouty arthritis affecting the base of the great toe includes: A. slow onset of discomfort over many days. B. greatest swelling and pain along the median aspect of the joint. C. improvement of symptoms with joint rest. D. fever.

B. greatest swelling and pain along the median aspect of the joint.

First-line treatment options for bursitis usually include: A. corticosteroid bursal injection. B. heat to area. C. weight-bearing exercises. D. (NSAIDs)

B. heat to area.

Risk factors for prostate cancer include all of the following except: A. African ancestry. B. history of genital trauma. C. family history of prostate cancer. D. high-fat diet.

B. history of genital trauma.

First-line treatment of SLE in a patient with mild symptoms is: A. systemic corticosteroids. B. hydroxychloroquine plus NSAIDs. C. anakinra. D. methotrexate.

B. hydroxychloroquine plus NSAIDs.

You are examining an 18-year-old man who is seeking a sports clearance physical examination. You note a mid-systolic murmur that gets louder when he stands. This most likely represents: A. aortic stenosis. B. hypertrophic cardiomyopathy. C. a physiologic murmur. D. a Still murmur.

B. hypertrophic cardiomyopathy.

The most common cause of acute bursitis is: A. inactivity. B. joint overuse. C. fibromyalgia. D. bacterial infection.

B. joint overuse.

Which of the following is inconsistent with the clinical presentation of fibromyalgia? A. widespread body aches B. joint swelling C. fatigue D. cognitive changes

B. joint swelling

Physical examination findings in lymphogranuloma venereum include: A. verruciform lesions. B. lesions that fuse and create multiple draining sinuses. C. a painless crater. D. plaquelike lesions.

B. lesions that fuse and create multiple draining sinuses.

Women with PID typically present with all of the following except: A. dysuria. B. leukopenia. C. cervical motion tenderness. D. abdominal pain.

B. leukopenia.

Oral antimicrobial treatments recommended for rosacea include all of the following except: A. metronidazole B. levofloxacin C. erythromycin D. doxycycline

B. levofloxacin

A 14-year-old male presents with acne consisting of 25 comedones and 20 inflammatory lesions with no nodules. This patient can be classified as having: A. mild acne. B. moderate acne. C. severe acne. D. very severe acne.

B. moderate acne.

You hear a fixed split second heart sound (S 2 ) in a 28-year-old woman who wants to start an exercise program and consider that it is: A. a normal finding in a younger adult. B. occasionally found in uncorrected atrial septal defect. C. the result of valvular sclerosis. D. often found in patients with right bundle branch block.

B. occasionally found in uncorrected atrial septal defect.

Finasteride (Proscar, Propecia) and dutasteride (Avodart) are helpful in the treatment of BPH because of their effect on: A. bladder contractility. B. prostate size. C. activity at select bladder receptor sites D. bladder pressure

B. prostate size.

Which of the following is the preferred medication to reverse the anticoagulant effects of unfractionated heparin? A. vitamin K B. protaimine sulfate C. platelet transfusion D. plasma components

B. protaimine sulfate

Psoriatic lesions arise from: A. decreased skin exfoliation. B. rapid skin cell turnover, leading to decreased maturation and keratinization. C. inflammatory changes in the dermis. D. lichenification

B. rapid skin cell turnover, leading to decreased maturation and keratinization

In ordering imaging studies in a patient with peripheral vascular disease, the use of radiocontrast medium can potentially result in: A. hepatic failure B. renal failure C. bone marrow suppression D. thrombocytopenia

B. renal failure

A patient with a lumbosacral strain will typically report: A--numbness in the extremities. B--stiffness, spasm, and reduced ROM. C--"electric" sensation running down one or both legs. D--pain at its worst when in sitting position.

B. stiffness, spasm, and reduced ROM.

A 53-year-old woman who is taking hormone therapy (HT) with conjugated equine estrogen, 0.45 mg/d, with MPA, 1.5 mg, has bothersome atrophic vaginitis symptoms. You advise that: A. her oral estrogen dose should be increased. B. the addition of a topical estrogen can be helpful. C. the MPA component should be discontinued. D. baking soda douche should be tried.

B. the addition of a topical estrogen can be helpful.

In providing care for a patient with superficial thrombophlebitis, the NP considers that: A. it is a benign, self- limiting disease B. the linear pattern of induration can help differentiate the process from cellulitis or other inflammatory processes. C. a chest radiograph should be obtained. D. limited activity enhances recovery.

B. the linear pattern of induration can help differentiate the process from cellulitis or other inflammatory processes.

You see a 42-year-old woman with a cluster of red,itchy spots on her left arm. She informs you that she recently stayed at a hotel that she later discovered was infested with bed bugs. You advise her that: A. she should immediately begin a regimen of oral antibiotics. B. the reaction is usually self-limiting and should resolve in 1 to 2 weeks. C. given that bed bug bites are usually not itchy, an alternative diagnosis should be considered. D. she should wash all of her clothes in cold water

B. the reaction is usually self-limiting and should resolve in 1 to 2 weeks

For severe, recalcitrant psoriasis that affects more than 30% of the body, all of the following treatments are recommended except: A. methotrexate B. topical anthralin (Drithocreme) C. tumor necrosis factor (TNF) modulators D. cyclosporine

B. topical anthralin (Drithocreme)

When prescribing tretinoin (Retin-A), the NP advises the patient to: A. use it with benzoyl peroxide to minimize irritating effects. B. use a sunscreen because the drug is photosensitizing. C. add a sulfa-based cream to enhance anti acne effects. D. expect a significant improvement in acne lesions after approximately 1 week of use

B. use a sunscreen because the drug is photosensitizing.

Clinical features of bullous impetigo include: A. intense itch. B. vesicular lesions. C. dermatomal pattern. D. systemic symptoms such as fever and chills.

B. vesicular lesions.

Metformin use increased risk of vitamin ____ deficiency due to malabsorption risk is dose and length of therapy related

B12

Older male complains of gradual development (years) of urinary obstructive symptoms such as weak urinary stream, postvoid dribbling, feelings of incomplete emptying, and occasional urinary retention. Nocturia very common.

BPH ■ PSA is elevated (norm ranges from 0 to 4 ng/mL). ■ Enlarged prostate that is symmetrical in texture and size (rubbery texture). Medications ■ Alpha-adrenergic antagonist: Terazosin (Hytrin) 5 mg or tamsulosin (Flomax). ■ 5-Alpha-reductase inhibitors (blocks testerone): Finasteride (Proscar). ■ Duration of treatment ranges from a few months to daily for many years. ■ Avoid drugs that worsen symptoms: Cold medications (antihistamines, decongestants), caffeine. ■ Herbal: Saw palmetto (mild improvement for some). Does not work for everyone. ■ Classify severity of BPH using the American Urological Association questionnaire.

A left shift is seen in the presence of severe _________ infection like appendicitis What is seen in a left shift?

Bacterial Neutrophilia, leukocytosis, and bandemia

which of the following pt complaints should be evaluated further in making a differential of irritable bowel syndrome a. a 52 y/o female with a first degree family hx of colorectal cancer, recent constipation, and abd pain b. a middle aged adult with low albumin and leukocytosis c. both patients outlined in responses A and B d. a 16 y/o female with chronic, alternating constipation and diarrhea when she is studying for high school exams and worrying about her parents' impending divorce?

Both a. a 52 y/o female with a first degree family hx of colorectal cancer, recent constipation, and abdominal pain b. a middle aged adult with low albumin and leukocytosis

Few anticholinergic effects Energizing Possible increased libido, agitation (25%) Avoid with significant manifestation of anxiety, agitation, insomnia Blocks reuptake of dopamine at presynaptic neuron, especially in high doses, some increase in norepinephrine transmission. Dopamine receptor sites likely stimulated in substance abuse, making _______a helpful antidepressant for a person with a history of substance abuse. Nonaddicting and nonintoxicating Avoid use in presence of eating disorder or if anorexia is a major component of depression. Weight loss often seen (28% >5 lb [2.3 kg]) after initiation of therapy Do not give if history of or risk for seizure, closed head injury history, history of quiescent epilepsy Seizure risk worsens if dose increased rapidly

Bupropion (Wellbutrin) [selective dopamine reuptake inhibitor]

A 45-year-old woman just had a normal Pap test result and has an absence of high-risk HPV. You recommend her next Pap test in: A. 1 year. B. 3 years. C. 5 years. D. 7 years.

C. 5 years.

For adults 70 years and younger, what is the recommended daily intake of vitamin D? A. 200 IU B. 400 IU C. 600 IU D. 1000 IU

C. 600 IU

In reference to reactive arthritis (also known as Reiter syndrome), which of the following statements is false? A. When the disease is associated with urethritis, the male:female ratio is about 9:1. B. When the disease is associated with infectious diarrhea, the male and female incidences are approximately equal. C. ANA analysis reveals a speckled pattern. D. Results of joint aspirate culture are usually unremarkable.

C. ANA analysis reveals a speckled pattern.

Which of the following statements is true concerning vaginal diaphragm use? A. When in place, the woman is aware that the diaphragm fits snugly against the vaginal walls. B. This is a suitable form of contraception for women with recurrent urinary tract infection. C. After insertion, the cervix should be smoothly covered. D. The device should be removed within 2 hours of coitus to minimize the risk of infection.

C. After insertion, the cervix should be smoothly covered.

Which of the following represents the most accurate patient information when using permethrin (Elimite) for treating scabies? A. To avoid systemic absorption, the medication should be applied over the body and rinsed off within 1 hour. B. The patient should notice a marked reduction in pruritus within 48 hours of using the product. C. Itch often persists for a few weeks after successful treatment. D. It is a second-line product in the treatment of scabies

C. Itch often persists for a few weeks after successful treatment. -debris is eliminated from the body in a few weeks. the medication should be applied and left on for 8-14 hours to be effective.

52-year-old woman has RA. She now presents with decreased tearing, "gritty"-feeling eyes, and a dry mouth. You consider a diagnosis of: A. systemic lupus erythematosus. B. vasculitis. C. Sjögren syndrome. D. scleroderma.

C. Sjögren syndrome.

Which of the following statements is most accurate in the assessment of a patient with varicose veins? A. The degree of venous tortuosity is well-correlated with the amount of leg pain reported. B. As the number of affected veins increases, so does the degree of patient discomfort. C. Symptoms are sometimes reported with minimally affected vessels. D. Lower-extremity edema is usually seen only with severe disease.

C. Symptoms are sometimes reported with minimally affected vessels.

Patient education about the use of sildenafil (Viagra) includes the following: A. A spontaneous erection occurs about 1 hour after taking the medication. B. This medication helps regain erectile function in nearly all men who use it. C. With the use of the medication, sexual stimulation also is needed to achieve an erection. D. Nitrates can be safely used concurrently.

C. With the use of the medication, sexual stimulation also is needed to achieve an erection.

Postmenopausal HT used for use can result in: A. a reduction in the rate of cardiovascular disease. B. an increase in the rate of rheumatoid arthritis. C. a reduction in the frequency& severity of vasomotor symptoms. D. a disturbance in sleep patterns.

C. a reduction in the frequency& severity of vasomotor symptoms.

The most helpful diagnostic test to perform during acute gouty arthritis is: A. measurement of erythrocyte sedimentation rate (ESR). B. measurement of serum uric acid. C. analysis of aspirate from the affected joint. D. joint radiography

C. analysis of aspirate from the affected joint.

Comprehensive treatment for a person with peripheral occlusive arterial disease and diabetes mellitus includes all of the following iexcept: A. daily aspirin use B. lipid lowering with an HMG-CoA reductase inhibitor (statin) C. application of a topical antimicrobial to the affected area. D. maintenance of glycemic control

C. application of a topical antimicrobial to the affected area.

The most important aspect of skin care for individuals with atopic dermatitis is: A. frequent bathing with antibacterial soap. B. consistent use of medium-potency to high-potency topical steroids. C. application of lubricants. D. treatment of dermatophytes.

C. application of lubricants. -minimize skin dryness by limiting soap and water exposure

A physiological murmur has which of the following characteristics? A. occurs late in systole B. is noted in a localized area of auscultation C. becomes softer when the patient moves from supine to standing D. frequently obliterates S2

C. becomes softer when the patient moves from supine to standing

Definitive diagnosis of skin cancer requires: A. skin examination B. CT scan C. biopsy D. serum antigen testing

C. biopsy

Which of the following diagnostic approaches is used for confirmation of sarcoidosis? A. chest x-ray B. high-resolution CT scan C. biopsy D. ANA fluorescent staining

C. biopsy

Antihistamines exhibit therapeutic effect by: A. inactivating circulating histamine. B. preventing the production of histamine. C. blocking activity at histamine receptor sites. D. acting as a procholinergic agent.

C. blocking activity at histamine receptor sites

Which of the following dietary supplements is associated with increased risk for gout? A. vitamin A B. gingko biloba C. brewer's yeast D. glucosamine

C. brewer's yeast

A 62-year-old woman presents 2 days after noticing a"bug bite" on her left forearm. Examination reveals a warm, red, edematous area with sharply demarcated borders. The patient is otherwise healthy with no fever. This most likely represents: A. contact dermatitis. B. an allergic reaction. C. cellulitis. D. erysipelas

C. cellulitis

A 72-year-old man presents at an early stage of osteoarthritis in his left knee. He mentions that he heard about the benefits of using glucosamine and chondroitin for treating joint problems. In consulting the patient, you mention all of the following except: A. any benefit can take at least 3 months of consistent use before observed. B. glucosamine is not associated with any drug interactions. C. clinical studies have consistently shown benefit of long- term use of glucosamine and chondroitin for treating OA of the knee. D. chondroitin should be used with caution because of its anti-platelet effect.

C. clinical studies have consistently shown benefit of long-term use of glucosamine and chondroitin for treating OA of the knee.

A 24-year-old man arrives at the walk-in center. He reports that he was bitten in the thigh by a raccoon while walking in the woods. The examination reveals a wound that is 1 cm deep on his right thigh. The wound is oozing bright red blood. Your next best action is to: A. administer high-dose parenteral penicillin B. initiate antibacterial prophylaxis with amoxicillin C. give rabies immune globulin and rabies vaccine D. suture the wound after proper cleansing

C. give rabies immune globulin and rabies vaccine

Which of the following is the least potent topical corticosteroid? A. betamethasone dipropionate 0.1% (Diprosone) B. clobetasol propionate 0.05% (Cormax) C. hydrocortisone 2.5% D. fluocinonide 0.05% (Lidex)

C. hydrocortisone 2.5%

In the treatment of acne vulgaris, which lesions respond best to topical antibiotic therapy? A. open comedones B. cysts C. inflammatory and pustular lesions D. superficial lesions

C. inflammatory and pustular lesions

Nonchemical means to eliminate bed bugs can include all of the following except: A. vacuuming crevices. B. washing bedding and other items in hot water. C. isolating the infested area from any hosts for at least 2 weeks. D. running bedding and other items in a dryer on high heat for 20 minutes

C. isolating the infested area from any hosts for at least 2 weeks. They can live for weeks w/o feeding. Can place in freezer that is set for 0 degrees for at least 4 days

In assessing a man with testicular torsion, the NP is most likely to note: A. elevated PSA level. B. white blood cells reported in urinalysis. C. left testicle most often affected. D. increased testicular blood flow by color-flow Doppler ultrasound.

C. left testicle most often affected.

Patients with subscapular bursitis typically present with: A. limited shoulder ROM. B. heat over affected area. C. localized tenderness under the superomedial angle of the scapula. D. cervical nerve root irritation.

C. localized tenderness under the superomedial angle of the scapula.

The use of sunscreen has minimal impact on reducing the risk of which type of skin cancer? A. squamous cell carcinoma B. basal cell carcinoma C. malignant melanoma D. all forms of skin cancer

C. malignant melanoma sunscreen is protective to SSC and BCS

Oral antifungal treatment options for onychomycosis include all of the following except: A. itraconazole B. fluconazole C. metronidazole D. terbinafine

C. metronidazole

In diagnosing onychomycosis, the NP considers that: A. nails often have a single midline groove. B. pitting is often seen. C. microscopic examination reveals hyphae. D. Beau lines are present

C. microscopic examination reveals hyphae

A 28-year-old woman presents to your practice with chief complaint of a cat bite sustained on her right ankle. Her pet cat had bitten her after she inadvertently stepped on its paw while she was in her home. Her cat is 3 years old, is up-to-date on immunizations, and does not go outside. Physical examination reveals pinpoint superficial puncture wounds on the right ankle consistent with the presenting history. She washed the wound with soap and water immediately and asks if she needs additional therapy. Treatment for this patient's cat bite wound should include standard wound care with the addition of: A. oral erythromycin B. topical bacitracin C. oral amoxicillin-clavulanate D. parenteral rifampin

C. oral amoxicillin-clavulanate for 3-5

In patients with varicose veins, which essel is most often affected? A. femoral vein B. posterior tibial vein C. peroneal vein D. saphenous vein

C. peroneal vein

The human papillomavirus responsible for non genital warts is mainly passed through: A. contact with infected surfaces. B. exposure to saliva from infected person. C. person-to-person contact. D. exposure to infected blood

C. person-to-person contact

Which of the following medications is approved by the U.S. Food and Drug Administration (FDA) for pain management in a person with fibromyalgia? A. trazodone B. nortriptyline C. pregabalin D. gabapentin

C. pregabalin (Lyrica)

A patient presents with a painful, blistering thermal burn involving the first, second, and third digits of his right hand. The most appropriate plan of care is to: A. apply an anesthetic cream to the area and open the blisters B. apply silver sulfadiazine cream (Silvadene) to the area followed by a bulky dressing C. refer the patient to burn specialty care D. wrap the burn loosely with a nonadherent dressing and prescribe an analgesic agent

C. refer the patient to burn specialty care

Spider varicosities are: A. usually symptomatic B. a potential site for thrombophlebitis C. responsive to laser obliteration D. caused by sun exposure.

C. responsive to laser obliteration

Syphilis is most contagious during which of the following? A. before onset of signs and symptoms B. primary stage C. secondary stage D. tertiary stage

C. secondary stage

In counseling a patient with scabies, the NP recommends all of the following methods to eliminate themite from bedclothes and other items except: A. wash items in hot water. B. run items through the clothes dryer for a normal cycle. C. soak items in cold water for at least 1 hour. D. place items in a plastic storage bag for at least 1 week

C. soak items in cold water for at least 1 hour The pt can either wash the items in hot water, run through dryer for a cycle of hot heat or place items in a plastic storage bag for at least 1 week (mites can't live more than 3-4 without host)

You see a 45-year-old woman who is considering HT. She has a family history of cervical dysplasia, hyperlipidemia, and VTE. You advise her on all of the following except: A. the use of progestin can minimize the risk of endometrial cancer for a woman on HT and who has not had a hysterectomy. B. supplemental estrogen should be avoided in women who are at high risk of breast cancer or uterine cancer. C. supplemental estrogen should be avoided in women who are at high risk of cardiovascular disease. D. short-term studies demonstrate that oral HT is associated with lower thromboembolic risk than transdermal forms of HT.

C. supplemental estrogen should be avoided in women who are at high risk of cardiovascular disease.

In counseling a patient on the use of topical products to treat nail fungal infections, the NP considers that: A. nail lacquers, such as ciclopirox olamine 8% solution (Penlac), offer similar effectiveness to oral antifungals. B. some herbal products, such as tea tree oil, can be an effective alternative to oral agents. C. topical products have limited penetration through the nail matrix to reach the site of infection. D. cream-based products are more effective than gel- based products in treating nail fungal infections

C. topical products have limited penetration through the nail matrix to reach the site of infection nail lacquers, such as ciclopirox olamine 8% solution (Penlac), effectiveness is limited and typically poorer than oral anti fungal agents. some herbal products, such as tea tree oil are usually not effective bc the nails are too thick and hard for the external applications to penetrate.

• Beta-lactam antibiotics - Penicillins - Cephalosporins • Macrolides - Azithromycin, erythromycin, but not clarithromycin • Acetaminophen • Nitrofurantoin (Macrodantin®, Macrobid®) - Avoid at term due to potential risk of hemolysis • Select inhaled corticosteroids - Budesonide (Pulmicort®) but not fluticasone (Flovent®) what category

Category B B= Best Because Nothing in A • Animal studies have not demonstrated fetal risk but no controlled study in humans. - Or- • Animal studies have shown adverse effect but not demonstrated in human study

ACEI (-pril suffix), ARB (-sartan) - Risk of fetal hypotension, IUFD, renal atrophy • The tetracyclines - Doxycycline=D • Carbamazepine - Tegretol® • Valproic acid - Depakote® • Lithium - 1:400-1:4000 risk of Ebstein anomaly with tricuspid valve and other cardiac problems Paroxetine=Category D - 50-100% increase in cardiac defects • From 1% to 2% in one study • From 1% to 1.5% in another study - Most commonly observed=ASD, VSD category?

Category D ,D= Danger • Positive evidence of human fetal risk • Benefit from use in pregnant woman might be acceptable despite the risk - Life-threatening disease

CLASSIC CASE: Complains of a gradual onset of a small superficial nodules that is discrete and moveable on the upper eyelid that feels like a bead. Can slowly enlarge over time. Benign. If nodule enlarges or does not resolve in a few weeks, biopsy to r/o squamous cell cancer. If large and affects vision, surgical removal is an option.

Chalazion

For uncomplicated __________infection, a test-of-cure following completion of the antimicrobial course is not needed unless the patient has persistent symptoms or is pregnant. In pregnancy, testing for cure should be performed 3 weeks after completion of treatment.

Chlamydia

Doxycycline is Treatment for which of the following STI? Choose from the list below? A. Genital Warts b. Gonoccocal Urethritis c. Chlamydia Trachomatis cervicitis D. Syphilis

Chlamydia trachomatis cervivitis Syphillis (Penicillin allergy only)

A 30-year-old woman presents without symptoms but states that her male partner has dysuria without penile discharge. Examination reveals a friable cervix covered with thick yellow discharge. This description is most consistent with an infection caused by

Chlamydia trachomatis.

If perforation and peritonitis are suspected due to appendicitis, what antibiotics should be given?

Choices include: ampicillin, gentamicin, clindamycin, flagy, unasyn timentin

Elderly male with history of several weeks of suprapubic or perineal discomfort that is accompanied by irritative voiding symptoms such as dysuria, nocturia, and frequency. Not accompanied by systemic symptoms. Some men are asymptomatic. ■ Prostate may feel normal or slightly "boggy" to palpation. Not tender. ■ UA: Normal (unless patient has cystitis). ■ Urine mixed with prostatic fluid: Positive for E. coli.

Chronic Bacterial Prostatitis Txt: ■ Trimethoprim-sulfamethoxazole (Bactrim) PO BID for 4 to 6 weeks (if sensitive). ■ Some prefer ofl oxacin (Floxin) BID or levofl oxacin (Levaquin) daily for 4 to 6 weeks.

Anti-HCV, HCV RNA

Chronic Hep C:

cilostazol (Pletal) and pentoxifylline (Trental) is used in what disorder

Chronic venous insufficiency cilostazol impaires platelet aggregation and increases vasodilation (NOT TO BE USED in/ HF) pentoxifylline thins bloods and improves flow by altering RBCs ability to pass through disease vessels (helpful in exercise tolerance) Plavix is also used, mainly in ppl who are allergic to ASA or do not tolerate it or have underlying hyper coagulable state -works by preventing fibrinogen binding

Recommended non pharmacological options to treat actinic keratosis include all of the following except: A. chemical peel B. cryotherapy C. laser resurfacing D. Mohs micrographic surgery

D. Mohs micrographic surgery

Which of the following does not directly contribute to the development of varicose veins? A. leg crossing B. pregnancy C. heredity D. Raynaud disease

D. Raynaud disease

Among the following, who is at greatest risk of developing seborrheic dermatitis? A. a 15-year-old boy residing in a rural setting B. a 34-year-old woman who smokes 2 packs per day(PPD) C. a 48-year-old male truck driver D. a 72-year-old man with Parkinson disease

D. a 72-year-old man with Parkinson disease Seborrhea is often found in pts who are chronically ill or immunocompromised (i.e) elder adults, parkinson's disease

When examining a mole for malignant melanoma, allof the following characteristics can indicate a melanoma except: A. asymmetry with nonmatching sides. B. color is not uniform. C. a recently formed lesion. D. a lesion that has been present for at least 2 years

D. a lesion that has been present for at least 2 years ABCD "E" evolving lesions- either new or changing, most melanoma manifest as a new lesion

Surgical intervention in BPH should be considered with all of the following except: A. recurrent urinary tract infection. B. bladder stones. C. persistent obstruction despite medical therapy. D. acute tubular necrosis.

D. acute tubular necrosis.

Drug therapy that had previously thought to worsen symptoms in lower-extremity arterial vascular disease includes the use of: A. beta2-agonists B. calcium channel antagonists C. direct thrombin inhibitors D. beta-adrenergic antagonists

D. beta-adrenergic antagonists propanolol, metoprolol Beta2-agonist are for lungs bronchodilators

Pseudogout is caused by the formation of what type of crystals in joints? A. uric acid B. calcium oxalate C. struvite D. calcium pyrophosphate dihydrate

D. calcium pyrophosphate dihydrate

For a woman with bothersome hot flashes who cannot take HT, alternative options with demonstrated efficacy and limited adverse effects include the use of all of the following except: A. venlafaxine. B. sertraline. C. gabapentin. D. clonidine.

D. clonidine.

A skin biopsy results indicate the presence of malignant melanoma for a 53-year-old woman. You recommend: A. excision of the entire lesion B. electrodissection with curettage C. initiating treatment with topical cancer chemotherapy D. consultation with a skin cancer expert to direct next best action

D. consultation with a skin cancer expert to direct next best action

Patients with medial epicondylitis typically present with: A. forearm numbness. B. reduction in ROM. C. pain on elbow flexion. D. decreased grip strength.

D. decreased grip strength.

Which of the following joints is most likely to be affected by osteoarthritis (OA)? A. wrists B. elbows C. metacarpophalangeal joint D. distal interphalangeal joint

D. distal interphalangeal joint DIPs and PIPs=OA MCPs= RA

The initial diagnostic evaluation of a clinically stable patient with suspected DVT most often includes obtaining a/an: A. impedance plethysmography B. iodine 125 fibrinogen scan C. contrast venography D. duplex ultrasonography

D. duplex ultrasonography

Analgesia options for a patient with shingles can include all of the following except: A. topical lidocaine gel 5% with oral acetaminophen B. Burow's solution with a high-potency oral NSAID C. Burow's solution with an oral opioid D. fentanyl transdermal patch and a topical medium-potency corticosteroid on the affected area

D. fentanyl transdermal patch and a topical medium-potency corticosteroid on the affected area

Which of the following is not a contributing factor to development of venous thrombophlebitis? A. venous status B. injury to vascular intima C. malignancy-associated hypercoagulation states D. isometric exercise

D. isometric exercise

Initial preparticipation screening of athletes should include: A. a resting 12-lead ECG. B. echocardiogram. C. both a resting 12-lead ECG and echocardiogram. D. neither a resting 12-lead ECG nor echocardiogram.

D. neither a resting 12-lead ECG nor echocardiogram.

When taking a PDE-5 inhibitor, concomitant use of which medication must be avoided? A. statins B. sulfonylurea C. ACE inhibitors D. nitrates

D. nitrates include sildenafil. They inhibit cGMP PDE5 and prolong vasodilatory effect of nitric oxide. Also used to treat erectile dysfuntion.

Risk factors for angular cheilitis in adults include allof the following except: A. advanced age B. HIV infection C. alteration of facial vertical dimension due to loss of teeth D. obesity

D. obesity

You prescribe a topical medication and want it to have maximum absorption, so you choose the following vehicle: A. gel B. lotion C. cream D. ointment

D. ointment

You see a 52-year-old woman who was bitten by a rat while opening a Dumpster. The examination reveals a wound approximately 1 cm deep that is oozing bright red blood. Treatment of this patient should include standard wound care with the addition of: A. rabies immune globulin B. rabies vaccine C. oral ciprofloxacin D. oral amoxicillin-clavulanate

D. oral amoxicillin-clavulanate

Nonsurgical options for the treatment of squamous and basal cell carcinoma include all of the following except: A. cryotherapy B. electrodissection with curettage C. topical cancer chemotherapy D. oral hydroxyurea

D. oral hydroxyurea

For the woman with a history of DVT who is having significant vasomotor symptoms, which of the following can be can be used for symptom management? A. 17-17β-estradiol patch B. drospirenone C. estrone. D. paroxetine.

D. paroxetine.

You are examining a 19-year-old man who is diagnosed with a murmur of mitral regurgitation. When he asks about participation in sports activities, you counsel that: A. participation in sports activities should not be affected by his condition. B. he should refrain from any activities requiring physical exertion. C. participation will depend on the degree of atrial atrophy. D. participation will depend on the degree of ventricular enlargement.

D. participation will depend on the degree of ventricular enlargement.

You are examining a 19-year-old man who is diagnosed with a murmur of mitral regurgitation. When he asks about participation in sports activities, you counsel that: A. participation in sports activities should not be affected by his condition. B. he should refrain from any activities requiring physical exertion. C. participation will depend on the degree of atrial atrophy. D. participation will depend on degree of ventricular enlargement.

D. participation will depend on the degree of ventricular enlargement.

The anticipated result of debridement as part of the treatment of venous stasis ulcers includes all of the following except: A. enhanced tissue granulation B. encouragement of reepithelialization C. reduction of bacterial burden D. prevention of peripheral arterial disease.

D. prevention of peripheral arterial disease.

The use of TNF modulators for the treatment of psoriasis is associated with an increased risk for: A. gastrointestinal disorders B. nephrotoxicity C. QTc prolongation D. reactivation of latent tuberculosis

D. reactivation of latent tuberculosis

A 24-year-old man presents with sudden onset of left-sided scrotal pain. He reports having intermittent unilateral testicular pain in the past but not as severe this current episode. Confirmation of testicular torsion would include all of the following findings except: A. unilateral loss of the cremasteric reflex. B. the affected testicle held higher in the scrotum. C. testicular swelling. D. relief of pain with scrotal elevation.

D. relief of pain with scrotal elevation.

You see a 45-year-old woman who is considering HT. She has a family history of cervical dysplasia, hyperlipidemia, and VTE. You advise her on all of the following except: A. the use of progestin can minimize the risk of endometrial cancer for a woman on HT and who has not had a hysterectomy. B. supplemental estrogen should be avoided in women who are at high risk of breast cancer or uterine cancer. C. supplemental estrogen should be avoided in women who are at high risk of cardiovascular disease. D. short-term studies demonstrate that oral HT is associated with lower thromboembolic risk than transdermal forms of HT.

D. short-term studies demonstrate that oral HT is associated with lower thromboembolic risk than transdermal forms of HT.

Which of the following findings is often found in a person with stage 1 Lyme disease? A. peripheral neuropathic symptoms B. high-grade atrioventricular heart block C. Bell's palsy D. single painless annular lesion

D. single painless annular lesion

Clinical presentation of advanced lower-extremity vascular disease includes all of the following except: A. resting pain B. absent posterior tibialis pulse C. blanching of the foot with elevation D. spider varicosities

D. spider varicosities

Treatment options in generalized psoriasis vulgaris include all of the following except: A. psoralen with ultraviolet A light (PUVA) therapy. B. methotrexate. C. cyclosporine. D. systemic corticosteroids

D. systemic corticosteroids -corticosteroids are not used systemically, they are used topically. -additionally, when psoriasis is generalized; covering more than 30% of the BSA, topicals are too expensive and difficult....consider UV A light exposure.

All of the following statements are true regarding skinreactions to bed bugs except: A. skin reactions are more common with repeated exposure to bed bug bites. B. skin reactions can typically involve papules, macules, or wheals. C. allergic reactions can be treated w/ topical corticosteroids. D. systemic skin reactions frequently occur following an initial exposure to bed bug bites

D. systemic skin reactions frequently occur following an initial exposure to bed bug bites Can happen with repeated exposure but is rare

Which of the following is not a common risk factor for erectile dysfunction (ED)? A. diabetes mellitus B. hypertension C. cigarette smoking D. testosterone deficiency

D. testosterone deficiency

140. All of the following are common sites of fracture in patients with osteoporosis except: A. the proximal femur. B. the distal forearm. C. the vertebrae. D. the clavicle.

D. the clavicle.

The most common sites for squamous and basal cell carcinoma include: A. palms of hands and soles of feet B. pelvic and lumbar regions C. the abdomen D. the face and scalp

D. the face and scalp

All of the following characteristics about bed bugs are true except: A. they can be found in furniture, carpeting, and floorboards. B. their peak feeding time is at dawn. C. during feeding, they are attracted to body heat and carbon dioxide. D. they prefer to harbor unsanitary environments.

D. they prefer to harbor unsanitary environments.

A 22-year-old man presents with new onset of pain and swelling in his feet and ankles, conjunctivitis, oral lesions, and dysuria. To help confirm a diagnosis of reactive arthritis, the most important test to obtain is: A. ANA analysis. B. ESR measurement. C. rubella titer measurement. D. urethral cultures.

D. urethral cultures.

A 49-year-old man presents with a skin lesion suspicious for malignant melanoma. You describe the lesion as having: A. deep black-brown coloring throughout. B. sharp borders. C. a diameter of 3 mm or less. D. variable pigmentation

D. variable pigmentation

Classic Case Gradual onset (over years). Early-morning joint stiffness with inactivity. Shorter duration of joint stiffness (less than 15 minutes) Pain aggravated by overuse of joint. During exacerbations, involved joint may be swollen and tender to palpation. May be one sided (e.g., right hip only). Diagnosis: Key presentation: Goal of Treatment:

Degenerative Joint Disease (Osteoarthritis) Arthritis occurs when the cartilage covering the articular surface of joints becomes damaged. Large weight-bearing joints (hips and knees) and the hands are most commonly affected. Risk factors: Older age, overuse of joints, and positive family history. Dx: Hx and presentation Key presentation: ■ Heberden's nodes: Bony nodules on the distal interphalangeal joints (DIP). ■ Bouchard's nodes (Figure 14.6): Bony nodules on the proximal interphalangeal joints (PIP). Goal of Treatment ■ Pain relief ■ Preserve joint mobility & function ■ Minimize disability & protect joint

DIAPERS pneumonic for treatable causes of urinary incontinence

Delirium Infection (urinary) Atrophic urethritis and vaginitis Pharmaceuticals (diuretics etc) Psychologic disorders (depression) Excessive Urine output (HF, DM) Restricted Mobility Stool Impaction

<1% failure rate -Progesterone ONLY -Injection q3 months with 2 week grace period -Delayed return of fertility (up to 1 yr) -Possible reduction in bone density with long-term use -Menstrual irregularities common

Depo-Provera

Diffuse facial nerve involvement with paralysis of the facial muscles • Onset is over 1-2 days; progressive, max symptoms reached by 3 weeks • 85% signs of recovery in 3 weeks • Function returns within 3-4 months

Diagnosis Bell's Palsy

Inability to produce an erection fi rm enough to perform sexual intercourse. Vascular insufficiency, neuropathy (diabetics), medications (selective serotonin reuptake inhibitors, beta-blockers), smoking, alcohol, hypogonadism. May be psychic causation or mixture of both.

ED ■ Organic cause: inability to have an erection under any circumstance. Due to neurovascular or vascular damage. ■ Psychiatric cause: Spontaneously has early morning erections or can achieve a firm erection with masturbation. Medications ■ First line: Phosphodiesterase type 5 inhibitors drug class. ■ Take Viagra and Levitra on an empty stomach. Food and fats delays action. ■ Sildenafi l citrate (Viagra) 25/50/100 mg; take one dose 30 to 60 minutes before sex. Use only one dose every 24 hours. ■ Vardenafi l (Levitra); take one dose 30 to 60 minutes before sex. Duration 4 hours. ■ Taladafi l (Cialis) 5 to 20 mg; take 2 hours before sex. Duration up to 36 hours. ■ Adverse effects: Headache, facial fl ushing, dizziness, hypotension, nasal congestion, priapism. ■ Other forms of treatment: Intracavernous injections (alprostadil or Caverject). Contraindications ■ Concomitant nitrates. Caution with alpha-blockers, recent post-myocardial infarction, post-CVA, major surgery, or any condition where exertion is contraindicated.

Prevention for osteoporosis?

ERT--Estrogen replacement therapy Weight bearing exercise (30 min walking 3-5x/week), walking, jogging, dancing, climbing stairss, aerobics, strength training, Increase calcium intake

Gilbert's disease

Elevated Bilirubin but ALL other LFTs normal is: most common inherited cause of unconjugated hyperbillirubinemia

The findings from an assessment of a 70-year-old patient with swelling in his ankles include jugular venous pulsations 5 cm above the sternal angle when the head of his bed is elevate 45 degrees. The nurse knows that this finding indicates:

Elevated pressure related to heart failure.

-85% effective -Levonorgestrel used to prevent fertilization or the implantation of a fertilized egg within the uterus -May be purchased OTC at any age -Should be taken within 72 hours of unprotected sex -May also insert copper IUD within 5 days to achieve effect (~99% effective) -Major SE: N&V, changes in timing and/or flow of next cycle

Emergency Contraception (Plan B)

CLASSIC CASE: Complains of acute onset of nasal bleeding secondary to trauma. Bright red blood may drip through nasal passages and or the posterior pharynx. Professed bleeding can lead to vomiting of blood. Tilt head forward, responds to pressure at Kiesselbach plexus, can use Afrin, nasal packing. Anterior manageable, referral for posterior.

Epistaxis

(T/F) An abnormally elevated D-dimer test is highly sensitive and specific for the diagnosis of thromboembolic disease

F D-dimer is more sensitive than specific D-dimer can be falsely elevated in presence of: -high rheumatoid factors -MI -pregnancy -metastatic cancers -surgery/trauma *a low risk patient with normal d-dimer is unlikely to have DVT

(T/F) With the use of a direct thrombin inhibitor, ongoing INR monitoring is required.

F- direct thrombin inhibitors= [DTIs] dabigatran (Pradaxa)

True or False: Most acute-onset necrotic skin lesions reported in North America are caused by spider bites

False

True or False: Skin lesions infected by community-acquired MRSA (CA-MRSA) often occur spontaneously on intact skin.

False

T/F: With lumbosacral strains, A precipitating event is usually reported by most patients because lumbosacral strain is usually the result of acute injury.

False A precipitating event is reported by only a few patients because lumbosacral strain is usually the culmination of many events, including repeated use of improperly stretched muscles in patients with overall poor conditioning.

T/F? When choosing an antimicrobial agent for the treatment of chronic bacterial prostatitis, the NP considers that choosing an antibiotic with gram-negative coverage is NOT critical.

False, it is

T/F: A free T3 is usually indicated because it evaluates the metabolically active portion of triiodothyronine (T3) when compared to T4. T3 is about 4x more metabolically active.

False, usually NOT indicated because it evaluates the metabolically active portion of triiodothyronine (T3) when compared to T4. T3 is about 4x more metabolically active

Occurs in approx 50% females (+) BREAST TENDERNESS Cyst is mobile (may be soft or firm) (-) Nipple discharge Tx: Warm soaks TID, low Na+ diet, Vit E, hormonal tx, surgical intervention if needed

Fibrocystic Breast Disease

During an assessment of a healthy adult, where would the nurse expect to palpate the apical impulse?

Fifth left intercostal space at the midclavicular line

___________ burn -Characterized by erythema, hyperemia and pain. -Affected skin blanches with ease

First-degree

What DM Medication? [ACTION] 1.) Stimulates insulin production in response to increase in plasma glucose. 2.) Inhibits postprandial glucagon release. 3.) Slows gastric emptying, often leading to appetite suppression and weight loss [EFFECTIVENESS] -A1C reduction of 1-1.5% w/ intensified use. ***indicated to improve glycemic control in combo w/glucophage and or sulfonylureas*** [ADVANTAGES] -wt loss average 2-6lbs -very rare hypoglycemic effect [DISADVANTAGES] -N&V -injectable only form [COST] -EXPENSIVE $300 mo "injection only" [SAFETY] **FDA advisory to monitor patient carefully for the development of pancreatitis.*** FDA advises clinicians to teach signs of acute pancreatitis & the need for emergent attention (persistent abdominal pain, usually accompanied w/vomiting) -Use cautiously in pts with mild-moderate renal impairment (CrCl 30-50) [CONTRAINDICATIONS] *Hx of pancreatitis *CrCl<30ml/min *Gastroparesis [ELIMINATION OF DRUG} -renally eliminated

GLUCAGON-LIKE PEPTIDE 1 AGONIST (GLP) [incretin mimetics] exenatide (Byetta) (Bydureon) liraglutide (Victoza)

Found primarily in the liver and kidney. In liver disease, usually parallels changes in alkaline phosphatase Marked elevation often noted in: 1. obstructive jaundice 2. Hepatic metastasis 3. intrahepatic cholestasis

Gamma glutamyl transferase (GGT)

lateral bowing tibia joint laxity normal until age 2 common in toddler Full ROM [Refer to orthopedics if:] continues after age 2 unilateral progressively worse after 1st yr

Gene Varum think too much rum

CLASSIC CASE OF? -Middle aged female looses large amount of weight rapidly, becomes irritable, anxious, and hyperactive -Insomnia with more frequent bowel movements -Amenorrhea and heat intolerance -Goiter present -Hypertension -Goiter - diffusely enlarged without nodules / may be tender to palpation or asymptomatic -Hands / fingers - fine tremors on hands & sweaty palms -Eyes - Exophthalmos in late stages -Cardiac - tachycardia, a-fib, CHF

Grave's Disease classic case

During a cardiac assessment on a 38-year-old patient in the hospital for "chest"pain," the nurse finds the following: jugular vein pulsations 4 cm above sternal angle when he is elevated at 45 degrees, blood pressure 98/60mmHg, heart rate 130 beats per minute, ankle edema, difficult in breathing when supine, and an S3 on auscultation. Which of these conditions best explains the cause of these findings?

HF

SE of prolonged use of topical steroids can cause (6)

HPS axis suppression (excessive prolonged use) striae, skin atrophy, telangiectasis, acne, and hypopigmentation.

CLASSIC CASE OF? Middle age woman who is overweight complains of fatigue, weight gain, cold intolerance, constipation, and menstrual abnormalities. She may have an autoimmune disorder. May have afib In severe cases, she will present with puffy face, hands, and feet. she has thickening of her skin, thinning of the outer third of the eyebrow, and cognitive symptoms such as slowed thinking, poor short-term memory and depression or dementia

Hashimoto's Thyroiditis

HBsAg (Hep B surface antigen) HBsAg(+) means?

Hepatitis B virus on board You can't have HBsAg+ without having the HB (surrogate marker)

CLASSIC CASE: Complains of acute onset of eye pain, photophobia, and blurred vision of the affected eye. Rash noted on the side of the temple and on the tip of the nose. *Use fluorescein dye with black lamp in dark room* -fernlike lines Requires referral and STAT meds

Herpes Keratitis Referral to ER or ophthalmologist STAT STAT Zovirax or Valtrex NO STEROID DROPS

A viral skin infection of the finger(s) that is caused by HSV 1 or 2, from direct contact with either a cold sore or genital herpes lesion. CLASSIC CASE: Pt c/o of acute onset of extremely painful red bumps and small blisters on the sides of the finger or the cuticle area or on the terminal phalanx of one or more fingers, may have recurrent outbreaks. ***Assess for symptoms of HSV1 and 2.

Herpetic Whitlow [Mild self-limiting infection] Analgesic or NSAIDs for pain [Severe infection] Treat w/ acyclovir (Zovirax) Pt education: cover skin lesion completely with large band-aid until the lesions heal. avoid sharing personal items, gloves towels.

PCOS

Hyperinsulinemia/insulin resistance causes altered HPA feedback. -LH & FSH are increased ***Higher risk for*** CHD, DM, INFERTILITY, obesity, BREAST and ENDOMETRIAL CANCER bc of unopposed estrogen Tx: weight reduction, OCPs, clomiphene, ketoconazole, spironolactone

which of the following statements is most consistent with IBD, IBS, or both conditions? extraintestinal manifestations occasionally include nondestructive arthritis and renal calculi

IBD

which of the following statements is most consistent with IBD, IBS, or both conditions? the etiology likely involves an autoimmune response to the GI tract

IBD

which of the following statements is most consistent with IBD, IBS, or both conditions? this is a potentially life-threatening condition

IBD

On physical presentation, a person with __________ usually has tenderness in the sigmoid region; the remainder of the examination is usually normal.

IBS

which of the following statements is most consistent with IBD, IBS, or both conditions? the condition is often referred to as spastic colon by the general population

IBS

which of the following statements is most consistent with IBD, IBS, or both conditions? the etiology is considered to be an alteration in small and large bowel motility

IBS

which of the following statements is most consistent with IBD, IBS, or both conditions? the pt population is predominately female

IBS

there are four typical bowel patterns with irritable bowel syndrome: diarrhea/constipation alternating, eventually one becomes more dominant

IBS-A

there are four typical bowel patterns with irritable bowel syndrome: constipation predominant - can be episodic then intractable to laxative with hard, narrowed stool caliber and a sense of incomplete evacuation for wks or months interrupted with bouts of diarrhea

IBS-C

there are four typical bowel patterns with irritable bowel syndrome: mixed diarrhea and constipation

IBS-M

Appropriate antimicrobial treatment for a 25-year-old man with acute bacterial prostatitis is:

IM ceftriaxone followed by oral doxycycline.

what is the emergency tx of hypocalcemia?

IV replacement of calcium and is guided by the severity of hypocalcemia paired with the pt's s/s pg 831

What do these labs indicate? HbsAg-negative Anti-HBc-positive Anti-HBs-postive

Immune due to natural infection

Classic case: Tachypnea (>25 breaths/min), tachycardia or bradycardia, cyanosis, anxiety, exhaustion, fatigue, diaphoretic, accessory muscle use, "quiet lungs" without audible wheezing or breath sounds

Impending Respiratory Failure in Asthmatic Patient STAT adrenaline injection, call 911. O2 4-5L/min, albuterol nebs, parenteral steroids, antihistamines. Improvement if wheezing and breath sounds present after treatment

-Failure rate 0.01% -Progesterone only -Protects for 3 years -Very few SE except increased cost -Requires informed consent

Implanon

Case Scenerio for boards: (Leik) A type 1 diabetic patient is on regular insulin and NPH( not mixed) that is injected BID. The first dose is before breakfast and the second dose is at bedtime. The blood sugar results rom her logs (fasting, before lunch, dinner and bedtime) show that the lunchtime values are higher than normal. What insulin dose should be increased?

In this case it would the NPH component that she takes in the morning. Reg insulin peaks between breakfast and lunch (most gone by lunchtime). NPH peaks between 6-14 hours so it will cover the postprandial spike that she gets after lunch.

Specific lumbar nerve root findings... numbness to lateral aspect of foot and posterior calf weakness on plantar flexion; butt pain, absent achilles reflex, how do you assess?

L5-S1: **(have pt walk on toes)**

In graves disease the TSH would be __________ (High or low) and the free T4 would be ___________ (High or low)

LOW, HIGH

insidious limp with knee pain, moves up thigh to groin, has passive internal rotation

Legg Calve Perthes Disease less than 6, less than 1/2 of femoral head involved older than 6, more than 1/2 of femoral head is involved =referral

asceptic or avascular necrosis of the femoral head ***insidious onset of limp w/pain in the knee, up the thigh out towards the groin laterally *** Test Q limited passive internal rotation and abduction of the hip joint common in sickle cell patients GOAL of TX= Restore ROM while maintaining femoral head w/in the acetabulum

Legg-Calve Perthes Disease

Initial management of IFG (impaired fasting glucose)

Lifestyle modifications: -7% of body weight loss -Mod physical activity of at least 150min per week

Describe the process of the menstrual cycle

Low estrogen promotes the hypothalamus to secretes gonadotropin releasing hormone (GnRH) which causes the pituitary gland to release leutinizing and follicle stimulating hormone (LH & FSH). FSH promotes development of follicles into a corpus luteum which secretes progesterone. Progesterone thickens and stabilizes the endometrial lining Estrogen is secreted by the ovaries proliferates the endometrial lining. A surge of LH causes ovulation (egg is released). (Starts leuteal phase) High estrogen and progesterone levels activate the negative feedback loop, and FSH and LH production is stopped. Estrogen and progesterone levels drop causing ischemia and vasoconstriction of endometrium. 4-24 hr later, the endometrium begins to slough away (menstruation) Bleeding starts day 1 of menses and follicular phase

DM medication that is virtually identical in action to sulfonylureas

Meglitinide analogues "meglitinides" repaglinide ( ) nateglinide (Starlix)

4 times day with food. prophylaxis against NSAID induced ulcers. stimulates mucous and bicarb production, stimulates uterine contraction and induce abortion. D/C offending agent if possible.

Misoprostol (cytotec)

When is stiffness the worst with RA?

Morning

Middle-aged adult female complains of many weeks of plantar foot pain that is worsened by walking, especially while wearing high heels or tight narrow shoes. The pain is described as burning and/or numbness and is located on the space between the third and fourth toes (metatarsals) on the forefoot. Physical exam of the foot may reveal a small nodule on the space between the third and fourth toes. Some patients palpate the same nodule and report it as "pebble-like."' Diagnosis? Treatment?

Morton's Neuroma Diagnosis ■ Diagnosed by clinical presentation and history. ---Mulder Test--- ■Refer to podiatrist Treatment Plan ■ Avoid wearing tight narrow shoes and high heels ■ Use forefoot pad. ■Wear wellpadded shoes.

What medications are used for urge incontinence?

Muscarinic receptor agonists: Immediate release 1. Oxybutynin 2. Tolterodine 3. Trospium

Difficult walking, muscle weakness, frequent falls, delay in muscle skills, drooping eyelids (ptosis), drooling, intellectual retardation, contractures, and skeletal deformities. genetic diagnosed around age 3-5 more common in males Gower's maneuver (weak pelvic girdle) typically wheelchair dependent by age 12 death from cardiopulmonary failure Labs: CK elevated (markedly) Diagnostics: ECG abnormal EMG =myopathy biopsy of muscle=necrosis DNA analysis

Muscular dystrophy

The nurse is performing a genitourinary assessment on a 50-year-old obese male laborer. On examination the nurse notices a painless round swelling close to the pubis in the area of the internal inguinal ring that is easily reduced when the individual is supine. These findings are most consistent with a(n) _____ hernia.

NS: direct inguinal Direct inguinal hernias occur most often in men over the age of 40 years. It is an acquired weakness brought on by heavy lifting, obesity, chronic cough, or ascites. The direct inguinal hernia is usually a painless, round swelling close to the pubis in the area of the internal inguinal ring that is easily reduced when the individual is supine. See Table 24-6 for a description of scrotal hernia. See Table 24-7 for descriptions of femoral hernias and indirect inguinal hernias.

>-1.0 SD=___________ -1.0-2.5=__________ -2.5=___________

Normal Osteopenia Osteoporosis

Sacrcoidosis findings

Ocular symptoms include blurred vision, eye pain, severe redness, and sensitivity to light. pulmonary complaints include dyspnea on exertion, cough, and chest pain. Dermatologic signs include rash, lesions, color change, and nodule formations just under the skin. Systemic symptoms include fever, fatigue, anorexia, and arthralgias, Hypercalcemia and hypercalciuria

Off Label Migraine Prophylaxis

Off Label: o CCBs o TCAs (amitriptyline) o SSRIs, clonidine, lithium, Cyproheptadine (Periactin®) o B2 (400 mg riboflavin) & Mg o Feverfew

You need to make sure patient with Bell's Palsy can close the eye completely or it may lead to a:

Permanent change in vision!!! Every hour while awake use a lubricating drop Sleeping at night need to inert a viscous eye drop (lacro lube)

DANGER SIGNS OF? -Random episodes of severe HTN -SBP >200 or DBP >110 w/ HA, tachy, anxiety -Resolve spontaneously and occur at random -Between episodes - BP is normal

Pheochromocytoma

What kind of nipple discharge would you expect to see in physiologic drainage vs ductal ectasia vs malignancy?

Physiologic (normal if ever breastfed): white to brown Ductal ectasia: thick, cheesy, green Malignant: clear, serous, serosanguinous, or sanguineous

RLQ pain when pressure is applied to the LLQ

Positive Rovsing's sign: late sign of appendicitis

carpal tunnel test: tapping over the median nerve on the flexor surface of the wrist produces a tingling sensation radiating from the wrist to the hand

Positive Tinel's sign

Hale's Lactation Risk Category • L4 - Hazardous

Positive evidence of risk but may be used if maternal life-threatening situation • Lithium • Ergot preparations • Daily HD systemic corticosteroids - Equivalent prednisone ≥10 mg/d long term

Collateral Ligaments (Knees)

Positive finding is an increase in laxity of the damaged knee (ligament tear). ■ Valgus stress test of the knee: A test for medial collateral ligament (MCL). ■ Varus stress test of the knee: A test for the lateral collateral ligament (LCL).

Eclampsia

Preeclampsia + tonic clonic seizure* Usually stage 2 HTN Dx: CBC, CMP, coag panel for DIC concern Tx: Refer, hospitalization, anticipate delivery asap

The mother of a 3-month-old infant states that her baby has not been gaining weight. With further questioning, the nurse finds that the infant falls asleep after nursing and wakes up after a short amount of time, hungry again. What other information would the nurse want to have?

Presence of dyspnea or diaphoresis when sucking

_____ Signs of Pregnancy These are the softest and least objective signs. Can be caused by many other conditions besides pregnancy. ■ Amenorrhea. ■ Nausea/vomiting (most common in fi rst trimester in the morning, usually disappears by the second trimester). ■ Breast changes (swollen and tender). ■ Fatigue. ■ Urinary frequency. ■ Slight increase in body temperature.

Presumptive Signs of Pregnancy

______ Signs of Pregnancy These are the softest and least objective signs. Can be caused by many other conditions besides pregnancy. ■ Amenorrhea. ■ Nausea/vomiting (most common in fi rst trimester in the morning, usually disappears by the second trimester). ■ Breast changes (swollen and tender). ■ Fatigue. ■ Urinary frequency. ■ Slight increase in body temperature.

Presumptive Signs of Pregnancy

Male complains of a prolonged and painful erection of the penis for several hours (at least 2 to 4 hours).

Priapism Males with sickle cell disease are at very high risk (6% to 45%). Other risk factors are high doses of erectile dysfunction drugs, cocaine, quadriplegia, and others. Several types of priapism. The ischemic form of priapism is a medical/surgical emergency.

CLASSIC CASE: Sudden blockage of aqueous humor causes marked increase of the IOP, causing ischemia and permanent damage to the CNII (optic nerve). Assessment reveals fixed and mildly dilated cloudy pupil that looks more oval than round-shaped and the conjunctival injection with increased lacrimation. ED referral, Blindness if not treated

Primary Angle Closure glaucoma

■ Goodell's sign (4 weeks): cervical softening. ■ Chadwick's sign (6-8 weeks): blue coloration of the cervix and vagina. ■ Hegar's sign (6-8 weeks): softening uterine isthmus. ■ Enlarged uterus. ■ Ballottement (seen in mid-pregnancy); when the fetus is pushed, it can be felt to bounce back by tapping the palpating fingers inside the vagina. ■ Urine or blood pregnancy tests (beta hCG). ■ "Quickening": the mother feels the baby's movements for the first time. Starts at 16 weeks.

Probable Signs of Pregnancy

Older to elderly male complains of a new onset of low back pain, rectal area/perineal pain or discomfort accompanied by obstructive voiding symptoms such as weaker stream and nocturia. May be asymptomatic. More common in older males

Prostate Cancer May be asymptomatic. More common in older males (greater than 50 years), obese males, men with a family history of prostate cancer (father, brother), and Black males.

Painless and hard fi xed nodule (or indurated area) on the prostate gland on an older male that is detected by DRE. ■ Elevated PSA: Greater than 4.0 ng/mL. ■ Diagnostic test: Biopsy of prostatic tissue (obtained by transurethral ultrasound). ■ Screening test: PSA level with DRE. If limited life span (less than 10 years), not recommended. Treatment Plan ■ Refer to urologist if PSA greater than 4.0 ng/mL, suspect or not sure if prostate cancer. ■ Individualize screening based on risk factors. Discuss risk (bleeding, infection, impotence, procedures, and psychological trauma) versus benefi ts. ■ Most cancers are not aggressive and are slow-growing. Watchful waiting/monitoring by urologist. ■ Drug therapy with antiandrogens (Proscar), hormone blockers (i.e., Lupron), others

Prostate cancer

CLASSIC CASE: The patient complains of pruritic erythematous plaques covered with fine silvery-white scales along with pitted fingernails and toenails. The plaques are distributed in the scalp, elbows, knees, sacrum and the intergluteal folds. Partially resolving plaques are pink colored with minimal scaling. When affects the joints, pt c/o of painful red warm swollen joints (migratory arthritis) in addition to the skin plaques. Inherited skin disorder in which squamous epithelial cells undergo rapid mitotic division abnormal maturation. Rapid turnover.

Psoriasis Inherited skin disorder in which squamous epithelial cells undergo rapid mitotic division abnormal maturation. Rapid turnover.

proximal interphalangeal joints metacarpophalangeal joints wrists, swollen edema with REDNESS AND HEAT synovial aspirate with inflammatory changes and WBC Symmetrical inflammation, 35-50yrs, Women more common, Worse in morning, better with activity

RA high dose salicylate, NSAID, SMARDS, steroids, gold salt injections, methotrexate' fatigue, weakness, malaise, anorexia, wt loss, ESR elevated, ANA +

CLASSIC CASE: The classic rash looks like small red spots (petechiae) and starts to erupt on both the hands and feet (incl palms and soles), rapidly progressing toward the trunk until it becomes generalized. The rash appear on the 3rd day after the abrupt onset of fever of 103-105, that is accompanied by a severe HA, myalgia, conjunctival injection, N&V, and arthralgia. Can be fatal, with mortality rate ranging from 3-9%. Most cases occur in the spring and early summer

RMSF

A chronic systemic disease characterized by inflammation of the joints, prolonged stiffness, pain, and swelling. It involves multiple joints and has a symmetrical distribution. Classic Case Adult to middle-aged female complains of gradual onset of symptoms over months with daily fatigue, low-grade fever, generalized body aches, and myalgia. Complains of generalized aching joints, which usually involves the fingers/hands and wrist. Morning stiffness lasts longer than DJDs with painful, warm, and swollen joints. Swollen fingers with warm tender joints (PIP and DIP). Also called "sausage joints." accompanied by systemic symptoms like fatigue, fever, normocytic anemia, etc.

Rheumatoid Arthritis [Labs] ■ Sedimentation rate: Elevated ■ CBC: Mild microcytic or normocytic anemia common ■ Rheumatoid factor (RF): Positive in 75% to 80% of patients ■ Radiographs: Bony erosions, joint space narrowing, subluxations (or dislocation) Refer to rheumatologist for early aggressive mgmt to minimize joint damage. [Medications] ■ NSAIDs (i.e., ibuprofen, naproxen sodium) help to relieve inflammation &pain. ■ Steroids: Systemic oral doses. ■ Steroid joint injections (synovial space). ■ Disease-modifying agent for rheumatoid disease (DMARD) such as methotrexate, sulfasalazine, cyclosporine, and hydroxychloroquine (an antimalarial drug). ■ Gold salt compounds as intramuscular (IM) injections (i.e., Ridaura, Myochriysine). ■ If poor response to DMARDs, use other options: anti tumor necrosis factor (anti-TNF) drugs (Humira, Enbrel, Remicade), alkalating agents (i.e., Cytoxan, cyclosporine). ■ Surgery: Joint replacement (hip, knees). ■ Biologics or anti-TNF drugs are contraindicated (or should be stopped) if signs and symptoms of infection (fever, sore throat). [Complications] ■ Uveitis (ophthalmologist STAT). ■ Scleritis, vasculitis, pericarditis, and so

Infectious (bacterial) epididymitis treatment Improvement by __________, if not refer out

Rocephin 250mg IM plus Doxycycline 100mg BID for 10day 3days/72hrs

Classic Case? Physically active patient (jogs or runs) complains of ball-like mass behind one knee that is soft and smooth. Pressure pain or asymptomatic. If cyst ruptures, will cause an inflammatory reaction resembling cellulitis on the surrounding area (the calf) such as redness, swelling, and tenderness. Diagnosed by clinical presentation and history. MRI if diagnosis is uncertain. Can be plain or infected/

Ruptured Baker's Cyst (Bursitis) A Baker's cyst is a type of bursitis that is located behind the knee (popliteal fossa). Sometimes when a joint is damaged and/or infl amed, synovial fluid production increases, causing the bursa to enlarge. Bursae are the protective synovial sacs that are located on certain on joints.

Dry skin, loss of elasticity, atherosclerosis, CAD, decrease in breast size and tone, vasomotor instability, mood changes, depression, sleep disturbance, osteoporosis, cystitis, stress incontinence, atrophic vaginitis, dyspareunia, uterine prolapse,

S/S of menopause?

What DM Medication? [ACTION] Lowers renal glucose threshold. Increases urinary glucose excretion (blocks 90% of glucose) [EFFECTIVENESS] -A1C reduction of 0.7-1% [ADVANTAGES] -weight loss [DISADVANTGES] - Very expensive -UTI -vaginal yeast infections [COST] -approx $450/mo [SAFETY] -watch for hypotension -hypokalemia? [CONTRAINDICATION] -poor renal function [DRUG ELIMINATION] -renally eliminated

SODIUM-GLUCOSE COTRANSPORTER 2 INHIBITOR (SGLT2) canagliflozin (Invokana) dapagliflozin (Farxiga) empagliflozin (Jardiance) "flozin" think urinary flow

HELLP (Hemolysis, Elevated Liver Enzymes, and Low Platelets) Syndrome

Serious but rare complication of preeclampsia. Classic patient is a multipara older than 25 years who is in the third trimester of pregnancy. Presence of the signs and symptoms of preeclampsia that is accompanied by right upper quadrant (or midepigastric) abdominal pain with nausea/vomiting and malaise. Symptoms can present suddenly. Lab abnormalities are elevation of the alanine aminotranferease (AST), aspartate aminotransferase (ALT), total bilirubin, lactate dehydrogenase (LDH) with decreased number of platelets (<100,000 cells/microl) and hemoglobin and hematocrit.

Spontaneous dislocation of femoral head. pain is in the groin and goes down the thigh to the knee ***unable to properly hip as femur abducts, rotate externally*** may have limb shortening (due to proximal displacement of metaphysis ****immediate referral to orthopedist**** -monitor other hip for same problem -Need Xray, often require surgery to pin fixate joint no ambulation to prevent irreparable joint damage

Slipped Capital Femoral Epiphysis (SCFE)

Rebound hyperglycemia -Severe nocturnal hypoglycemia stimulates counter regulatory hormones (glucagon) to be released from the liver -Cause high fasting blood glucose by 0700 -R/t over treatment with evening/bedtime insulin -More common with DM type I -Requires glucose check very early (0300) for 1-2 wks -Tx w/Snack before bedtime (OR) d/c intermediate-acting insulin (NPH) dose or lower bedtime dose for both (NPH) & reg insulin

Somogyi Effect

Which test is demonstrated when the examiner applies pressure to the top of the head with the neck bending forward, producing pain or numbness in the upper extremities?

Spurling

CLASSIC CASE: The classic lesion appear target-like (bulls-eye), then multiple lesions start erupting abruptly and can range from hives, blisters (bullae), petechiae, purpura and painful hemorrhagic lesions. Extensive mucosal surface involvement (eyes, nose, mouth, esophagus and bronchial tree) is observed. There could be a prodrome of fever w/ flu-like symptoms before rash appears. The is a rare and severe hypersensitivity reaction that is causes by medications, infections, and malignancies. The drug classes often most associated are: PCNs, sulfas, barbituates, and phenytoin. Mortality is 25-35%. HIV infected pts have a 40-fold increased risk of contracting due to their TMP-SX compared to regular population.

Stevens-Johnson Syndrome (Erythema Multiforme Major)

CLASSIC CASE: Acute onset of pharyngitis, pain on swallowing, and mildly enlarged submandibular nodes. Not associated with rhinitis, watery eyes, or congestion (coryza) as seen in the common cold. Anterior cervical nodes mildly enlarged. Adult may report having a small child attending preschool. Complications include scarlet fever, rheumatic fever and peritonsillar abscess.

Strept Throat Throat C&S or Rapid Strep testing 1st line treatment PCN QID x10days PCN allergy, switch to Fluoroquinolone (CI <18y)

CLASSIC CASE: Complains of sudden onset of bright red blood in one eye after an incident of severe coughing, sneezing, or straining. May also be due to trauma such as a fall. Denies vision loss or pain. Watchful waiting and reassurance.

Subconjunctival hemorrhage

often precipitated by large, fatty meal sudden appearance of steady, severe pain in epigastrium or right hypochondriac, vomiting

Sx of cholecystitis inflammation the gallbladder, associated with gall stones in >90% of cases

Classic Case Typical patient is a woman of age between 20 and 35 years. Classic rash is the maculopapular butterfly-shaped rash on the middle of the face (malar rash). May have nonpruritic thick scaly red rashes on sun-exposed areas (discoid rash). Urinalysis positive for proteinuria

Systemic Lupus Erythematosus (SLE)

(T/F) In a patient with suspected superficial thrombophlebitis in the calf, the abnormalities in the lower-extremity examination are potentially enhanced by having the patient stand for approximately 2 minutes.

T

(T/F) In the treatment of venous stasis ulcer that is not responding to standard therapy, additional therapeutic options include hyperbaric oxygen therapy.

T

T/F: If a patient with hypothyroidism has heart disease, start with a low dose of levothyroxine, and titrate slowly, otherwise you may precipitate CHF or MI

T

(T/F) Prescribing a direct thrombin inhibitor is an acceptable therapeutic to reduce the risk of recurrent DVT.

T Pradaxa is considered an alternative for heparin and coumadin. It can be used for: - stroke prevention r/t afib -prevent DVT recurrence -prevent PE recurrence -

Destruction of B-cells in the islets of Langerhans Abrupt cessation of insulin production Uncorrected - body fat used for fuel - cause ketones to build up - cause ketotic acidosis - coma Most juveniles - occasionally adults (Maturity onset of diabetes of the young (MODY))

T1DM

Progressive decreased secretion of insulin w/ peripheral insulin resistance Chronic state of hyperglycemia and hyperinsulinemia Strong genetic component 85-90% of US cases of DM

T2DM

What DM Medication? Cant be mixed with insulins Not to be given w/HF Not to be given w/insulin -Rare risk of hepatic toxicity (<0.5%) mainly if used w/ insulin or sulfonylureas *-monitor ALT -high dose associated with bone fx, osteopenia *-can cause/exacerbate CHF -edema risk -both r/t to increase in circulating volume A1c reduction of 1-2% with intensive (max) use

THIAZOLIDINEDIONE (TZDs) pioglitazone (Actos) rosiglitazone (Avandia)

Management with epididymitis for >35 y.o.

TMP/SMZ-DS OR Cipro

Bacterial prostatitis is hard to treat, because the prostate gland is hard to penertrate treatment with? how long?

TMPS or chipper for 6 weeks

subclinical hypothyroidism pattern? H, L, or N TSH T4 T3

TSH-H T4-N T3-N

primary hyperthyroidism pattern? H, L, or N TSH T4 T3

TSH-L T4-elevated T3-N

primary hypothyroidism pattern? H, L, or N TSH T4 T3

TSH-high T4-low T3-N

What DM medication? Their contraindications: -w/nitrate use - w/insulin use -dx of heart failure -can cause and or exacerbate heart failure -potential increased risk of bladder cancer

TZDs "glitazones" pioglitazone (Actos) rosiglitazone (Avandia)

How do you minimize the risk of drug-induced esophagitis, with biphosphonates?

Take the medication in the morning with a full glass of water. 30 minutes before food, other liquids, or medications. Remain upright for at least 1 hour. alendronate (Fosamax), ibandronate (Boniva), risedronate (Actonel) zoledronic acid (Reclast)

What do you do if one pill is missed?

Take two tomorrow Use back up

Bulge test evaluates for

Test is used to determine the presence of excess fluid in the knee. To perform the test with the patient's knee extended, milk the medial aspect of the knee upward two or three times, and then tap the lateral side of the patella.

straight-leg raise test

Test often performed to determine whether a patient with low back pain has an underlying herniated disk

Teenage to young adult male complains of nodule, sensation of heaviness or aching, one larger testicle, and/or tenderness in one testicle.

Testicular Cancer Testicular cancer can present as a new onset of a hydrocele (from tumor pressing on vessels). Usually painless and asymptomatic until metastasis. More common in White males age 15 to 30 years. Rare in African Americans. Higher risk associated with history of cryptorchidism

A male (usually adolescent) reports waking up at midnight or in the morning with male (usually adolescent) reports waking up at midnight or in the morning withabrupt onset of an extremely painful and swollen red scrotum. Frequently accompanied by nausea and vomiting. Affected testicle/scrotum is located higher and closer to the body than the unaffected testicle. Missing cremasteric reflex. Majority of cases (2/3) occur between the ages of 10 and 20 years.

Testicular Torsion

Antibiotics for Pregnant Women

The antibiotics below are category B. ■ Penicillins - Amoxicillin (Amoxil), penicillin, dicloxacillin. ■ Cephalosporins - First generation: cephalexin (Keflex), cefadroxil (Duricef). - Second generation: cefuroxime axetil (Ceftin), cefaclor (Ceclor), cefprozil (Cefzil). - Third generation: ceftriaxone (Rocephin) injections, cefdinir (Omnicef), cefixime (Suprax). - Fourth generation: cefepime (Maxipime) injection/IV (used mainly in hospitals). ■ Macrolides - Erythromycins - Erythromycin ethylsuccinate (E-mycin). -Erythromycin estolate (EES) can cause hepatoxity in pregnant women and its use in pregnancy is not recommended. - Azithromycin (Zithromax), clarithromycin (Biaxin) - Clarithromycin (Biaxin) is the only macrolide that is a category C. Avoid use during pregnancy. Consult with physician before use and discuss risk versus benefits. ■ Nitrofurantoin - Nitrofurantoin (Furadantin, MacroBid) - Do not use with glucose-6-phospate dehydrogenase defi ciency (G6PD anemia)since it will cause hemolysis (anemia, jaundice, dark urine).

Aside from corticosteroids, Other agents used in the treatment of sarcoidosis include what drugs?

The antimalarial drug hydroxychloroquine Immune suppressing medications used to treat rheumatoid arthritis (e.g., methotrexate, azathioprine tumor necrosis factor inhibitors [e.g., infliximab]). Other agents : chlorambucil, cyclophosphamide, and cyclosporine.

What is the ADA's recommendation for screening people generally for DM?

The entire population >=45 every year if screening is normal

Action: Enhances insulin sensitivity in muscle tissue - decreases peripheral resistance - and reduces hepatic glucagon production -Take daily with breakfast -Avoid Class III/IV HF (Causes water retention & edema) -Actos is associated with a rare form of bladder Ca -Causes weight gain -Monitor ALT

Thiazolidinediones (TZDs) Rosiglitazone (Avandia) & pioglitazone (Actos)

How could you treat hyper-thyroidism at time of diagnosis?

Treatment of Hypothyroidism can include: 1. Start beta-blocker with B1 and B2 blockade (Propanolol, nadolol) to counteract tachycardia/tremor 2. Anti-thyroid medication such as propylthiouracil (PTU) and methimazole (Tapazole) - can consult endocrinology. 3. Radioactive iodine (RAI) with end-result of thyroid ablation and hypothyroidism

what are two neuromuscular signs that can be tested and are indicative of hypocalcemia?

Trousseau's sign (carpal spasm) Chvostek's sign (spasm of the facial muscle) pgs 830-831

T/F in epicondylitis Local corticosteroid injection can be helpful if symptoms persist beyond 6 to 8 weeks or are particularly severe.

True

T/F: • Migraine with and without aura increases relative risk of ischemic stroke

True

True or False: In an adult with BMI greater than 40 kg/m2 who is being treated with TMP-SMX for CA-MRSA,the recommended dose is two tablets bid

True

weight loss medication: true or false? phentermine/topiramate (Qsymia) carries a warning about potential teratogenic effects

True

a microaneurysm is the earliest manifestation of a diabetic retinopathy True False

True they appear as small round dark red dots on the retinal surface

Screening for sports physical recommendations

Urinalysis, CBC, and radiographic imaging of the spine are generally NOT recommended for individuals with an unremarkable medical history because these are not helpful screening

Varicose veins in scrotal sac (feels like "bag of worms"). New-onset varicocele can signal testicular tumor (20%) or a mass that is impeding venous drainage. Order an ultrasound of the scrotum.

Varicocele

What Does DEXA scan measure?

WASH-- Wrist Ankle, Spine Hip

appendiceal rupture/perforation is characterized by what (4) findings

WBC >20,000-30,000mm3 fever over 102 peritoneal inflammation findings (Rovsing, obturbator, psoas, ) symptoms lasting >48hrs

Which joints are most often affected with OA? Symmetric or asymmetric

Weight bearing, Fingers, hands, wrists, Heberdens nodes (DIPs) Bouchards nodes (PIPs) asymmetric

S/S of Icteric hepatitis?

Weight loss, jaundice, pruritis, RUQ pain, clay colored stool, dark urine, low-grade fever, hepatosplenomegaly

Which Antidepressant should be Avoided in presence of eating disorder or if anorexia is a major component of depression. Weight loss often seen (28% >5 lb [2.3 kg]) after initiation of therapy And not to be given if history of or risk for seizure, closed head injury history, history of quiescent epilepsy Seizure risk worsens if dose increased rapidly

Wellbutrin

RECOMMENDATIONS FOR OSTEOPOROSIS SCREENING

Women age 65 and older and men age 70 and older, regardless of risk factors • Younger postmenopausal women, women in the menopausal transition, and men age 50 to 69 with clinical risk factors for fracture • A woman or man after age 50 who has broken a bone • Adults with a condition (e.g., rheumatoid arthritis) or taking a medication (e.g., long-term glucocorticoid) associated with low bone mass or bone loss]

Inherited skin disorder that results in extremely dry skin that is may involve mucosal surfaces such as the mouth or conjunctiva of the eye.

Xerosis mouth (xerostomia) or conjunctiva of the eye (xerophthalmia)

After a while Lantus or Levemir won't be enough and additional insulin is needed. How will you treat, there are two options.

[Basal Plus] administration of a short acting insulin that is given before biggest meal. (easiest method) [Basal Bolus] administration of a short acting insulin at each meal. *requires frequent BG checks and a motivated patient! (difficult method) Short-acting insulin (Humulin R / Novolin N)

Risk factors for nephrolithiasis avoid?

[Hx] family hx low fluid intake gout bariatric surgery [Avoid] rhubard spinach beets chocolate tea meats

When do you refer out for Genu Varum (bowleg) (3)

[Refer to orthopedics if:] continues after age 2 unilateral progressively worse after 1st yr

a 65 y/o diabetic has been on oral anti-hyperglycemic agents and is still having poor glycemic control. His AM fasting glucoses range from 140s-160s. You decide to add insulin. He weighs 127 kilograms. What should the NP order as an initial starting dose? a. 10 units long-acting insulin at bedtime b. 30 units long-acting before breakfast c. 5 units intermediate insulin at bedtime d. 20 units short-acting insulin at breakfast

a. 10 units long-acting insulin at bedtime according to the ADA,2009 an intermediate or long-acting insulin should be started at bedtime or morning as a once daily dosage. A prudent starting dose is either 10 units insulin or 0.2 units per kg (approximately 25 units of insulin)

you see a 22 y/o male who is an injection drug user who has recently been dxed with chronic HBV infection. you recommend additional testing for all of the following EXCEPT: a. Lyme disease b. HIV c. HAV d. HCV

a. Lyme disease

concomitant use of an SSRI with which of the following herbal products can potentially lead to serotonin syndrome? a. St. John's wort b. kava kava c. gingko biloba d. valerian root

a. St. John's wort

what is the most sensitive lab assay for screening and identifying the vast majority of ambulatory pts with primary hypothyroidism? a. TSH only b. TSH and T4 c. TSH, T4 and T3 d. TSH and TRT (thyrotropin releasing hormone?

a. TSH only

the mechanism of action of the DPP-4 inhibitors is as: a. a drug that increases levels of incretin, increasing synthesis and release of insulin from pancreatic beta cells b. a product virtually identical in action to sulfonylureas c. a drug that increases insulin action in the peripheral tissues and reduces hepatic glucose production d. a facilitator of renal glucose excretion

a. a drug that increases levels of incretin, increasing synthesis and release of insulin from pancreatic beta cells DIPEPTIDYL-PEPTIDASE-4 INHIBITOR "gliptans" sitagliptan (Januvia) saxagliptin (Onglyza) linagliptin (Tradjenta) alogliptin (Nesina) [ACTION] increases levels of incretin, increasing synthesis and release of insulin from pancreatic beta cells and decreasing release of glucagon from the pancreatic alpha cells [EFFECTIVENESS] -A1C reduction of 0.6-1.4% w/ intensified use. ***indicated to improve glycemic control in combo with Metformin or TZD*** [Common SE] -Well tolerated -Weight neutral [COST] -EXPENSIVE $300-400 mo [SAFETY] -FDA advisory to monitor patient carefully for the development of pancreatitis after initiation or dose increase. Mainly with sitagliptin (Januvia) sitagliptin/metformin (Janumet) -Dose adjustment in renal impairment [CONTRAINDICATION] **Pancreatitis** [ELIMINATION OF DRUG} -renally eliminated, hence the need adjust dose based on renal impairment

first-line pharmacological interventions for milder OA should be a trial of: a. acetaminophen b. tramadol c. celecoxib d. intrarticular corticosteroid injection

a. acetaminophen

a 30 y/o male who is sexually active presents with pain during bowel movements. he is negative when checked for hemorrhoids, but has a tender prostate gland. what should be suspected? a. acute bacterial prostatitis b. prostate cancer c. benign prostatic hyperplasia d. gonorrhea

a. acute bacterial prostatitis

a 50 y/o comes to the NP clinic for evaluation. he c/o fever 101F, chills, pelvic pain, and dysuria. he should be dx with: a. acute bacterial prostatitis b. chronic bacterial prostatitis c. urinary tract infection d. non-bacterial prostatitis

a. acute bacterial prostatitis

a 50 y/o male comes to the NP clinic for evaluation. He c/o fever 101F, pelvic pain, and dysuria. He should be dx with: a. acute bacterial prostatitis b. chronic bacterial prostatitis c. urinary tract infection d. non-bacterial prostatitis

a. acute bacterial prostatitis

the primary risk factor for development of breast cancer in women of average risk is: a. age b. smoking hx c. number of live births d. exposure to estrogen

a. age

what class of medications can be used to tx benign prostatic hyperplasia and provide immediate relief? a. alpha-1 blockers b. 5-alpha reducatse inhibitors c. diuretics d. analgesics

a. alpha-1 blockers

a 13 y/o male has exhibited the first sign that he is experiencing sexual maturation. he has: a. an increase in testicular size b. an enlargement of the scrotum c. an increase in length of the penis d. scrotal and penile changes

a. an increase in testicular size

sequelae of genital human papilomavirus infection in a man can include: a. anorectal carcinoma b. low sperm count c. paraphimosis d. Reiter syndrome

a. anorectal carcinoma

which of the following tests, if positive, is part of the criteria for systemic lupus erythematosus? a. antinuclear antibody (ANA) b. rheumatoid factor c. elevated liver function studies d. erythrocyte sedimentation rate (ESR)

a. antinuclear antibody (ANA)

SSRI withdrawal syndrome is best characterized as: a. bothersome but not life-threatening b. potentially life-threatening c. most often seen with dc of agents with a long half-life d. associated with seizure risk

a. bothersome but not life-threatening

which of the following best describes colonic diverticulosis? a. bulging pockets in the intestinal wall b. poorly contracting intestinal walls c. strictures of the intestinal lumen d. flaccidity of the small intestine

a. bulging pockets in the intestinal wall

you see a 28 y/o man who has been dx with moderate depression and has not responded well to SSRI txment over the past 3 months. he was involved in a motor vehicle accident 2 yrs ago that resulted in head trauma and now occasionally experiences occasional tonic clonic seizures. when considering alternative antidepressant tx, which of the following should be AVOIDED? a. bupropion b. trazodone c. citalopram d. duloxetine

a. bupropion

hyperthyroidism may affect the blood pressure: a. by producing an increase in systolic and diastolic readings b. by producing a decrease in diastolic blood pressure c. when the heart rate is increased d. with unpredictable results

a. by producing an increase in systolic and diastolic readings

a 76 y/o pt has fasting glucose values of 151 mg/dL and 138 mg/dL on different days. This pt: a. can be dx with type 2 diabetes b. has impaired fasting glucose c. should have a HgbA1c performed for dx d. has normal glucose values

a. can be dx with type 2 diabetes

commonly encountered dx other than acute appendicitis can include which of the following in a 28 y/o with a 2 day hx of lower abdominal pain and with rt sided pain slightly worse than left (more than one can apply) a. constipation b. PID c. ectopic pregnancy d. splenetic infarct

a. constipation b. PID c. ectopic pregnancy

a NP performs a fundoscopic exam. He identifies small areas of dull, yellowish-white coloration in the retina. What might these be? a. cotton wool spots b. microaneurysms c. hemorrhages d. exudates

a. cotton wool spots the most common causes of cotton wool spots are diabetes and high blood pressure

the mechanism of action of radioactive iodine in the tx of Graves' disease is to: a. destroy the overactive thyroid tissue b. reduce production of TSH c. alter thyroid metabolic rate d. relieve distress caused by increased thyroid size

a. destroy the overactive thyroid tissue

a female pt who is 45 yrs old states that she is having urinary frequency. She describes episodes of "having to go right now" and not being able to wait. Her urinalysis is normal. What is part of the differential? a. diabetes b. lupus c. stress incontinence d. asymptomatic bacteriuria

a. diabetes

the DSM5 criteria for substance use tolerance includes: a. diminished effect with the same amt of substance used b. desiring to get an amplified effect with higher doses c. ability to decrease the frequency of substance use d. absence of withdrawal sxs when substance is not used for a prolonged period

a. diminished effect with the same amt of substance used

Mr. M. is a frail, 87 y/o in your nursing home. he has a hx of multiple strokes, diabetes type 2, heart disease managed medically, and hyperthyroidism. he has a life expectancy of less than 10 yrs. he has no overt urinary sxs. his anxious daughter, Karen, is requesting prostate cancer screening. what would be your most appropriate response? a. discuss how a positive test would not change Mr. M' txment plan because his life expectancy is less than 10 yrs. Review the risk-benefit ratio of txment should cancer be found and discuss this together with Mr. M b. order a PSA test and perform a DRE c. send Mr. M for a CT scan d. send Mr. M for a MRI

a. discuss how a positive test would not change Mr. M' txment plan because his life expectancy is less than 10 yrs. review the risk- benefit ratio of txment should cancer be found and discuss this together with Mr. M

rectal bleeding associated with anal fissure is usually described by the pt as: a. drops of blood noticed when wiping b. dark brown to black in color and mixed in with normal- appearing stool c. a large amt of brisk red bleeding d. significant blood clots and mucus mixed with stool

a. drops of blood noticed when wiping

which of the following agents should be avoided in heavy alcohol users due to a potential risk for hepatotoxicity? a. duloxetine (cymbalta) b. desvenlafaxine c. escitalopram d. bupropion

a. duloxetine (cymbalta)

when considering an IBS dx, the NP should be aware that: a. dx is largely based on clinical presentation and application of the ROME III Criteria b. a colonscopy should be done routinely when the dx is suspected c. CBC, ESR, CRP and serum albumin should be the initial labs for an IBS workup d. once an IBS dx has been confirmed, you can assure the pt that tx is generally curative

a. dx is largely based on clinical presentation and application of the ROME III Criteria

which of the following is usually not seen in the dx of acute cholecystitis? a. elevated serum creatinine b. increased alkaline phosphatase level c. leukocytosis d. elevated aspartate aminotransferase level (AST)

a. elevated serum creatinine

lab findings in heatstroke usually include: a. elevated total creatine kinase b. anemia c. metabolic alkalosis d. hypokalemia

a. elevated total creatine kinase

a 75 y/o has pain from osteoarthritis in her rt knee. what intervention is considered first line to tx her pain? a. exercise b. acetaminophen c. ibuprofen d. tramadol

a. exercise

what is the most common adverse effect noted with alpha-glucosidase inhibitor use? a. gastrointestinal upset b. hepatotoxicity c. renal impairment d. symptomatic hypoglycemia

a. gastrointestinal upset ALPHA-GLUCOSIDASE INHIBITOR acarbose (Precose) miglitol (Glyset) [ACTION] Delays intestinal carbohydrate absorption by reducing postprandial digestion of starches and disaccharides via "enzyme action inhibitors". DOES NOT ENHANCE INSULIN SECRECTION OR SENSITIVITY [EFFECTIVENESS] A1C reduction of 0.3-0.9% w/ intensified use. ***TAKE WITH 1ST BITE OF MEAL*** ***helpful in mgmt of postprandial hyperglycemia***** [COST] ? [COMMON SE] ***GI upset!!!*** [SAFETY] NR [CONTRAINDICATIONS] -IBS -Impaired renal function

J. is a 47 y/o man who reports constant sadness following the death of his wife in a motor vehicle accident 2 weeks ago. he has not been able to function at work and avoids socializing with friends and family. you recommend: a. giving him time and support during this period of acute grief b. weekly psychotherapy sessions c. prescribing an anxiolytic to help with grief sxs d. psychotherapy plus a RX for an antidepressant

a. giving him time and support during this period of acute grief

you see a 48 y/o woman who has been taking a COS-2 inhibitor for the past 3 yrs. in counseling her, you mention that long-term use of COX-2 inhibitors is associated with all of the following EXCEPT: a. hepatic dysfunction b. gastropathy c. cardiovascular events d. cerebrovascular events

a. hepatic dysfunction

an example of an appropriate question to pose to a person with obesity who is in the precontemplation change stage is: a. how do you feel about your wt? b. what are barriers you see to losing wt? c. what is your personal goal for wt loss? d. how do you envision my helping you meet your wt loss goal?

a. how do you feel about your wt?

the gastric parietal cells produce: a. hydrochloric acid b. a protective mucosal layer c. prostaglandins d. prokinetic hormones

a. hydrochloric acid

which of the following is an unlikely consequence of untreated metabolic syndrome and insulin resistance in a woman of reproductive age? a. hyperovulation b. irregular menses c. acne d. hirsutism

a. hyperovulation

esophageal squamous cell cancer is usually located: a. in the upper esophagus b. near the upper esophageal sphincter c. at the junction of the esophagus and stomach d. in the lower esophagus

a. in the upper esophagus

niacin is known to: a. increase fasting glucose levels b. produce hypertension c. decrease triglycerides d. decrease HDLs

a. increase fasting glucose levels

J and Jane have been married for 6 months and are unable to conceive. they ask you to recommend an infertility specialist. what do you tell them? a. infertility is not an issue until you have had unprotected sex for at least 1 yr b. let's run some routine tests first; then I'll recommend someone c. tell me about your sexual experiences d. it's usually a problem with the woman, so let's have Jane examined first

a. infertility is not an issue until you have had unprotected sex for at least 1 yr

Hesselbach's triangle forms the landmark for: a. inguinal hernia b. femoral hernia c. abdominal hernia d. umbilical hernia

a. inguinal hernia

Mr. S. comes to you with scrotal pain. the exam of his scrotum, penis, and rectum are normal. which of the following conditions outside of the scrotum may present as scrotal pain? a. inguinal herniation and peritonitis b. renal colic and cardiac ischemia c. pancreatitis and Crohn's disease d. polyarteritis nodosa and ulcerative colitis

a. inguinal herniation and peritonitis

which of the following statements best describes the Somogyi effect? a. insulin-induced hypoglycemia triggers excess secretion of glucagon and cortisol, leading to hyperglycemia b. early morning elevated blood glucose levels result in part from growth hormone and cortisol-triggering hepatic glucose release c. late evening hyperglycemia is induced by inadequate insulin dose d. episodes of postprandial hypoglycemia occur as a result of inadequate food intake

a. insulin-induced hypoglycemia triggers excess secretion of glucagon and cortisol, leading to hyperglycemia

a common cause of in-toeing in childhood is: a. internal tibial torsion b. femoral retroversion c. external tibial torsion d. flat feet

a. internal tibial torsion

colchicine may be used to terminate an acute attack of gouty arthritis, as well as to prevent recurrent episodes. the mechanism of action is to: a. interrupt the cycle of urate crystal deposition and inflammatory response b. increase serum uric acid levels c. potentiate the excretion of uric acid d. inhibit the tubular reabsorption of urate,promoting the excretion of uric acid

a. interrupt the cycle of urate crystal deposition and inflammatory response

indicate whether each finding would be present in acute epididymitis (yes or no) a. irritative voiding sxs - yes b. irritative voiding sxs - no

a. irritative voiding sxs - yes

a hx of UTI in males is often seen in men with chronic bacterial prostatitis. other s/s of chronic bacterial prostatitis include: a. irritative voiding sxs, low back pain, and perineal pain. b. nausea and vomiting as well as fever c. loss of appetite and wt loss d. irritative voiding sxs,inability to ambulate, and fever

a. irritative voiding sxs, low back pain, and perineal pain.

which of the following statements concerning developmental dysplasia of the hip (DDH) is correct? a. it is often associated with being the 1st born female child b. it results from an orthopedic malformation in utero c. it has no genetic predisposition d. it is more common in males

a. it is often associated with being the first born female child

which of the following statements is most accurate concerning rheumatoid arthritis? a. joint erosions are often evident on radiographs or MRI b. rheumatoid arthritis is seldom associated with other autoimmune diseases c. a butterfly-shaped facial rash is common d. parvovirus B19 infection can contribute to its development

a. joint erosions are often evident on radiographs or MRI

after use, the onset of action of lispro (Humalog) occurs in: a. less than 30 min b. approximately 1 hr c. 1-2 hrs d. 3-4 hrs

a. less than 30 min

according to DSM-5, a dx of depression must include either depressed mood or which of the following? a. loss of interest or pleasure b. recurrent thoughts of death c. feelings of worthlessness d. weight change (either increase or decrease)

a. loss of interest or pleasure

Major risk factors for diverticulosis include all the following except: a. low fiber diet b. family hx of condition c. older age d. select connective tissue disorders (Marfan Syndrome)

a. low fiber diet

which of the following is a common finding in a man with varicocele? a. lower sperm count with increased number of abnormal forms b. increased rate of testicular cancer b. recurrent scrotal pain d. BPH

a. lower sperm count with increased number of abnormal forms

which of the following characteristics applies to type 2 diabetes? a. major risk factors are heredity and obesity b. pear-shaped body type is commonly found c. exogenous insulin is needed for control of disease d. physical activity enhances insulin resistance

a. major risk factors are heredity and obesity

a 70 y/o African American male c/o pain in his back and trun. Cardiovascular disease is ruled out. he has a normocytic normochromic anemia with hypercalcemia. a likely dx is: a. multiple myeloma b. lymphoma c. leukemia d. prostate cancer

a. multiple myeloma

which of the follow are possible consequences of obesity? (more than one can apply) a. obstructive apnea b. steathepatitis c. female inferitlity d. endometrial cancer

a. obstructive apnea b. steathepatitis c. female inferitlity d. endometrial cancer

first-line tx for Crohn's disease or ulcerative colitis is: a. oral aminosalicylates b. parenteral corticosteroids c. antibiotics d. immune modulators

a. oral aminosalicylates

carpopedal spasm is: a. painful contraction of the hands or feet b. painful flexion of the hands or feet c. spasm of the car pool

a. painful contraction of the hands or feet pg 830

which of the following statements concerning panic d/o is false? a. panic d/o rarely occurs with depression b. up to 4% of the general population suffers from panic d/o c. new onset panic d/o rarely occurs after 45 years of age d. family hx of panic d/o is a risk factor for the condition

a. panic d/o rarely occurs with depression

the psoas sign can be best described as abdominal pain elicited by: a. passive extension of the hip b. passive flexion and internal rotation of the hip c. deep palpation d. asking the pt to cough

a. passive extension of the hip

you are performing a school PE on D. age 5. you are unable to retract his foreskin over the glans penis while inspecting his penis. this is referred to as: a. phimosis b. paraphimosis c. microphallus d. priapism

a. phimosis

secondary adrenal insufficiency can occur with the presence of a diseased or malfunctioning: a. pituitary gland b. thyroid c. pancreatic beta cells d. hypothalamus

a. pituitary gland

what is the earliest detectable glycemic abnormality in a pt with type 2 diabetes? a. postprandial glucose elevation b. nighttime hyperglycemia c. fasting glucose elevation d. abnormal Hgb A1C

a. postprandial glucose elevation

a 63 y/o presents to you with hematuria, hesitancy, and dribbling. DRE reveals a moderately enlarged prostate that is smooth. the clt's PSA is 1.2. what is the most appropriate management strategy for you to follow at this time? a. prescribe an alpha adrenergic blocker b. recommend saw palmetto c. prescribe an antibiotic d. refer the client to urology

a. prescribe an alpha adrenergic blocker

a mammogram in a healthy 40 years female pt is considered to be an example of: a. primary prevention b. secondary prevention c. teritiary prevention d. quaternary prevention

a. primary prevention

which of the following medications is a helpful txment option for relief of tremor and tachycardia seen with untxed hyperthyroidism? a. propranolol b. diazepam c. carbamazepine d. verapamil

a. propranolol

low-grade, localized prostate cancer can be txed successfully with: a. radical prostatectomy or radiation b. chemotherapy c. cryosurgery (freezing) of a small part of the gland d. watchful waiting

a. radical prostatectomy or radiation

which of the following is most likely to be found in a 50 y/o woman with new-onset reflux esophagitis? a. recent initiation of estrogen-progestin hormonal tx b. recent wt loss c. report of melena d. evidence of H. pylori infection

a. recent initiation of estrogen-progestin hormonal tx

undiagnosed diabetes may present as: a. recurrent vaginal candidiasis b. chronic halitosis c. varicose veins d. paresthesias in the upper extremities

a. recurrent vaginal candidiasis

clinical presentation of type 1 diabetes usually includes all of the following EXCEPT: a. report of recent unintended wt gain b. ketosis c. thirst d. polyphagia

a. report of recent unintended wt gain

untreated Cushing's syndrome can lead to all of the following EXCEPT: a. rheumatoid arthritis b. hypertension c. type 2 diabetes d. osteoporosis

a. rheumatoid arthritis

the use of which of the following meds is often associated with weight gain? a. risperidone (risperdal) b. topiramate (topamax) c. metformin (glucophage) d. sitagliptin (Januvia)

a. risperidone (risperdal)

Murphy's sign can be best described as abdominal pain elicited by: a. rt upper quadrant abdominal palpation b. asking the pt to stand on tiptoes and then letting body wt fall quickly onto the heels c. asking the pt to cough d. percussion

a. rt upper quadrant abdominal palpation

which of the following characteristics applies to type 1 diabetes? a. significant hyperglycemia and ketoacidosis result from lack of insulin b. this condition is commonly dx on routine exam or workup for other health problems c. initial response to oral sulfonylureas is usually favorable d. insulin resistance is a significant part of the disease

a. significant hyperglycemia and ketoacidosis result from lack of insulin

which of the following characteristics applies to type 1 diabetes? a. significant hyperglycemia and ketoacidosis result from lack of insulin b. this condition is commonly dx on routine exam or workup for other health problems c. initial response to oral sulfonylureas is usually favorable d. insulin resistance is a significant part of the disease

a. significant hyperglycemia and ketoacidosis result from lack of insulin

which of the following daily doses has the lowest lipid-lowering effect? a. simavastin 10mg b. rosuvastin 5mg c. atorvastin 10mg d. pravastatin 40mg

a. simavastin 10mg

to confirm the results of a McMurray test, you ask the pt to: a. squat b. walk c. flex the knee d. rotate the ankle

a. squat

recommended exercises for pts with OA of the knee include all of the following except: a. squatting with light weights b. straight-leg raises without weights c. quadriceps sets d. limited weight-bearing aerobic exercises

a. squatting with light weights

rectal bleeding associated with hemorrhoids is usually described as: a. streaks of bright red blood on the stool b. dark brown to black in color and mixed in with normal - appearing stool c. a large amt of brisk red bleeding d. significant blood clots and mucus mixed with stoola. streaks of bright red blood on the stool

a. streaks of bright red blood on the stool

cyclooxygenase-2 (COX-2) contributes to: a. the inflammatory response b. pain transmission c. maintenance of gastric protective mucosal layer d. renal arteriole constriction

a. the inflammatory response

you see a 72 y/o woman who reports vomiting and abdominal cramping occurring over the past 24 hrs. In evaluating a pt with suspected appendicitis, the clinician considers that: a. the presentation can differ according to the anatomical location of the appendix b. this is a common reason for acute abdominal pain in elderly pts c. vomiting before onset of abdominal pain is often seen d. the presentation is markedly different from the presentation of pelvic inflammatory disease

a. the presentation can differ according to the anatomical location of the appendix

you see a 72 y/o woman who reports vomiting and abdominal cramping occurring over the past 24 hrs. In evaluating a pt with suspected appendicitis, the clinician considers that: a. the presentation can differ according to the anatomical location of the appendix b. this is a common reason for acute abdominal pain in elderly pts c. vomiting before onset of abdominal pain is often seen d. the presentation is markedly different from the presentation of pelvic inflammatory disease

a. the presentation can differ according to the anatomical location of the appendix Can be in the epigastrium, flank or groin,

one week into sertraline (zoloft) txment, a pt c/o new onset recurrent dull frontal headache that is relieved promptly with acetaminophen. which of the following is true in this situation? a. this is a common, transient side effect of SSRIs b. she should dc the med c. fluoxetine should be substituted d. desipramine should be added

a. this is a common, transient side effect of SSRIs

T. the mother of a 2 y/o is concerned because her daughter walks on her toes all the time. what do you tell her? a. toe walking is consider normal until age 3 b. don't worry, she will outgrow it c. toe walking is normal until she starts kindergarten d. we should do further testing now

a. toe walking is consider normal until age 3

which of the following would enable you to r/o a dx of testicular cancer when examining a client? a. transillumination of the suspected mass b. white race c. Scandinavian background d. hx of cryptorchidism

a. transillumination of the suspected mass

A 64-year-old male with type 2 diabetes comes to the clinic with c/o "my feet feel like they are on fire." He has loss of vibratory sense, +1 Achilles reflex, and a tack embedded in his left heel. Which of the following would be an appropriate treatment? a. tricyclic antidepressants b. capsacin cream c. vitamin B12 injections d. insulin

a. tricyclic antidepressants pg 875

the most common type of injury causing a sport-related meniscal tear involves: a. twisting of the knee b. hyperextension of the knee c. repetitive hard impact on the knee (i.e. running on hard surface) d. an unknown origin in most cases

a. twisting of the knee

which of the following imaging studies potentially exposes the pt being evaluated for abdominal pain to the lowest ionizing radiation burden? a. ultrasound b. barium enema c. CT scan d. abdominal flat plate

a. ultrasound

an example of primary prevention is: a. using a condom to prevent infection with an STD b. dx of chlamydia prior to sx development c. tx of chlamydia concurrently with gonorrhea d. early txment of sexual partners

a. using a condom to prevent infection with an STD

common physical findings of SLE include all of the following except: a. weight gain b. joint pain and swelling c. fatigue d. facial rash

a. weight gain

the risk of HIV transmission is increased: a. when other STDs are present b. in females c. when pts are aware of their HIV status d. in pts with diabetes

a. when other STDs are present

changes to the joint during osteoarthritis can typically include all of the following except: a. widening of the joint space b. articular cartilage wears away c. formation of bone spurs d. synovial membrane thickens

a. widening of the joint space

a pt has suspected plantar fasciitis. the plantar fascia is best examined: a. with the great toe dorsiflexed b. with the foot in neutral position c. while the pt stands d. with the ankle at a 90 degree angle

a. with the great toe dorsiflexed

irritable bowel syndrome is characterized by all of the following EXCEPT: a. wt loss and malnutrition b. abdominal pain or discomfort (hypersensitivity) at least 3 times per month for a 6 month period c. altered bowel pattern in the absence of detected structural abnormalities d. occurs two to three times more often in women than men

a. wt loss and malnutrition

Individuals of __________ ancestry are more likely to have Sarcoidosis than white Americans and tend to have more severe disease that can cause pulmonary issues. Peak age is

african 20-40

Celiac disease leaves you intolerant to

allergy to gluten: unable to digest insoluble protein - wheat (gliadin), barley (hordein), rye (secalin) - presence of gluten in small intestine causes inflammation

identify if the following characteristics are noted in anorexia nervosa, bulimia nervosa or both disorder: lanugo

anorexia nervosa,

The etiology of graves disease is autoimmune production of thyroid stimulating ______________.

antibodies

chondroitin is generally well tolerated, it should be used with caution because of a potential __________ effect.

anticoagulant

Alternatives to HRT for tx of hot flashes

antidepressants gabapentin clonidine CAM: black cohosh, soy, red clover, vitamin E, flaxseed (no better than placebo in studies)

S. aureus accounts for most cases of bullous impetigo and for a substantial portion of nonbullous. What is the 1st line treatment? What is the second line treatment?

antimicrobials with gram (+) coverage and are stable in the presence of beta-lactamase. 1st line: 1st gen cephalosporin -cephalexin (Keflex) 2nd gen cephalosporin- cefuroxime axetil (Ceftin) cefprozil (Cefzil) cefaclor (Ceclor) 2nd line: PCN-dicloxacillin

what are the three tx options for Graves' disease?

antithyroid drugs, radioactive iodine, and surgery pg 842

with an 8am dose of the following insulin forms, followed by an inadequate dietary intake and/or excessive energy use, at approximately what time would hypoglycemia be most likely to occur with Lispro?

anywhere from 0830 to 0930 peak time for Lispro is 30min to 2.5hrs ??? i thought it should be anywhere from 0830-1030 -test is multiple choice so oh well

When listening to heart sounds, the nurse knows that the valve closures that can be heart best at the base of the heart are:

aortic and pulmonic

RLQ guarding with rebound tenderness, Psoas sign, Obtruator sign, rosving's sign, low grade fever (high fever suggests perf.)

appendicitis

Obturator sign evaluates for

appendicitis used for acute appendicitis or any suspected retroperitoneal area acute process. rotate right hip through full range of motion. Positive sign if pain with the movement or flexion of the hip

Phytoestrogens

apples, carrots, coffee, potatoes, yams, soy products, flaxseed, ginseng, bean sprouts, red clover sprouts, sunflower seeds, rye, wheat, sesame seeds, linseed, black cohosh, and bourbon

PDE-5 inhibitors (sildenafil, vardenafil, avanafil, and tadalafil) should be taken when?

approximately one hr before the anticipated sexual activity they wk by enhancing the effects of nitric oxide, a chemical that relaxes the smooth muscles int he penis during sexual stimulation and allows increased blood flow

Reactive arthritis (Reiter's syndrome)

asymmetric arthritis that involves large joints below the waist (knee and ankle), mucocutaneous lesions (balanitis, stomatitis), urethritis and conjunctivitis Dx of arthritis which manifests within 2 month of a bacterial gastroenteritis or nongonococcal urethritis/cervicitis (most commonly C. trachomatis)

gynecomastia lasting longer than 1 year is usually:

asymptomatic

When do you incorporate HPV with Pap? When do you repeat? Then how often for pap? Stop when? Any stipulation?

at age 30 repeat in 5 years if both negative then pap every 3 years Stop at age 65= if they have 10 yr negative history

anticipated organ survival exceeds 85% with testicular decompression within how many hrs or torsion? a. 1 b. 6 c. 16 d. 24

b. 6

which HPV are most likely to cause condyloma acuminatum? a. 1, 2, and 3 b. 6 and 11 c. 16 and 19 d. 22 and 24

b. 6 and 11

C., age 62, has swollen, bony proximal interphalangeal joints. you describe these as: a. Heberden's nodes b. Bouchard's nodes c. Osler's nodes d. Murphy's nodes

b. Bouchard's nodes

Which of the following is a sign of hypothyroidism? a. Warm, smooth, moist skin. b. Brittle hair. c. Gynecomastia. d. A thyroid bruit.

b. Brittle hair.

an 18 y/o man presents with periumbilical pain, vomiting, and abdominal cramping over the past 48 hrs. PE reveals rebound tenderness and lab analysis shows the presence of bandemia and a total WBC of 28,000 mm3. to support the dx of acute appendicitis with suspected appendiceal rupture, you consider obtaining the following abdominal imaging study: a. MRI b. CT scan c. ultrasound d. flat plate

b. CT scan

Mr. YM, 55 years old, is seen in the clinic with concerns about his left foot. He has a 40-year history of type 1 diabetes with "fairly good" control on twice-daily insulin. He denies injury but states that he tripped a few months previous and that his foot is sore when he walks. Physical examination reveals an edematous, erythemic, and warm foot. There is a superficial ulcer on the plantar surface. Which of the following is the most likely diagnosis? a. fallen arch b. Charcot joint c. arthritis d. sprained ankle

b. Charcot joint pg 875 can be confused with cellulitis Disease in joint due to nerve damage thus joint is unable to respond appropriately by adjusting to increased force on joint, person is unable to perceive pain coming from the joint 2. Can lead to joint deformity from repeated injury

in a 28 y/o man who presents with a 6 month hx of involuntary wt loss, recurrent abdominal cramping, loose stools, and anterior and posterior anal fissure, which of the following dx should be considered? a. ulcerative colitis b. Crohn's disease c. C. difficile colitis d. condyloma acuminata

b. Crohn's disease

a 46 y/o woman c/o fatigue, weakness, lethargy, decreased concentration and memory, and increased facial hair over the past 2 months. She also reports gaining over 30 pounds in the past 2 months. She has a hx of asthma with repeated flares during the past 6 months requiring multiple courses of prednisone tx. A likely dx for this pt is: a. type 2 diabetes b. Cushing's syndrome c. Cushing's disease d. central obesity

b. Cushing's syndrome Central obesity, moon face, buffalo hump, acne, purple striae, hirsutism, amenorrhea, poor wound healing, frequent infections, labile mood, *hypertension, hyperglycemia

a 75 y/o female, who knits daily, has a positive Finkelstein test. what is her likely dx? a. Gamekeeper's thumb b. DeQuervain's tenosynovitis c. osteoarthritis of the thumb d. trigger thumb

b. DeQuervain's tenosynovitis

which of the following men is more prone to prostatitis? a. Jim, age 52, who wears tight jeans and sits at a computer all day b. Jerry, age 30 who is cross-country runner c. Marvin, age 46, who is a dog trainer d. Justin, age 39 who is a boat salesman

b. Jerry, age 30 who is a cross-country runner

you are caring for a pt that has a hx of psoriasis and now is showing signs of musculoskeletal s/s with joint involvement. seropositivity provides a definitive dx of psoriatic arthritis (PsA). your initial tx choice for management of the pt is: a. disease modifying antirheumtic drugs (DMARDs) b. NSAIDs c. tumor necrosis factor-alpha inhibitors (TNF-alphal inhibitor) d. uricosuric

b. NSAIDs

ACE inhibitors are given to clients with diabetes who have a. Insulin sensitivity. b. Persistent proteinuria. c. An elevated serum creatinine level. d. An elevated glycohemoglobin level.

b. Persistent proteinuria. quiz 624 kidney protection mechanism

the mechanism of action of sulfonylureas is as: a. an antagonist of insulin receptor site activity b. a product that enhances insulin release c. a facilitator of renal glucose excretion d. an agent that can reduce hepatic glucose production

b. a product that enhances insulin release SULFONYLUREAS glipizide (Glucotrol) glyburide (Diabeta) glimepride (Amary) best one ^most potent [ACTION] insulin secretagogue (enhance release) [IDEAL FOR] -non-obese/mild obesity -strict budget [EFFECTIVENESS] -A1C reduction of 1-2% with intensified use. -Typically less effective after 5 yrs of use. (b/c they require functioning beta cells) [ADVANTAGES] -Cheap [DISADVANTGES] - hypogylcemia -tend to cause weight gain [COST] -$4 drug list [SAFETY] -can cause severe hyperglycemia, especially in older adults -d/c if patient goes on insulin

with the use of ezetimibe (zetia), the NP expects to see: a. a marked increase in HDL cholesterol b. a reduction in LDL cholesterol c. a significant reduction in triglyceride levels d. increased rhabdomyolysis when the drug is used in conjunction with HMG-CoA reductase inhibitor

b. a reduction in LDL cholesterol

tenesmus is defined as which of the following? a. rectal burning with defecation b. a sensation of incomplete bowel emptying that is distressing and sometimes painful c. wt loss that accompanies many bowel diseasesd. appearance of frank blood in the stool

b. a sensation of incomplete bowel emptying that is distressing and sometimes painful

what medication is recommended by American College of Rheumatology as a first line agent for a pt who has been unsuccessful with non-pharmacologic interventions for osteoarthritis pain? a. naproxen b. acetaminophen c. ibuprofen d. tramadol

b. acetaminophen

a 43 y/o woman has a 12 hr hx of sudden onset of rt upper quadrant abdominal pain with radiation to the shoulder, fever, and chills. She has had similar, milder episodes in the past. Exam reveals marked tenderness to rt upper quadrant abdominal palpation. Her most likely dx is: a. hepatoma b. acute cholecystitis c. acute hepatitis d. cholelithiasis

b. acute cholecystitis

the most common form of esophageal cancer in the US is: a. squamous cell cancer b. adenocarcinoma c. basal cell carcinoma d. melanoma

b. adenocarcinoma

in collection of a specimen for a PAP smear., how is the endocervical specimen collected? a. after the ectocervical specimen with a broom b. after the ectocervical specimen with a brush c. before the ectocervical specimen with a broom d. before the ectocervical specimen with a brush

b. after the ectocervical specimen with a brush

which of the following is least likely to be found in a pt with gastric ulcer? a. hx of long-term naproxen use b. age younger than 50 yrs c. previous use of histamine2-receptorantagonist d. cigarette smoking

b. age younger than 50 yrs

when providing care for a pt taking an HMG-CoA reductase inhibitor, initial evaluation when starting medication includes checking which of the following serological parameters? a. potassium b. alanine aminotransferase c. bilirubin d. alkaline phosphatase

b. alanine aminotransferase

which of the following is a mineralocorticoid? a. cortisol b. aldosterone c. insulin d. hydrocortisone

b. aldosterone

antiprostaglandin drugs cause stomach mucosal injury primarily by: a. a direct irritative effect b. altering the thickness of the protective mucosal layer c. decreasing peristalsis d. modifying stomach pH level

b. altering the thickness of the protective mucosal layer

you perform a DRE on a 72 y/o and find a lesion suspicious for prostate cancer. the findings are described as: a. rubbery, enlarged prostatic lobe b. an area of prostatic induration c. an indurated gland d. prostatic tenderness

b. an area of prostatic induration

a pt has been dx with Grave's disease. He is likely to have: a. an elevated alkaline phosphatase b. an elevated T3 or T4 c. an elevated TSH d. elevated liver function studies

b. an elevated T3 or T4

a pt who has been txed for hypothyroidism presents for her annual exam. Her TSH is 14.1 (normal =0.4 -3.8). She c/o weight gain and fatigue. How should the nP proceed? a. ask what time of day she is taking her medication b. ask if she is taking her medication c. increase her dose of thyroid supplement d. repeat the TSH in 2-3 months

b. ask if she is taking her medication

J., age 16 was involved in a motor vehicle accident. he walks into the office with an obvious facial fx, then collapses. what should your first action be? a. call his parents for permission to tx b. assess for an adequate airway c. obtain head and maxillofacial Ct d. assess for a septal hematoma

b. assess for an adequate airway

hemoglobin A1c should be tested: a. at least annually for all pts b. at least two times a yr in pts who are meeting tx goals and who have stable glycemic control c. monthly in pts whose therapy has changed or who are not meeting glycemic goals d. only via standardized lab testing because of inaccuracies associated with point-of-service testing

b. at least two times a yr in pts who are meeting tx goals and who have stable glycemic control *quarterly in those who are not at goal/requiring medication adjustments.

which of the following is an example of moderate-intensity statin tx? a. fluvastatin 10mg b. atorvastin 10mg c. simvastin 10mg d. pravastatin 20mg

b. atorvastin 10mg

a 65 y/o pt has a firm, non-tender, symmetrical enlarged prostate gland on exam. his PSA is 3.9ng/mL. this probably indicates: a. prostate cancer b. benign prostatic hypertrophy c. prostate infection d. a perfectly normal prostate gland

b. benign prostatic hypertrophy

a dx of osteoporosis can be made when: a. an x-ray indicate gragility fractures b. bone mineral density is 2.5 standard deviations below the mean c. T-score greater than 2.5 d. DXA scan indicates fragility fracture index

b. bone mineral density is 2.5 standard deviations below the mean

a dx of osteoporosis can be made when: a. an x-ray indicates fragility fractures b. bone mineral density is 2/5 standard deviations below the mean c. T-score greater than 2.5 d. DXA scan indicates fragility fracture index

b. bone mineral density is 2/5 standard

G. is c/o a scrotal mass; however, the scrotum is so edematous that it is difficult to assess. how do you determine if it is a hernia or a hydrocele? a. you can always return a hernia's contents to the abdominal cavity b. bowel sounds may be heard over a hernia c. you can transilluminate a hernia d. with a hydrocele, a bulge appears on straining

b. bowel sounds may be heard over a hernia

an adverse effect associated with the use of glucosamine is: a. elevated ALT and AST b. bronchospasm c. increased bleeding risk d. QT prolongation

b. bronchospasm

an alternative to methadone that can be used to curb opioid withdrawal sxs is the use of: a. gabapentin b. buprenorphine plus naloxone c. methylnatrexone d. topiramate

b. buprenorphine plus naloxone

which of the following characteristics is predictive of severity of chronic liver disease in a pt w/ chronic hepatitis C? a. female gender, age younger than 30 b. co-infection with Hepatitis B, daily alcohol use c. acquisition of virus through IV drug use, hx of hepatitis A infection d. frequent use of aspirin, nutritional status

b. co-infection with Hepatitis B, daily alcohol use

Mr. Smith, an overweight 48 y/o male with undiagnosed type 2 diabetes presents to your clinic. which sx is least likely associated with type 2 diabetes? a. fatigue b. constipation c. athlete's foot d. impetigo

b. constipation

in general, pharmacological intervention for pts with depression should: a. be given for about 4months on average b. continue for a minimum of 6 months after remission is achieved c. be cont indefinitely with a first episode of depression d. be titrated to a lower dose after sx relief is achieved

b. continue for a minimum of 6 months after remission is achieved

you are assessing M., age 16 after a football injuryt to his rt knee. you elicit a positive anterior/posterior drawer sign. this test indicates an injury to the: a. lateral meniscus b. cruciate ligament c. medial meniscus d. collateral ligament

b. cruciate ligament

the anticipated effect on the lipid profile with plant stanol and sterol use includes: a. increase in HDL b. decrease in LDL c. decrease in select lipoprotein subfractions d. decrease in triglycerides

b. decrease in LDL

A 25 y/o male pt with subacute bacterial epididymitis should be txed initially with: a. an oral quinolone b. doxycycline c. anti-inflammatories and analgesics d. ice and scrotal support

b. doxycycline

a 65 y/o pt presents to your clinic with evidence of hyperthyroidism. In assessing her cardiovascular status, what should the NP assess immediately? a. cardiac enzymes b. electrocardiogram c. electroyltes d. auscultation of systolic murmurs

b. electrocardiogram, an urgent threat is the possibility of stroke from atrial fibrillation

when a patient is exhibiting either Trousseau's or Chvostek's signs and accompanied by respiratory distress(stridor, loud crowing noises and cyanosis), what type of care do they need? a. home care b. emergency care c. ibuprofen d. rest

b. emergency care

you see a 62 y.o man dx with esophageal columnar epithelial metaplasia. you realize he is at increased risk for: a. esophageal stricture b. esophageal adenocarcinoma c. gastroesophageal reflux d. H. pylori colonization

b. esophageal adenocarcinoma

according to the American Cancer Society recommendations, which of the following is the preferred method for ANNUAL colorectal cancer screening in a 51 y/o man? a. digital rectal exam b. fecal occult blood test c. colonoscopy d. barium enema study

b. fecal occult blood test

a 64 y/o woman presents with a 3 month hx of upper abdominal pain. she describes the discomfort as an intermittent, centrally located burning feeling in the upper abdomen, most often with meals and often accompanied by mild nausea: use of an OTC histamine2-receptor antagonist affords partial sx relief. she also uses diclofenac on a regular basis for the control of osteoarthritis pain. her clinical presentation is most consistent with: a. acute gastroenteritis b. gastric ulcer c. duodenal ulcer d. chronic cholecystitis

b. gastric ulcer

which of the following medications should be used with caution in a person with suspected or known sulfa allergy? a. metformin b. glyburide c. rosiglitazone d. NPH insulin

b. glyburide

an adolescent athlete has injured his ankle playing basketball. he has rt ankle pain, ecchymosis, significant edema, and he is unable to bear wt at the time of the clinical exam. which dx is least likely? a. an avulsion fx b. grade I sprain c. grande II sprain d. grade III sprain

b. grade I sprain

which of the following hepatitis forms is most effectively transmitted from the man to the woman via heterosexual vaginal intercourse? a. hepatitis A b. hepatitis B c. hepatitis C d. hepatitis D

b. hepatitis B

a pt has fatigue, weight loss, and a TSH of .05. What is his likely diagnosis? a. hypothyroidism b. hyperthyroidism c. subclinical hypothyroidism d. more tests are needed to establish his dx

b. hyperthyroidism

G., age 26, runs marathons and frequently c/o painful contractions of his calf muscles after running. you attribute this to: a. hypercalcemia b. hyponatremia c. heat exhaustion d. dehydration

b. hyponatremia

to prevent an outbreak of hepatitis D infection, the NP plans to: a. promote a campaign for clean food supplies b. immunize the population against hepatitis B c. offer antiviral prophylaxis against the agent d. encourage frequent hand washing

b. immunize the population against hepatitis B

Mr. Sam is a 58 y/o man with type 2 diabetes who is using a single 10 unit daily dose of the long-acting insulin glargine. His fasting blood glucose has been between 141-180 mg/dL. Which of the following best describes the next step in his tx? a. continue on the currentglargine dose b. increase his glargine dose by 4 units per day c. increase his glargine dose by 1 unit per day d. increase his glargine dose by 6 units per day

b. increase his glargine dose by 4 units per day

when prescribing a fibrate, the NP expects to see which of the following changes in lipid profile? a. marked decrease in LDL level b. increase in HDL level c. no effect on triglyceride level d. increase in very low density lipoprotein level

b. increase in HDL level

when prescribing a niacin, the NP expects to see which of the following changes in lipid profile? a. marked decrease in LDL level b. increase in HDL level c. no effect on triglyceride level d. increase in very low density lipoprotein level

b. increase in HDL level

in caring for a pt with symptomatic gastroensophageal reflux, you prescribe a PPI to: a. enhance motility b. increase the pH of the stomach c. reduce lower esophageal pressure d. help limit H. pylori growth

b. increase the pH of the stomach

which of the following statements about Helicobacter. pylori is false? a. H. pylori is a gram-negative, spiral shaped bacterium b. infection with H. pylori is the most potent risk factor for duodenal ulcer c. the organism is often resistant due to the production of beta-lactamase d. H. pylori is transmitted via the oral-fecal or oral-oral route

b. infection with H. pylori is the most potent risk factor for duodenal ulcer

a 22 y/o male who is otherwise healthy c/o scrotal pain. his pain has developed over the past 4 days. he is dx with epididymitis. what is the most likely reason? a. his age b. infection with chlamydia c. underlying hydrocele d. urinary tract infection

b. infection with chlamydia

a 50 y/o male pt reports that he has a sensation of scrotal heaviness. he reports that the sensation is worse at the end of the day. he denies pain. what is the likely etiology of these sxs? a. strangulated hernia b. inguinal hernia c. epididymitis d. hydrocele

b. inguinal hernia

what does a positive anterior drawer test demonstrate with an injured knee? a. injury to the lateral meniscus b. instability of the anterior cruciate ligament (ACL) c. stability of the posterior cruciate ligament (PCL) d. stability of the lateral knee

b. instability of the anterior cruciate ligament

Mr. Jones, a pt with type 2 diabetes, brings his obese 15 y/o son in to see the NP. You examine the 15 y/o son and identify acanthosis nigricans. This probably indicates: a. undiagnosed diabetes b. insulin resistance c. familial skin changes d. poor hygiene

b. insulin resistance

after a decade of disease, a person with ulcerative colitis is at increased risk of malignancy involving the: a. small bowel b. large intestine c. duodenum d. stomach

b. large intestine

the location of discomfort with acute diverticulitis is usually in which of the following areas of the abdomen? a. epigastrium b. left lower quadrant c. right lower quadrant d. suprapubic

b. left lower quadrant

the hormone cortisol plays a role in all of the following processes EXCEPT: a. maintaining glucose control b. maintenance of thyroid function c. suppressing the immune response d. helping the body respond to stress

b. maintenance of thyroid function

risk factors for the development of cholelithiasis include all of the following except: a. rapid wt loss b. male gender c. obesity d. Native American ancestry

b. male gender

a 70 y/o male presents to your clinic with a lump in his breast. How should this be evaluated? a. palpation and ultrasound b. mammogram and ultrasound c. ultrasound only d. mammogram only

b. mammogram and ultrasound

Mrs. S. is a 42 y/o mother of 3, who reports pain and stiffness in multiple joints that have lasted for more than 6 months. she is dx with rheumatoid arthritis. she has no other clinical conditions of significance. you recommend which of the following txments for first-line tx? a. topical analgesics and oral NSAIDs b. methotrexate plus oral NSAIDs c. acetaminophen plus leflunomide d. anakinra and systemic corticosteroids

b. methotrexate plus oral NSAIDs

an example of a medication with prokinetic activity is: a. dicyclomine b. metoclopramine c. loperamide d. psyllium

b. metoclopramine

the most appropriate screen for diabetic nephropathy is: a. creatinine clearance b. microalbuminuria c. BUN/Cr d. serum creatinine

b. microalbuminuria

a patient who is 5 feet tall and weighs 150 pounds (29.3kg/m2). How would the healthcare provider classify this patient? a. overweight b. mild obese c. moderate obese d. morbid obese

b. mild obese

which of the following meds is a prostaglandin analog? a. sucralfate b. misoprostol c. esomeprazole d. metoclopramide

b. misoprostol

a 78 y/o woman has HTN, a 100 pp/year hx of cigarette smoking, peripheral vascular disease, and reduced renal function (GFR equal to 47 mL/min/1.73m2) Triglycerides level is 280 mg/dL; HDL is 48 mg/dL; and LDL is 135 mg/dL. Which of the following represents the most appropriate pharmacological intervention for this pt's lipid disorders? a. due to her age and comorbidity, no further intervention is required b. moderate intensity statin tx is the preferred tx option c. a resin should be prescribed d. the use of ezetimibe (zetia) will likely be sufficient to achieve dyslipidemia control

b. moderate intensity statin tx is the preferred tx option

a 38 y/o male pt, thought to be in good health, presents to a primary care clinic. On routine exam, the pt's fasting blood sugar is 242 mg/dL. A repeat value after eating is 288 mg/dL. Which fo the following is least helpful in the initial evaluation of this patient? a. blood pressure b. non-fasting lipids c. hgb A1c d. microalbuminuria

b. non-fasting lipids

a college age basketball player landed awkwardly on his foot and ankle after jumping during a game yesterday. he states that he sprained his ankle. he c/o ankle pain and foot pain but is able to limp into the exam room. how should he be managed? a. rest, ice, compression, elevation b. non-wt bearing until fx is ruled out c. short leg splint d. NSAIDs and rest with partial non-wt bearing

b. non-wt bearing until fx is ruled out

the International Diabetes Federation's dx criteria for metabolic syndrome include: a. an obligatory finding of persistent hyperglycemia b. noted ethnic-specific waist circumference measurements c. documentation of microalbuminuria d. a family hx of type 2 diabetes

b. noted ethnic-specific waist circumference measurements

E's C/C is heaviness in the scrotum. you assess swelling of the testicle, along with warm scrotal skin. what do you suspect? a. cryptorchidism b. orchitis c. testicular torsion d. epididymitis

b. orchitis

a pt who is 35 y/o has identified a small, discrete mass in one breast. How should this be evaluated? a. ask whether the mass changes at the time of menses b. order a mammogram and ultrasound to assess the mass c. have her return 3-10 days after next menses d. order a mammogram 3-10 days after next menses

b. order a mammogram and ultrasound to assess the mass

a pt has been prescribed pioglitazone. The NP must remember to: a. check a hemoglobin A1c in 3 months b. order liver function studies in about 2-3 months c. screen for microalbuminuria d. wean off metformin

b. order liver function studies in about 2-3 months

the use of exenatide has been associated with the development of: a. leukopenia b. pancreatitis c. lymphoma d. vitiligo

b. pancreatitis

the obturator sign can be best described as abdominal pain elicited by: a. passive extension of the hip b. passive flexion and internal rotation of the hip c. deep palpation d. asking the pt to cough

b. passive flexion and internal rotation of the hip

b. presents to the ER with a dx of priapism. despite application of cold compresses and pain meds, relief is unsuccessful. what is the txment of choice? a. terbutaline 0.25mg SQ b. phenylephrine injection 0.3-0.5mL into the corpora cavernosa c. doxazosin 5mg SL d. lidocaine 1% via the glans

b. phenylephrine injection 0.3-0.5mL into the corpora cavernosa

a 45 y/o male who is in good health presents with c/o pain in his left heel. he states that the first few steps out of bed in the morning are extremely painful. he has no hx of trauma. what is the likely etiology of his pain? a. archilles tendinitis b. plantar fasciitis c. calcaneal spur d. arthritis of the foot

b. plantar fasciitis

a pt taking candesartan should avoid: a. strenuous exercise b. potassium supplements c. protein rich meals d. grapefruit juice

b. potassium supplements

a pt who is taking an ACE inhibitor should avoid: a. strenuous exercise b. potassium supplements c. protein rich meals d. grapefruit juice

b. potassium supplements

when initiating SSRI txment for a pt with panic d/o, the NP should consider all of the following EXCEPT: a. start with a low dose and slowly escalate doses as necessary b. preferable to use agents that are more energizing than ' less energizing c. select agents with a low rate of insomnia and akathisia d. SSRI tx can precipitate panic attacks with early use

b. preferable to use agents that are more energizing than less energizing

UTI in the male clt are divided into upper and lower-tract infections. a classic example of upper-tract infection includes: a. cystitis b. pyelonephritis c. prostatitis d. epidymitis

b. pyelonephritis

to help prevent meniscal tear, you advise: a. limiting participation in sports b. quadriceps-strengthening exercises c. using a knee brace d. applying ice to the knee before exercise

b. quadriceps- strengthening exercises

a gallstone that is not visualized on standard x-ray is said to be: a. radiopaque b. radiolucent c. calcified d. unclassified

b. radiolucent

Measures to prevent colonic diverticulosis and diverticulitis include all of the following except: a. increased wholegrain intake b. reg aerobic exercise c. adequate hydration d. refraining from excessive alcohol intake

b. reg aerobic exercise

intervention for pts with heatstroke includes: a. total body ice packing b. rehydration c. fluid restriction d. potassium supplementation

b. rehydration

rebound tenderness (Blumberg's sign) is best described as abdominal pain that worsens with: a. light palpation at the site of the discomfort b. release of deep palpation at the site of the discomfort c. palpation on the contralateral side of the abdomen d. deep palpation at the site of the discomfort

b. release of deep palpation at the site of the discomfort

a 30 y/o female pt who c/o fatigue has a screening TSH performed. Her TSH value is 8 mU/L. What should be done next? a. begin thyroid supplementation b. repeat the TSH and add T4 c. begin supplementation and repeat the TSH in 4-6 wks d. measure the T4 and consider repeating the test in one months

b. repeat the TSH and add T4

you suspect adolescent idiopathic scoliosis in V., age 15 who is in her growth spurt. you perform the Adams forward-bending test and note a right-sided rib hump. what is this indicative of? a. rt lumbar shifting b. rt thoracic curvature c. rt truncal shift d. spondylolysis

b. rt thoracic curvature

M., a 55 y.o female, presents to you for evaluation of left hand and wrist pain and swelling after a slip and fall on the ice yesterday. on exam, you note tenderness at her anatomical snuffbox. you know this probably indicates: a. ulnar styloid fx b. scaphoid fx c. hamate fx d. radial head fx

b. scaphoid fx Navicular Fracture (Scaphoid Bone Fracture) Initial x-ray of the wrist may be normal, but.... ***repeat x-ray in 2 weeks will show the scaphoid fracture (due to callus bone formation). High risk of avascular necrosis and nonunion. Splint wrist (thumb spica splint) & refer to hand surgeon.

indicate whether each finding normally would be present in gonorrheal urethritis in an otherwise well 28 y/o (yes or no) a. scrotal swelling - yes b. scrotal swelling - no

b. scrotal swelling - no

bone mineral density screening in women over age 65 yrs is an example of: a. primary prevention b. secondary prevention c. teritiary prevention d. quaternary prevention

b. secondary prevention

you prescribe a fluoroquinolone antibiotic to a 54 y/o woman who has occasional GERD sxs that she txs with an antacid. when discussing appropriate medication use, you advise that she should take the antimicrobial: a. with the antacid b. separated from the antacid use by 2-4 hrs before or 4-6 hrs after taking the fluoroquinolone c. without regard to antacid use d. apart from the antacid by about 1 hr on either side of the fluoroquinolone

b. separated from the antacid use by 2-4 hrs before or 4-6 hrs after taking the fluoroquinolone

you prescribe a fluoroquinolone antibiotic to a 54 y/o woman who has occasional GERD sxs that she txs with an antacid. when discussing appropriate medication use, you advise that she should take the antimicrobial: a. with the antacid b. separated from the anacid use by 2-4 hrs before or 4-6 hrs after taking the fluoroquinolone c. without regard to antacid use d. apart from the antacid by about 1 hr on either side of the fluoroquinolone

b. separated from the anacid use by 2-4 hrs before or 4-6 hrs after taking the fluoroquinolone

you have been asked to manage thyroid disease in a pregnant pt. A pregnant pt took L-thyroxin prior to becoming pregnant. What should be done about the L-thyroxin now that she is pregnant? a. it should be d/c during pregnancy b. she should continue it and have monthly TSH levels c. she should be switched to a supplement with a category B rating d. She can continue it during pregnancy without concern

b. she should continue it and have monthly TSH levels

a 20 y/o female reports that her grandmother and mother have osteopenia. What should she be encouraged to do to reduce her risk of osteopenia? a. aerobic exercise, st loss, low fat diet b. smoking cessation, weight bearing exercise c. calcium supplementation, vitamin A, C, & D intake d. optimal caloric intake, vitamin D supplementation

b. smoking cessation, weight bearing exercise

what is the most common site of prostate metastasis? a. lungs b. spine c. heart d. blood vessels

b. spine

J. age 13, notices a sparse growth of long, slightly pigmented, downy pubic hair at the base of his penis; slightly larger testes; and a larger red scrotum with a different texture. what Tanner stage is he in? a. stage 1 b. stage 2 c. stage 3 d. stage 4

b. stage 2

a pt with a lumbosacral strain will typically report: a. numbness in the extremities b. stiffness, spasm, and reduced ROM c. "electric" sensation running down one or both legs d. pain at its worst when in sitting position

b. stiffness, spasm, and reduced ROM

cancer of the prostate often begins with subtle sxs that develop very slowly over time and, if left untxed, will lead to metastasis. what prognostic finding is a significant indicator of probable metastatic disease? a. Gleason score of 2 b. sudden onset of weakness of the legs in a man with known prostate cancer c. TNM staging T1a, N0, M0 d. bladder outlet obstruction

b. sudden onset of weakness of the legs in a man with known prostate cancer

which of the following meds for ED txment has the longest half-life? a. sildenafil (viagra) b. tadafil (cialis) c. vardenafil (levitra) d. avanafil (stendra)

b. tadafil (cialis)

when the serum free T4 concentration falls: a. the TSH falls b. the TSH rises c. thee is no relationship between T4 and TSH d. T3 falls

b. the TSH rises

a 48 y/o woman with newly dx hypothyroidism asks about a "natural thyroid" medication she read about online and provides the drug's name, desiccated thyroid. As you counsel her about this medication, you consider all of the following EXCEPT: a. this product contains a fixed dose of T3 and T4 b. the medication is a plant-based product c. its pharmacokinetics differ significantly when compared to levothyroxine d. the majority of the study on tx for hypothyroidism has been done using levothyroxine

b. the medication is a plant-based product

the avoid the development of acute diverticulitis, tx of diverticulosis can include: a. avoiding foods with seeds b. the use of fiber supplements c. ceasing cigarette smoking d. limiting alcohol intake

b. the use of fiber supplements

which of the following statements is true about older men with HIV? a. they have a slower disease progression when compared with their younger cohorts b. they have a more rapid disease progression when compared with theiryounger cohorts c. they are rarely if ever injection drug users d. they cannot undergo txment with antiretrovirals

b. they have a more rapid disease progression when compared with their younger cohorts

as part of an evaluation of a 3cm round, mobile thyroid mass, you obtain a thyroid ultrasound scan revealing a fluid-filled structure. The most likely dx is: a. adenoma b. thyroid cyst c. multinodular goiter d. vascular lesion

b. thyroid cyst

you are advised a 58 y/o woman with hypothyroidism about the correct use of levothyroxine. She also takes a calcium supplement. All of the following information should be shared with the pt EXCEPT: a. take the med on an empty stomach b. to help with adherence, take your calcium supplement at the same time as your thyroid medication c. you should take the medication at approximately the same time every day d. do not take your medication with soy milk

b. to help with adherence, take your calcium supplement at the same time as your thyroid medication

you see a 42 y/o woman recently dx with sarcoidosis. she is reluctant to use any first-line meds for the condition because of severe adverse effects experienced previously. you consider prescribing all of the following except: a. hydroxychloroquine b. tofacitinib c. infliximab d. azathioprine

b. tofacitinib

noninfectious epididymitis is common in: a. soccer players b. truck drivers c. marathon runners d. men who wear "boxers"

b. truck drivers

J. appears with a rapid onset of unilateral scrotal pain radiating up to the groin and flank. you are trying to differentiate between epididymitis and testicular torsion. which test to determine whether swelling is in the testis or the epididymis should be your first choice? a. x-ray b. ultrasound c. technetium scan d. PE

b. ultrasound

a 51 y/o female pt presents with a 2cm palpable breast mass. How should this be evaluated to determine whether it is solid or cystic in nature? a. mammogram b. ultrasound c. MRI d. clinical breast exam

b. ultrasound

all of the following are risks for statin-induced myositis EXCEPT: a. advance age b. use of a low-intensity statin tx with a resin c. low body weight d. high-intensity statin tx

b. use of a low-intensity statin tx with a resin

To avoid development of acute diverticulitis, treatment of diverticulosis can include: a. avoiding foods w/seeds b. use of fiber supplements c. smoking cessation d. limiting alcohol intake

b. use of fiber supplements

erectile dysfunction is a complex phenomenon with a variety of causes. the predominant cause is: a. psychological b. vascular c. neurogenic d. drug related

b. vascular

a 36 y/o man c/o nausea, fever, malaise, and abdominal pain. He shows signs of jaundice and reports darkly-colored urine. Dx results show elevated serum aminotransferase less than 10x upper limits of normal (ULN). His most likely dx is: a. GERD b. viral hepatitis c. Crohn's disease d. Barrett esophagus

b. viral hepatitis

an example of an appropriate question to pose to a person with obesity who is in the contemplation change stage is: a. how do you feel about your wt? b. what are barriers you see to losing wt? c. what is your personal goal for wt loss? d. how do you envision my helping you meet your wt loss goal?

b. what are barriers you see to losing wt?

Drugs/class that decrease anticoagulant effect (10)

barbiturates carbamazepine chlordiazepoxide cholestryramine griseofulvin rifampin sucralfate azathioprine cyclosporine trazodone

In assessing for an S4 heart sound with a stethoscope, the nurse would listen with the:

bell at the apex with the patient in the left lateral position

plants sterols (5) examples lowers (3)

benecol spread, wheat germ, sesame oil, corn oil, peanuts decreased cholesterol, LDL, and Triglycerides

with an 8am dose of the following insulin forms, followed by an inadequate dietary intake and/or excessive energy use, at approximately what time would hypoglycemia be most likely to occur with Regular insulin?

between 10a-11am Regular insulin onset of action =30min-1 hr peak=2-3 hr duration=4-6 hr when trying to determine when a pt might be hypogylcemic after insulin administration, USE PEAK

Treatment for H. Pylori (+) ulcer: -Quadruple therapy is indicated and include

bismuth subsalicylate tab 600mg QID metronidazole tab 250mg QID tetracycline 500mg cap QID x2wks PPI daily for 4-6 weeks after or longer

During an assessment of a 68-year-old man with a recent onset of right-sided weakness, the nurse hears a blowing, swishing sounds with the bell of the stethoscope over the left carotid artery. This finding would indicate:

blood flow turbulence

which of the following statements is most consistent with IBD, IBS, or both conditions? pts should be advised to avoid trigger foods

both

Additional options for SSRI sexual dysfunction include adding _________ to the therapeutic regimen; support for this common practice is largely based on anecdotal reports. Also used to treat depression as well as seasonal affective disorder (SAD) and as an aid in smoking cessation.

bupropion The drug is nonaddicting and nontoxicating and is associated with few anticholinergic effects.

of the following individuals in need of an antidepressant, who is the best candidate for fluoxetine (prozac) txment? a. 80 y/o woman with depressed mood for 1yr after the death of her husband b. 45 y/o man with mild hepatic dysfunction c. 28 y/o man who occasionally "skips a dose" of the med d. 44 y/o man with decreased appetite

c. 28 y/o man who occasionally "skips a dose" of the med

which of the following is most consistent with the dx of dysthymia? a. 23 y/o man with a 2 month episode of depressed mood after a job loss b. 45 y/o woman with jitterness and difficulty initiating sleep for the past 6 months d. 15 y/o boy with a school adjustment problem and weekend marijuana use for the past yr

c. 38 y/o woman with fatigue and anhedonia for the past two years dysthymia is characterized as low level depression treat w/psychotherapy plus antidepressants

Morton has Type 2 diabetes. His treatment, which includes diet, exercise, and oral antidiabetic agents, is insufficient to achieve acceptable glycemic control. Your next course of action is to: a. Discontinue the oral antidiabetic agents and start insulin therapy. b. Suggest treatment using an insulin pump. c. Add a dosage of long-acting insulin at bedtime to the regimen. d. Increase the dosage of the oral antidiabetic agents.

c. Add a dosage of long-acting insulin at bedtime to the regimen. key word "agents"

abdominal palpation that yields rebound tenderness is also known as a positive___________ sign. a. Markel's b. Murphy's c. Blumberg's d. Nikolsky's

c. Blumberg's (Rebound Tenderness)

Mr. MK was recently given a dx of degernative joint disease. which assessment test would you use to check for effusion on his knee? a. Thomas test b. Tinel's sign c. Bulge test d. Phalen's test

c. Bulge test

you are seeing Mr. Smith, a 68 y/o man with suspected acute colonic diverticulitis. in choosing an appropriate imaging study to support this dx, which fo the following abdominal imaging studies is most appropriate? a. flat plate b. ultrasound c. CT scan with contrast d. barium enema

c. CT scan with contrast

periodic routine screening for hypothyroidism is indicated in the presence of which of the following clinical conditions? a. digoxin use b. male gender c. Down syndrome d. alcoholism

c. Down syndrome

which of the following hemograms would be expected for a 46 y/o woman with poorly controlled rheumatoid arthritis? a. Hg = 11.1 g/dL (12-14 g/dL); MCV = 66 fL (90=96 fL); reticulocytes = 0.8% (1%-2%) b. Hg = 10.1 g/dL (12-14 g/dL); MCV = 103 fL (90=96 fL); reticulocytes = 1.2% (1%-2%) c. Hg = 9.7 g/dL (12-14 g/dL); MCV = 87 fL (90=96 fL); reticulocytes = 0.8% (1%-2%) d. Hg = 11.4 g/dL (12-14 g/dL); MCV = 84 fL (90=96 fL); reticulocytes = 2.3% (1%-2%)

c. Hg = 9.7 g/dL (12-14 g/dL); MCV = 87 fL (90=96 fL); reticulocytes = 0.8% (1%-2%)

a 55 y/o female pt with diabetes has these fasting lipid values: Total cholesterol 200 mg/dL HDL 45 mg/dL LDL levels of 120 mg/dL Triglycerides 309 mg/dL This pt's Hgb A1c was measured. It is 9.2%. What is the relationship between HgbA1c and this patient's lipid values? a. there is no specific relationship b. elevated lipids will increase as HgbA1c increases c. HgbA1c decreases as triglycerides decrease d. HgbA1c will decrease as HDL increase

c. HgbA1c decreases as triglycerides decrease

which of the following is the most likely pt report with anal fissure? a. I have anal pain that is relieved with having a bowel movement b. even after having a bowel movement, I feel like i still need to go more c. I have anal pain for up to 1-2 hrs after I have a bowel movement d. I itch down there almost all of the time

c. I have anal pain for up to 1-2 hrs after I have a bowel movement

which medication used to tx diabetes is associated with diarrhea and flatulence? a. Pioglitazone b. Insulin c. Metformin d. Glimepiride

c. Metformin especially in the first 2 wks of use and also with increases in the dose - Metform is known to decrease morbidity and mortality associated with diabetes

an older adult has renal insufficiency, HTN, osteoarthritis, hypothyroidism, and varicose veins. Which medication should be avoided? a. acetaminophen b. beta blockers c. NSAIDS d. low dose aspirin

c. NSAIDS

you see a 26 y/o woman who has been recently dx with SLE and has initiated txment to control moderate sxs of the disease. including fatigue and joint pain. she mentions that she and her husband are hoping to start a family soon. in counseling her about pregnancy, you consider that: a. there is a low probablity of conception during symtomatic flares of SLE b. most rxments for SLE must be dc once a woman becomes pregnant c. SLE is associated with a high risk of pregnancy loss d. there is a higher risk of gestational diabetes in women with SLE

c. SLE is associated with a high risk of pregnancy loss

a 58 y/o man reports intermittent RUQ abdominal pain. He is obese and being actively txed for hyperlipidemia. Imagining in a pt with suspected symptomatic cholelithiasis usually includes obtaining an abdominal: a. MRI b. CT scan c. US of the RUQ d. flat plat

c. US of the RUQ

which of the following pts should be evaluated for possible surgical intervention for hemorrhoids? a. a 28 y/o woman with symptomatic external hemorrhoids who gave birth 6 days ago b. a 48 y/o man with Grad II internal hemorrhoids and improvement with standard medical tx c. a 44 y/o woman who has internal and external hemorrhoids with recurrent prolapse d. a 58 y/o man who has Grade I internal hemorrhoids and improvement with psyllium supplements

c. a 44 y/o woman who has internal and external hemorrhoids with recurrent prolapse

Cushing's disease is the specific type of Cushing's syndrome that is caused by: a. long-term exposure to corticosteroids b. a benign tumor of the adrenal gland c. a benign pituitary tumor d. an ectopic tumor that produces ACTH

c. a benign pituitary tumor the benign tumor on the pituitary gland produces ACTH in excess. also know that an ectopic tumor can also secrete ACTH. Read Fitzgerald Chapt 11, page 291, 4th edition

the pathophysiology of IBS can be best described as: a. shares the same pathophysiology as inflammatory bowel disease b. a patchy inflammatory process in the small bowel that most adolescents will outgrow with vigorous exercise and a low residue diet c. a condition that is the result of abnormal gut motor/sensory activity d. an overstimulation of pancreatic beta cell production

c. a condition that is the result of abnormal gut motor/sensory activity

the mechanism of action of metformin (Glucophage) is as: a. an insulin-production enhancer b. a product virtually identical in action to sulfonylureas c. a drug that increases insulin action in the peripheral tissues and reduces hepatic glucose production d. a facilitator of renal glucose excretion

c. a drug that increases insulin action in the peripheral tissues and reduces hepatic glucose production " Biguanide Class" metformin (Glucophage) -reduces hepatic glucose production and intestinal absorption **insulin sensitizer** via increased peripheral glucose uptake and utilization A1C reduction = 1-2% with intensified use

in txing a person with panic d/o using an SSRI, the NP should consider that there is: a. considerable abuse potential with these meds b. no significanttherapeutic advantage over TCAs c. a reduction in number and severity of panic attacks d. significant toxicity in overdose

c. a reduction in number and severity of panic attacks

the American Academy of Orthopaedic Surgeons favors all of the following in the management of sxs OA of the knee except: a. low-impact aerobic exercises b. weight loss for those with a BMI >=25kg/m c. acupuncture d. strengthening exercises

c. acupuncture

you see a 37 y/o man with RA who has been tx with hydroxychloroquine and oral NSAIDs for the past 3 months with little improvement in sxs. radiography indicates slight progression of RA in several major joints. you recommend: a. maintaining the current regimen b. increasing the dose of NSAIDs c. adding methotrexate to his regimen d. switching from hydroxychloroquine to a biologic agent

c. adding methotrexate to his regimen

which of the following grps should be screened for hepatitis B surface antigen (HBsAg)? a. pregnant women with no hx of receiving HBV vaccine b. pregnant women with documented prior HBV infection c. all pregnant women regardless of HBV vaccine hx d. all newborn infants born to mothers with chronic HBV infection

c. all pregnant women regardless of HBV vaccine hx

a 58 y/o man recently began taking an antihypertensive medication and reports that his heartburn has become much worse. he is most likely taking: a. atenolol (beta blocker) b. trandolapril (acei) c. amlodipine (CCB) d. losartan (arb)

c. amlodipine (calcium channel blocker)

the mechanism of action of pioglitazone is as: a. an insulin-production enhancer b. a reducer of pancreatic glucose output c. an insulin sensitizer d. a facilitator of renal glucose excretion

c. an insulin sensitizer ________________________________________ pioglitazone (Actos) and is in the class TZDs all end in "one" Cant be mixed with insulin -Rare risk of hepatic toxicity (<0.5%) mainly if used w/ insulin or sulfonylureas *-monitor ALT *-can cause/exacerbate CHF -edema risk -both r/t to increase in circulating volume -bladder risk (urinalysis, UA) -cause wt gain A1c reduction of 1-2% with intensive (max) use

When choosing a pharmacologic intervention to prevent recurrence of duodenal ulcer, you prescribe: a. PPI b. timed acid use c. antimicrobial therapy d. a H2 Blocker

c. antimicrobial therapy

when choosing pharmacological intervention to prevent recurrence of duodenal ulcer in a middle-aged man, you prescribe: a. a proton pump inhibitor b. timed antacid use c. antimicrobial tx d. a histamine2-receptor antagonist

c. antimicrobial tx

which of the following statements is true concerning hepatitis C? a. it usually manifests with jaundice, fever, and significant hepatomegaly b. among health-care workers, it is most commonly found in nurses c. at least 50% of person with acute hepatitis C go on to develop chronic infection d. interferon tx is consistently curative

c. at least 50% of person with acute hepatitis C go on to develop chronic infection

esophageal adenocarcinoma is usually located: a. in the upper esophagus b. near the upper esophageal sphincter c. at the junction of the esophagus and stomach d. in the lower esophagus

c. at the junction of the esophagus and stomach

hypothyroidism most often develops as a result of: a. primary pituitary failure b. thyroid neoplasia c. autoimmune thyroiditis d. radioactive iodine exposure

c. autoimmune thyroiditis

you see a 42 y/o man with uncomplicated urogenital gonorrhea. his medical records indicate a severe allergic rx to penicillin that includes difficulty breathing and diffuse urticaria. you recommend txment with: a. cefixime b. levofloxacin c. azithromycin d. tigecycline

c. azithromycin

DSM5 criteria for bulimia nervosa include all of the following except: a. eating an excessively lg amt of food w/in a discrete amt of time b. a sense of lack of control during binge eating episode c. binge eating and compensatory behavior occurring at least three times per week d. self-worth heavily influenced by body shape and wt

c. binge eating and compensatory behavior occurring at least three times per week

digital rectal exam may be performed to assess the prostate gland. which term does NOT describe a prostate gland that may have a tumor? a. nodular b. asymmetrical c. boggy d. indurated

c. boggy

when choosing an antimicrobial agent for the txment of chronic bacterial prostatitis, the NP considers that: a. gram positive organisms are the most likely cause of infection b. cephalosporins are the first-line choice of txment c. choosing an antibiotic with gram- negative coverage is critical d. length of antimicrobial txment is typically 5 days

c. choosing an antibiotic with gram-negative coverage is critical

in addition to the lab work described, results reveal the following for the pt who reports that she had yellow jaundice as a young child. Lab finding are: hepatitis B surface antigen (HBsAg) positive. these finding are most consistent with: a. no evidence of hepatitis B infection b. resolved hepatitis B infection c. chronic hepatitis B d. evidence of effective hepatitis B immunization

c. chronic hepatitis B

a 56 y/o woman is dx with mild diverticulitis. in addition to counseling her about increased fluid intake and adequate rest, you recommend antimicrobial txment with: a. amoxicillin with clarithromycin b. linezolid with daptomycin c. ciprofloxacin with metronidazole d. nitrofurantoin with doxycycline

c. ciprofloxacin with metronidazole

as you develop the initial txment plan for a woman with panic d/o, you consider prescribing: a. carbamazepine (tegretol) b. risperidone (risperdal) c. citalopram (celexa) d. bupropion (wellbutrin)

c. citalopram (celexa)

a 72 y/o man presents at an early stage of osteoarthritis in his left knee. he mentions that he heard about the benefits of using glucosamine and chondroitin for treating joint problems. in consulting the pt, you mention all of the following except: a. any benefit can take at least 3 months of consistent use before observed b. glucosamine is not associated with any drug interactions c. clinical studies have consistently shown benefit of long- term use of glucosamine & chondroitin for txtn OA of the knee d. chondroitin should be used with caution because of its antiplatelet effect

c. clinical studies have consistently shown benefit of long-term use of glucosamine and chondroitin for txing OA of the knee

according to the American Cancer Society recommendations, which of the following is the preferred screening method for colorectal cancer in adults at 10 year intervals starting at age 50? a. digital rectal exam b. fecal occult blood test c. colonoscopy d. barium enema study

c. colonoscopy

to plan for a community education program, the NP needs to know that persons at highest risk for developing thoracic outlet syndrome (TOS) are: a. bicycle riders b. dancers c. computer programmers d. swimming instructors

c. computer programmers thoracic outlet syndrome (TOS) is a result of subclavian artery compression resulting in severe ischemia of the arm. absent radial pulse with positional change

a 75 y/o female who is otherwise healthy has osteoarthritis in her rt knee. she c/o pain not relieved by acetaminophen 2000mg daily. what should be done? a. increase her acetaminophen to 4gm daily b. consider an NSAID c. continue acetaminophen and order physical therapy d. continue acetaminophen and refer to orthopedics

c. continue acetaminophen and order physical therapy

which muscle enzyme is elevated in polymyositis? a. aldolase A b. aspartate aminotransferase c. creatine kinase d. lactate dehydrogenase

c. creatine kinase elevated with brain and muscle breakdown

when d/c benzodiazepine txment after prolonged use, you recommend: a. terminating txment immediately b. decreasing the dose by 20% per day c. decreasing the dose by 25% per week d. decreasing the dose by 50% per week

c. decreasing the dose by 25% per wee

which of the following statements is true concerning anorexia nervosa? a. the disease affects men and women equally b. onset is usually in the mid-20s for men and women c. depression is often found concomitantly d. individuals with anorexia nervosa are aware of the extreme thinness associated with the disease

c. depression is often found concomitantly

a 24 y/o man presents with a 3 month hx of upper abdominal pain. he describes it as an intermittent, centrally located burning feeling in his upper abdomen, most often occurring 2-3 hrs after meals. his presentation is most consistent with the clinical presentation of: a. acute gastritis b. gastric ulcer c. duodenal ulcer d. cholecystitis

c. duodenal ulcer

metformin has all of the following effects EXCEPT: a. improved insulin-mediated glucose uptake b. modest wt loss with initial use c. enhanced fibrinolysis d. increased LDL cholesterol production

c. enhanced fibrinolysis I need to review this.... ????increased LDL cholesterol production???

a 24 y/o female runner is dx with Addison's disease. In counseling her about exercise, you recommend: a. tapering her running to only 10 min per day for 2 to 3 days per week b. ceasing any prolonged strenuous exercise c. ensuring an ample amt of sodium is ingested d. switching to a non-impact exercise

c. ensuring an ample amt of sodium is ingested An ample amt of sodium is recommended during heavy exercise, hot climates, and during GI upset (i.e. diarrhea) Remember that addison is a disorder that results from inadequate production of hormones from the adrenal gland. Glucocorticoids (cortisol) mineralocorticoid (aldosterone)....aldosterone regulates sodium and potassium balance....drop in sodium and potassium... muscle weakness, fatigue, low bp, salt cravings etc (hyponatremia and hypokalemia) ding did your light come on? Read Fitzgerald C.11 pg 289

three of the following interventions are appropriately used to prevent osteoporosis after menopause. Which ONE is NOT: a. limited alcohol intake b. performance of weight bearing activities for 40 min and at least 5 days/wk c. estrogen replacement therapy d. adequate calcium and vitamin D intake

c. estrogen replacement therapy

physical exam findings in pts with Graves' disease include: a. mucle tenderness b. coarse, dry skin c. eyelid retraction d. delayed relaxation phase of the patellar reflex

c. eyelid retraction

you are seeing 17 y/o Cindi. as part of the visit, you consider her risk factors for type 2 diabetes would likely include all of the following EXCEPT: a. obesity b. Native American ancestry c. family hx of type 1 diabetes d. personal hx of polycystic ovary syndrome

c. family hx of type 1 diabetes

which of the following is most consistent with the presentation of a pt with colonic diverticulosis? a. diarrhea and leukocytosis b. constipation and fever c. few or no sxs d. frank blood in the stool with reduced stool caliber

c. few or no sxs

a 48 y/o man with obesity and a 1 yr hx of classic GERD sxs has been on the consistent use of a therapeutic dose of a PPI for the past 6 months. he states he is really no better with the medicine and I have cut out most of the food that bothers my stomach I even cut out all alcohol and soda. PE reveals stable wt, mildly erythematous pharynx, and epigastric tenderness without rebound. Next step options include: a. obtaining an upper GI series b. referral for GERD surgery c. further eval with upper GI endoscopy d. obtaining FOBT testing

c. further eval with upper GI endoscopy

five yrs or more after type 2 diabetes diagnosis, which of the following medications is less likely to be effective in controlling plasma glucose? a. metformin b. pioglitazone c. glipizide d. insulin

c. glipizide sulfonylureas bc they rely on functioning beta cells

cardiovascular effects of hyperinsulinemia include: a. decreased renal sodium reabsorption b. constricted circulating volume c. greater responsiveness to angiotensin II d. diminished sympathetic activation

c. greater responsiveness to angiotensin II

a NP identifies filamentous structures and many uniform, oval shaped structures during a microscopic exam of vaginal discharge. these are probably: a. bacteria b. sperm c. hyphae and yeast d. candida albicans

c. hyphae and yeast

you notice that a 25 y/o woman has bruises on her rt shoulder. she states :I fell up against the wall:. the bruises appear finger shaped. she denies that another person injured her. what is your best choice of statement in response to this? a. your bruises really look as if they were caused by someone grabbing you b. was this really an accident? c. i notice the bruises are in the shape of a hand d. how did you fall?

c. i notice the bruises are in the shape of a hand

M. had an undescended testicle at birth, and at age 2 it remains in the inguinal region. his mother is afraid of surgery and asks for your advice. how do you respond? a. in many children,the testicle descends close to the 6th birthday b. even with only one normal testicle, he will have normal development c. if it hasn't descended by now, it probably won't. he needs to have surgery by age 6 d. don't worry; it can remain in that position forever with no problems

c. if it hasn't descended by now, it probably won't. he needs to have surgery by age 6

you see a 48 y/o woman with nonalcoholic fatty liver disease. Evaluation of infectious hepatitis includes the following: anti-HAV IgG(-) anti-HBs(-) anti-HCV (-) When considering her overall health status, you advise receiving which of the following vaccines? a. immunization against hepatitis A and B as based on her lifestyle risk factors b. immunization against hepatitis B and C c. immunization against hepatitis A and B d. immunization against A, B, C

c. immunization against hepatitis A and B

which choice best describes the most common presentation of a pt with type 2 diabetes? a. acute onset of hyperglycemia with other sxs b. hyperlipidemia and presence of retinopathy c. insidious onset of hyperglycemia with wt gain d. microalbuminuria

c. insidious onset of hyperglycemia with wt gain

a 62 y/o woman who reports frequent constipation is dx with an anal fissure. First-line tx includes all of the following except: a. stool-bulking supplements b. high fiber diet c. intraanal corticosteroids d. the periodic use of oral mineral oil

c. intraanal corticosteroids

the presence of esophageal cancer is commonly associated with: a. renal impairment b. chronic bronchitis c. iron-deficiency anemia d. unexplained wt gain

c. iron-deficiency anemia

a pt with newly diagnosed type 2 diabetes asks what his target blood pressure should be. The most correct response is: a. about 130/90 b. in the low 140s over the 90s c. less than 130/80 d. the systolic should be in the 120s-130s

c. less than 130/80

appropriate txment of acute epididymitis for a 32 y/o who has sex with other men is: a. ceftriaxone b. azithromycin c. levofloxacin d. TMP-SMX

c. levofloxacin

you examine a 64 y/o male with HTN and type 2 diabetes. His lipid profile is as follows: HDL is 38 mg/dL; LDL is 135 mg/dL; and Triglycerides level is 180 mg/dL. His estimated 10 year ASCVD risk is 5%. His current meds include a sulfonylurea, a biguanide, an ACEi, and a thiazide diuretic, and he has acceptable glycemic and blood pressure control. He states, I really watch the fats and sugars in my diet. Which of the following is the most appropriate advice? a. no further intervention is needed b. his lip profile should be repeated in 6 months c. lipid-lowering drug tx with a moderate intensity statin should be initiated d. the patient's dietary interventions appears adequate

c. lipid-lowering drug tx with a moderate intensity statin should be initiated

a pt c/o of rt leg numbness and tingling following a back injury. he has a diminished rt patellar reflex and his sxs are progressing to both legs. what test should be performed? a. lumbar x-rays b. lumbar CT scan c. lumbar MRI d. lumbar MRI with contrast

c. lumbar MRI

cyclooxygenase-1 (COX-1) contributes to: a. the inflammatory response b. pain transmission c. maintenance of gastric protective mucosal layer d. renal arteriole constriction

c. maintenance of gastric protective mucosal layer

IBD is associated with all of the following types of anemia EXCEPT: anemia of chronic disease b. iron-deficiency anemia c. megaloblastic anemia d. anemia associated with acute blood loss

c. megaloblastic anemia

a pt with dysuria has a urine specimen that reveals <10,000 bacteria and numerous trichomonads. How should this pt be managed? a. increased fluids and a urinary tract analgesic b. ciprofloxacin for 3 days c. metronidazole for 7 days d. ciprofloxacin and metronidazole

c. metronidazole for 7 days

a patient who is 5 feet tall and weighs 170 pounds (33.2kg/m2). How would the healthcare provider classify this patient? a. overweight b. mild obese c. moderate obese d. morbid obese

c. moderate obese

in txment resistant pts with panic d/o, which drug class is occasionally used? a. atypical antipsychotic b. selective dopamine reuptake inhibitor c. monoamine oxidase inhibitor d. neuroleptic

c. monoamine oxidase inhibitor

all of the following dx findings are expected in a pt with SLE except: a. elevated ESR b. anemia c. negative ANA test d. proteinuria

c. negative ANA test

as part of the evaluation of pts with OA, the NP anticipates finding: a. anemia of chronic disease b. elevated CRP level c. no disease-specific lab abnormalities d. elevated antinuclear antibody titer

c. no disease-specific lab abnormalities

diagnositic testing in IBS often reveals: a. evidence of underlying inflammation b. anemia of chronic disease c. normal results on most testing d. mucosal thickening on abdominal radiological imaging

c. normal results on most testing

which of the following best describes the hemogram results in a person with anemia of chronic disease that often accompanies IBD? a. microcytic, hypochromic b. macrocytic, normochromic c. normocytic, normochromic d. hyperprolifertive

c. normocytic, normochromic

to prevent a recurrence of testicular torsion, which of the following is recommended? a. use of a scrotal support b. avoidance of testicular trauma c. orchiopexy d. limiting the number of sexual partners

c. orchiopexy

a 63 y/o male has been your pt for several years. He is a former smoker who takes simvastatin, ramipril, and an aspirin daily. His BP and lipids are well controlled. he presents to your clinic with c/o fatigue and "just not feeling well" for the last few days. His VS and exam are normal. What should be done next? a. order a CBC and consider waiting a few days if normal b. inquire about feelings of depression and hopelessness c. order a CBC, metabolic panel, TSH, and urinalysis d. order a B12 level, TSH, CBC, and chest x-ray

c. order a CBC, metabolic panel, TSH, and urinalysis

a pt presents with rt shoulder pain (7/10 on the side scale) after an acute shoulder injury yesterday. he fell against a brick wall while working at his home. he reports pain that radiates into his upper arm. how should this be managed? a. immobilize the rt shoulder for 3 days b. rest, ice, and naproxen for one wk c. order an x-ray of the rt shoulder d. prescribe physical therapy for the pt

c. order an x-ray of the rt shoulder

the most sensitive and specific test for H. pylori infection from the following list is: a. stool Gram stain, looking for the offending organism b. serological testing for antigen related to the infection c. organism-specific stool antigen testing d. fecal DNA testing

c. organism-specific stool antigen testing

radiographic findings of OA of the knee often reveal: a. microfractures b. decreased density of subchondral bone c. osteophytes d. no apparent changes to the joint structure

c. osteophytes

what do affects older individuals, particularly women and is characterized by pain and stiffness in the cervical spine and shoulder and hip girdles, along with signs of systemic infection such as malaise, wt loss, sweats, and low-grade fever? a. fibromyalgia syndrome b. myofascial somatic dysfunction c. polymyalgia rheumatica d. Reiter's syndrome

c. polymyalgia rheumatica

diagnostic testing of a person with primary HSV-2 infection would likely show: a. negative virologic and serologic test results b. negative virologic test result and positive serologic test c. positive virologic test result and negative serologic test result d. positive virologic andserologic test results

c. positive virologic test result and negative serologic test result

the meglitinide analogues are particularly helpful adjuncts in type 2 diabetes care to minimize risk of: a. fasting hypoglycemia b. nocturnal hyperglycemia c. postprandial hyperglycemia d. postpradial hypoglycemia

c. postprandial hyperglycemia MEGLITINIDE ANALOGUE repaglinide (Prandin) nateglinide (Starlix) [ACTION] non-sulfonylurea insulin secretagogue agent (enhances release) very similar to sulfonylureas Meglitinides primarily work to lower blood sugar levels after meals, which, in turn, helps to lower A1C fast acting w/short half-life A1C reduction of 0.5-1.5% w/ intensified use. [EFFECTIVENESS] -typically less effective after 5 years of use because they require functioning pancreatic beta cells to be effective [COST] - on the $4 list (cheapest) [COMMON SE] -hypoglycemia -wt gain has been reported [SAFETY] -can cause severe hyperglycemia, especially in older adults (mainly repaglinide (Prandin) -adjust dose w/renal impairment [CONTRAINDICATIONS] -none just use w/ caution is known sulfa allergy [ELIMINATION OF DRUG] -renally eliminated, hence the the need adjust dose based on renal impairment.

which of the following meds should be avoided in a 65 y/o male with benign prostatic hyoperplasia? a. doxazosin b. ciprofloxacin c. pseudoephedrine d. propanolol

c. pseudoephedrine

concerning IBS, which of the following statements is most accurate? a. pts most often report chronic diarrhea as the most distressing part of the problems b. wt gain is often reported c. pts can present with bowel issues ranging from diarrhea to constipation d. the condition is associated with a strongly increased risk of colorectal cancer

c. pts can present with bowel issues ranging from diarrhea to constipation

J. comes in for an eval after a testicular self-exam. he states tha tit is probably nothing to worry about because his testicle is not tender, but he does have a tiny, hard nodule on the testicle. you confirm that there is a hard fixed nodule on his testicle. your next course of action would be: a. order a urinalysis b. schedule J. for a recheck next month c. refer J. to a specialist d. tell J. that it it is a cyst and if it does not resolve by itself, he will have it excised

c. refer J. to a specialist

you are caring for a 45 y/o woman from a developing country. She reports that she had yellow jaundice as a young child. her PE is unremarkable. her lab studies are as follows: AST, 22U/L (normal 0-31 U/L); alanine aminotransferase (ALT), 25 U/L ( normal 0-40U/L); hepatitis A virus immunoglobulin G (HAV IgG) positive. lab testing reveals: a. chronic hepatitis A b. no evidence of prior or current hepatitis A infection c. resolved hepatitis a infection d. prodromal hepatitis A

c. resolved hepatitis a infection

a pt who frequently has episodes of gout should avoid which groups of food? a. beans, rice,a nd tea b. scrambled eggs, milk, and toast c. roast beef with gravy, rice d. fish and steamed vegetables

c. roast beef with gravy, rice

which of the following statements is true concerning domestic violence? a. it is found largely among people of lower socioeconomic status b. the person in an abusive relationship usually seeks help c. routine screening is indicted during pregnancy d. a predictable cycle of violent activity followed by a period of calm is the norm

c. routine screening is indicted during pregnancy

you are seeing a pt who is taking warfarin and cholestyramine and provide the following advice: a. take both medication together b. you need to have additional hepatic monitoring tests while on this combination c. separate the cholestyramine from other medications by at least two hrs d. make sure you take these medications on an empty stomach

c. separate the cholestyramine from other medications by at least two hrs

a 13 y/o obese (BMI > than 90%) boy reports low-grade lft knee pain for the past 2 months. he denies antecedent trauma but admits to frequent horseplay with his friends. the pain has progressively worsened, and he is now unable to bear wt at all on his left leg. his current c/o include left groin, thigh, and medial knee pain and tenderness. his exam demonstrates negative drawer, Lachman, and McMurray tests; left hip with decreased internal rotation and abduction; and knee flexion causing external hip rotation. based on the above scenario, you suspect? a. left meniscal tear b. left anterior cruciate ligament (ACL tear) c. slipped capital femoral epiphysis (SCFE) d. Osgood-Schlatter disease

c. slipped capital femoral epiphysis (SCFE)

a 52 y/o presents with sxs of diabetes today. His glucose is 302 mg/dL. How should this be managed today? a. have him return tomorrow to recheck his blood glucose b. start metformin c. start insulin d. start metformin plus pioglitazone

c. start insulin this pt can be dx with diabetes today = he is symptomatic and his BG level is >200. this pt is glucose toxic

an 80 y/o pt who is overweight and sedentary has developed elevated, fasting glucose levels (142, 153, and 147 mg/dL). She was dx with diabetes today. Considering her age, how should the NP proceed? a. tx with dietary interventions only b. initiate insulin c. start metformin d. start a sulfonylurea

c. start metformin

a 26 y/o male reports that he has shared a needle with a friend during injection drug use. He is certain that his friend has chronic hepatitis B infection and is uncertain about his own immunization hx. You recommend: a. start the HBV vaccine series b. administering hepatitis B immune globulin c. starting the HBV vaccine series and administering hepatitis B immune globulin d. waiting until the HBsAg results before administering hepatitis B immune globulin

c. starting the HBV vaccine series and administering hepatitis B immune globulin

a 25 y/o male is training for a marathon. he reports an acute onset of scrotal pain after a 10 mile run. he has nausea and is found to have an asymmetric, high-riding testis on the right side. what should be suspected. a. sports hernia b. epididymitis c. testicular torsion d. prostatitis

c. testicular torsion

A. has been on finasteride (proscar) for 6 months for benign prostatic hypertrophy. a decrease in his prostate-specific antigen (PSA) from the original value of 5.4 has not occurred. your initial expectation is: a. that his PSA would remain stable, neither increasing nor decreasing b. A.'s dosage should be reduced only after he has been on the med for approximately 12 months c. that a significant reduction in the overall PSA would occur if the level is associated with true benign prostatic hypertrophy d. that an elevation of the antigen would occur because of the effect of the alpha-adrenergic antagonist

c. that a significant reduction in the overall PSA would occur if the level is associated with true benign prostatic hypertrophy

risk factors for the development of hemorrhoidal sxs include all of the following EXCEPT: a. the hemorrhoids do no prolapse b. the hemorrhoids prolapse upon defecation but reduce spontaneously c. the hemorrhoids prolapse upon defecation and must be reduced manually d. the hemorrhoids are prolapsed and cannot be reduced manually

c. the hemorrhoids prolapse upon defecation and must be reduced manually

in prescribing levothyroxine therapy for an elderly pt, which of the following statements is true? a. elderly persons require a rapid initiation of levothyroixine therapy b. TSH should be checked about 2 days after dosage adjustment c. the levothyroxine dose needed by elderly persons is 75% or less of that needed by younger adults d. TSH should be suppressed to a nondetectable level

c. the levothyroxine dose needed by elderly persons is 75% or less of that needed by younger adults

which of the following statements is true regarding depression and relapse? a. without maintenance txment, the relapse rate is typically less than 50% in the first year b. the risk of relapse is less for those who have experienced multiple episodes of major depressive d/o c. the risk of relapse is greatest in the first two months after dc of tx d. relapse rarely occurs if there is an absence of symptoms after 9 months of txment dc

c. the risk of relapse is greatest in the first two months after dc of tx

a 44 y/o man has been taking an SSRI for the past 4 months and c/o new onset of sexual dysfunction and difficulty achieving orgasm. you advise him that: a. this is a transiet side effect often seen in the first weeks of txment b. switching to another SSRI would likely be helpful c. this is a common adverse effect of SSRI that is unlikely to resolve without adjustment to his therapy d. he should see an urologist for further eval

c. this is a common adverse effect of SSRI that is unlikely to resolve without adjustment to his therapy

which of the following is least likely to be reported by pts with carpal tunnel syndrome? a. worst sxs during the day b. burning sensation in the affected hand c. tingling pain that radiates to the forearm d. nocturnal numbness

c. tingling pain that radiates to the forearm

a 26 y/o man presents with acute abdominal pain. as part of the eval for acute appendicitis, you order a WBC count with differential and anticipate the following results: a. total WBCs 4500 mm3; neutrophils 35%; bands 2%; lymphocytes 45% b. total WBCs 14,000 mm3; neutrophils 55%; bands 3%; lymphocytes 38% c. total WBCs 16,500 mm3; neutrophils 66%; bands 8%; lymphocytes 22% d. total WBCs 18,100 mm3; neutrophils 55%; bands 3%; lymphocytes 28%

c. total WBCs 16,500 mm3; neutrophils 66%; bands 8%; lymphocytes 22%

priapism is a potential adverse effect of which of the following psychotropic meds? a. bupropion b. sertraline c. trazodone d. amitriptyline

c. trazodone

which type of urinary incontinence results from Parkinson's disease and multiple sclerosis? a. overflow incontinence b. stress incontinence c. urge incontinence d. functional incontinence

c. urge incontinence

acroparesthesia frequently reported in pts with carpal tunnel syndrome is best described as: a. constant pain radiating from the elbow b. a transient inability to move the fingers c. waking up at night with numbness and burning pain in the fingers d. muscle spasms that cause fist clenching

c. waking up at night with numbness and burning pain in the fingers

you have just txed J.'s condylomata acuminata with podophyllum in benzoin. what instructions do you give him? a. refrain from sexual relations for 48 hrs b. don't take a shower until tomorrow morning c. wash the medication off within 1-2 hrs d. go into the bathroom now and wash the medication off

c. wash the medication off within 1-2 hrs

which beverage below does not increase the risk of gout in a male who is prone to this condition? a. vodka b. beer c. wine d. bourbon

c. wine

in caring for a pt with diabetes, microalbuminuria measurement should be obtained: a. annually if urine protein is present b. periodically in relationship to glycemic control c. yearly d. with each office visit related to diabetes

c. yearly

T. asks you about returning to his normal sex life after surgery for BPH. you tell him, a. you probably won't be able to have an erection after surgery; we need to discuss alternative ways of lovemaking b. you need to wait several months after surgery to make sure the site has healed c. you may resume sexual activity 4-6 wks after surgery d. you will have to ask the surgeon

c. you may resume sexual activity 4-6 wks after surgery

De Quervain's tenosynovitis

caused by an inflammation of the tendon and its sheath, which is located at the base of the thumb. The screening test is Finkelstein's, which is positive if there is pain and tenderness on the wrist (thumb side) upon ulnar deviation

Gonorrhea frequently produces beta-lactamase, therapeutic agents should include those with beta-lactamase stability and a ___________, such as injectable ceftriaxone or oral cefixime

cephalosporin

Examples of high-purine foods

certain seafood (scallops, mussels) organ and game meats beans spinach asparagus oatmeal baker's & brewer's yeasts when taken as dietary supplements

The ___________ lesion is typically found at the site of inoculation with a vesicular-form to pustular-form lesion that creates a painful, soft ulcer with a necrotic base. Multiple lesions, acquired through autoinoculation, usually are found. A dense, matted lymphadenopathy can be found on the ipsilateral side of the lesion. The affected nodes often spontaneously rupture.

chancroid

Appropriate antimicrobial treatment for a 65-year-old man with acute bacterial prostatitis is:

cipro

Term describing patterns of skin lesions wheal=

circumscribed area of skin edema (hive)

Giving a person with thalassemia iron can cause(2)

cirrhosis of liver cardiomyopathy

Osgood Schlatter presentation

common cause of knee pain in young athletes due to repetitive stress on patellar tendon by quadriceps muscle results in pain, tenderness, and swelling @ the tibial tuberosity common during rapid growth spurts and participation in sports (basketball,soccer) ****avoid sports that involve heavy quadricep loading or deep knee bending***

nursemaid elbow

common injury in young child resulting from swinging or pulling of child's arm, radial head subluxation X-ray is normal supportive care only

Treatment for severe recalcitrant psoriasis... SE of their use: Who at highest risk for SE?

cyclosporine, methotrexate, systemic retinoids, and new biologicals such as tumor necrosis factor (TNF) modulators etanercept (Enbrel), adalimumab (Humira) or ustekunumab (Stelara) are used. [SE: ] injection site and infusion reactions & reactivation of latent tuberculosis [Who] DM, HF, Concomitant of immunosuppressive

the commonly recommended physical activity level of 10,000 step per day is roughly the equivalent of walking ___ miles a. 1 to 2 b. 2 to 3 c. 3 to 4 d. 4 to 5

d. 4 to 5

a 40 y/o male has been dx with acute bacterial prostatitis. his prostate specific antigen is elevated on dx. hwo soon should his PSA be rechecked? a. 2-3 days b. 1 week c. 2 weeks d. 4 weeks

d. 4 weeks

a 40 y/o male has been dx with acute bacterial prostatitis. His prostate specific antigen (PSA) is elevated on dx. How soon should his PSA be rechecked? a. 2-3 days b. 1 wk c. 2 wks d. 4 wks

d. 4 wks

optimally, at what interval should TSH be reassessed after a levothyroxine dosage is adjusted? a. 1-2 wks b. 2-4 wks c. 4-6 wks d. 6-8 wks

d. 6-8 wks

in deciding whether to tx M., who has BPH, you use the American Urological Association scale. no txment is indicated if the AUA score is: a. 36 or higher b. 20-35 c. 8-19 d. 7 or lower

d. 7 or lower

approximately what percent of pts experiencing low back pain will have the sxs resolve within 1 month without specific tx? a. 33% b. 57% c. 78% d. 90%

d. 90%

which of the following is most likely to be noted in a 45 y/o woman with lab evidence of chronic excessive alcohol ingestion? a. alanine aminotransferase (ALT) 202 U/L (0 to 31 U/L, mean corpuscular volume (MCV) 70 fL (80-96) b. aspartate transaminase (AST) 149 U/L (0 to 31 U/L, mean corpuscular volume (MCV) 81 fL (80-96) c. alanine aminotransferase (ALT) 88 U/L (0 to 31 U/L, mean corpuscular volume (MCV) 140 fL (80-96) d. AST 80 U/L (0 to 31 U/L, mean corpuscular volume (MCV) 103 fL (80-96)

d. AST 80 U/L (0 to 31 U/L, mean corpuscular volume (MCV) 103 fL (80-96)

Which patient below is at greatest risk of development of NAFLD? a. 30 year old, Type I diabetic, BMI 22 b. Hypertensive, hyperlipidemic male, BMI 25 c. Female, BMI 31, metabolic syndrome d. Male, BMI 35, type 2 DM, hypertensive, hyperlipidemic

d. Male, BMI 35, type 2 DM, hypertensive, hyperlipidemic non-alcoholic fatty liver disease NAFLD -most common persistent liver disorder in Western countries with an estimated overall prevalence of 20-30%? -Pt has signs of insulin resistance, chronic elevation of ALT -on US the liver is fatty, enlarged, echogenic ("bright")

you examine a 46 y/o male who is 1 1/2 ppd cigarette smoker with HTN. He has no evidence of clinical atherosclerotic cardiovascular disease (ASCVD), and his estimated 10-yr ASCVD risk is 10%. His lipid profile is as follows: HDL is 48 mg/dL; LDL is 192 mg/dL; and Triglycerides level is 110 mg/dL He has been on a low-cholesterol diet for six months when these tests were taken. Which of the following represents the best next step? a. no further intervention is required b. a fibrate should be prescribed c. a low-intensity 3-hydroxy-3-methylglutaryl-coenzyme A reductase inhibitor should be prescribed d. a high-intensity 3-hydroxy-3-methylglutaryl-coenzyme A reductase inhibitor should be prescribed

d. a high-intensity 3-hydroxy-3-methylglutaryl-coenzyme A reductase inhibitor should be prescribed

Criteria for the Dx of T2DM include: a. classic sxs regardless of fasting plasma glucose measurement b. plasma glucose level of 126 mg/dL (7 mmol/L) as a random measurement c. a 2 hr glucose measurement of 156 mg/dL (8.6 mmol/L) after a 75 g anhydrous glucose load d. a plasma glucose level of 126 mg/dL (7 mmol/L) or greater after an 8 hr fast on more than one occasion

d. a plasma glucose level of 126 mg/dL (7 mmol/L) or greater after an 8 hr fast on more than one occasion

results of a nerve conduction study in a personw ith carpal tunnel syndrome would reveal: a. erratic nerve impulses during forearm muscle contraction b. a lack of nerve impulses in the carpal tunnel c. continued firing of the medial nerve while the forearm muscles are at rest d. a slowing of nerve impulses in the carpal tunnel

d. a slowing of nerve impulses in the carpal tunnel

a 49 y/o pt has osteoarthritis in the lumbar spine and hip. his hip x-ray demonstrates bone on bone. what can be done to resolve his c/o pain in his hip? a. acetaminophen only b. an NSAID only c. acetaminophen and PT d. acetaminophen and a referral to ortho

d. acetaminophen and a referral to ortho

routine anal Pap smears can be considered for all of the following pt populations EXCEPT: a. men with HIV b. men who have sex with men c. women with a hx of anogenital HPV infection d. all males under age25 years

d. all males under age 25 years

altering the gut pain threshold in IBS is a possible therapeutic outcome with the use of: a. loperamide b. dicyclomine c. bismuth subsalicylate d. amitriptylin (elavil)

d. amitriptylin (elavil)

a localized tumor in the prostate gland associated with early stage prostate cancer is likely to produce: a. urinary hesitancy b. low back pain c. urinary frequency d. an absence of sxs

d. an absence of sxs

the use of which of the following medications has the potential for causing the greatest reduction in HbA1c? a. a biguanide b. a thiazolidinedione c. a sulfonylurea d. an insulin form

d. an insulin form >=1.5-3.5%

thyroid stimulating hormone (TSH) is released by the: a. thyroid follicles b. adrenal cortex c. hypothalamus d. anterior lobe of the pituitary

d. anterior lobe of the pituitary

current vaccine guidelines recommend administering the immunization against HAV to: a. those living in or traveling to areas endemic for the disease b. food handlers and day-care providers c. military personnel d. any person who wishes to receive the vaccine

d. any person who wishes to receive the vaccine

you see a 37 y/o man c/o low back pain consisting of stiffness and spasms but without any sign of neurologic involvement. you recommend all of the following interventions except: a. application of cold packs for 20 mins 3-4 x a day b. use of NSAIDs or acetaminophen for pain control c. initiation of aerobic and toning exercises d. bed rest for at least 5 days

d. bed rest for at least 5 days

which of the following classes of meds is commonly recommended as part of first line tx in the newly dx person with type 2 diabetes? a. alpha-glucosidase inhibitor b. meglitinide c. thiazolidinedione d. biguanide

d. biguanide glucophage (Metformin)

regarding the current scientific evidence on the use of glucosamine and chondroitin for the management of symptomatic OA of the knee, the American Academy of Orthopaedic Surgeons: a. strongly favors their use b. provides a moderate-strength recommendation for their use c. cannot recommend for or against the use of these supplements (limited evidence) d. cannot recommend the use of these supplement

d. cannot recommend the use of these supplement

appropriate antimicrobial txment for a 65 y/o with acute bacterial prostatitis is: a. erythromycin b. cefepime c. TMP-SMX d. ciprofloxacin

d. ciprofloxacin

which of the following should be periodically monitored with the use of a biguanide? a. creatine kinase (CK) b. alkaline phosphatase (ALP) c. alanine aminotransferase (ALT) d. creatinine (Cr)

d. creatinine (Cr) rare renal risk for developing lactic acidosis

the anticipated effect on the lipid profile with high-dose omega-3 fatty acid use includes: a. increase in HDL b. decrease in LDL c. decrease in total cholesterol d. decrease in triglycerides

d. decrease in triglycerides

which of the following findings would you expect to encounter in a 33 y/o man with appendiceal abscess? a. leukopenia with lymphocytosis b. positive Cullen's sign c. protracted nausea and vomiting d. dullness to percussion in the abdominal RLQ

d. dullness to percussion in the abdominal right lower quadrant Fitzgerald pt 157 " an ill-defined RLQ abd mad that is usually dull to percuss w/wo degree of rebound tenderness in a person with a presentation of appendiceal perforation is suggestive of abscess formation". Sx after ABX has been initiated.

which lab abnormality very commonly accompanies hypothyroidism? a. hypernatremia b. polycythemia d. dyslipidemia d. hypoprolactinemia

d. dyslipidemia

risk facts for hepatitis B infection include all of the following EXCEPT: a. having multiple sexual partners b. having an occupation that exposes you to human blood c. injection drug user d. eating food prepared by a person with an HBV infection

d. eating food prepared by a person with an HBV infection

to avoid rebound gastric hyperacidity following discontinuation of long-term PPI use, all of the following methods can be used EXCEPT: a. gradually tapering the PPI dose with supplemental antacid b. switch toevery-other- day dosing of PPI with supplemental antacid c. switch to a low-dose histamine2 receptor antagonist therapy with supplemental antacid d. empiric H. pylori tx

d. empiric H. pylori tx

B. a 32 y/o gay man, has been dx with acute bacterial prostatitis. in addition to providing education, you would encourage him to avoid: a. rest b. NSAIDs c. hydration and stool softeners d. engaging in any activity that would elicit prostatic massage

d. engaging in any activity that would elicit prostatic massage rational- ^risk for bacteremia

when performing a newborn assessment of a male infant, you note that the urethral opening is on the dorsal side of the glans. this is referred to as: a. hypospadias b. Peyronie's disease c. priapism d. epispadias

d. epispadias

in an older adult with type 2 diabetes with gastroparesis, the use of which of the following medications should be avoided? a. insulin glargine (Lantus) b. insulin aspart (NovoLog) c. glimepride (Amaryl) d. exenatide (Byetta)

d. exenatide (Byetta) GLPs incretin mimetic injection only med part of their action slows gastric emptying

Hot flashes that occur during menopause are thought to be related to: a. low estrogen levels b. low progesterone levels c. fluctuating progesterone levels d. fluctuating estrogen levels

d. fluctuating estrogen levels

a 34 y/o woman c/o of progressive weakness, fatigue, poor appetite, and weight loss. She has also noticed the development of hyperpigmentation, mainly on theknuckles, elbows, and knees. All of the following blood tests can be used to help confirm a dx of Addison's disease EXCEPT: a. sodium b. potassium c. cortisol d. folate

d. folate

all of the following are effective methods to kill the hepatitis A virus except: a. heating food to more than 185F (85C) for at least 1 minute b. adequately chlorinating water c. cleaning surfaces with a 1:100 bleach solution d. freezing food for at least 1 hr

d. freezing food for at least 1 hr

what is the recommendation from American Cancer Society for screening an average risk 40 y/o Caucasian male for prostate cancer? He should have: a. digital rectal exam b. serum PSA c. PSA and DRE d. he should be screened starting at age 50 years

d. he should be screened starting at age 50 years

a pt has been taking fluoxetine for 1 wk and c/o mild nausea and diarrhea. you advise that: a. this is a common, long lasting side effect of SSRIs b. he should dc the med c. another antidepressant should be substituted d. he should be taking the medication with food

d. he should be taking the medication with food

lorazepam or oxazepam is the preferred benzodiazepine for txing alcohol withdrawal sxs when there is a concomitant hx of: a. seizure d/o b. folate deficiency anemia c. multiple substance abuse d. hepatic dysfunction

d. hepatic dysfunction

all fo the following are risks for lactic acidosis in individuals taking metformin EXCEPT: a. presence of chronic renal insufficiency b. acute dehydration c. radiographic contrast dye use d. hx of allergic rx to sulfonamides

d. hx of allergic rx to sulfonamides this relates to the need of cautious use of sulfonylureas in the presence of sulfa allergy

a 43 y/o man is experiencing an acute adrenal crisis and presents with prominent nausea, vomiting, and low blood pressure. He appears cyanotic and confused. The most appropriate txment is an injection of: a. epinephrine b. insulin c. adrenaline d. hydrocortisone

d. hydrocortisone During an Addisonian crisis, an immediate injection of hydrocortisone is needed with supportive treatment for low BP. Fitzgerald, pt.290 chapter 11.

which of the following statements is not true concerning the effects of exercise and insulin resistance? a. approximately 80% of the body's insulin mediated glucose uptake occurs in skeletal muscle b. with regular aerobic exercise, insulin resistance is reduced by approximately 40% c. the insulin resistance reducing effect of exercise persist for 48hrs after the activity d. hyperglycemia can occur as a result of aerobic exercise

d. hyperglycemia can occur as a result of aerobic exercise

a 50 y/o females believes that she is menopausal. She c/o of hot flashes and has not had menses in 12 months. Which of the following test results may be helpful for confirmation of menopause? a. increased thyroid stimulating hormone b. decreased follicle stimulating hormone c. hypoestrogenemia d. increased follicle stimulating hormone

d. increased follicle stimulating hormone

a woman is 65 y/o, presents to your clinic with a breast lump. She has had only normal annual mammograms and her last one was 6 months ago. What is true about this lump? a. it is probably breast cancer b. it may not be a lump at all c. it is likely a fibroadenoma d. it is probably a benign lesion

d. it is probably a benign lesion

initial txment for meniscal tear includes all of the following except: a. NSAID use b. applying ice to the affected area c. elevation of the affected limb d. joint aspiration of the affected knee

d. joint aspiration of the affected knee

the mechanism of action of glucosamine and chondroitin is: a. via increased production of synovial fluid b. through improved cartilage repair c. via inhibition of the inflammatory response in the joint d. largely unknown

d. largely unknown

which of the following is true concerning colorectal cancer? a. most colorectal cancer are found during rectal exam b. rectal carcinoma is more common than cancers involving the colon c. early manifestations include ab pain and cramping d. later disease presentation often includes IDA

d. later disease presentation often includes iron- deficiency anemia

an x-ray report of a pt's painful right knee indicates "joint space narrowing". what does this mean? a. the pt is aging b. gout is present c. this is a normal finding d. loss of cartilage has occurred

d. loss of cartilage has occurred

secondary causes of hyperglycemia potentially include the use of all of the following meds EXCEPT: a. high dose niacin b. systemic corticosteroids c. high dose thiazide diuretics d. low dose angiotensin receptor blockers

d. low dose angiotensin receptor blockers

what is the preferred test assess for ophthalmopathy resulting from Graves' disease? a. fine needle aspiration b. thyroid ultrasound c. computed tomography (CT) scan d. magnetic resonance imaging (MRI)

d. magnetic resonance imaging (MRI) pg 842

when teaching T. how to do a testicular self-exam, which of the following do you tell him? a. examine your testicles when you are cold because them makes them more sensitive b. make sure your hands are dry to create friction c. if you feel firmness above and behind the testicle, make an appt d. make an appt if you note any hard lumps directly on the testicle, whether they are tender or not

d. make an appt if you note any hard lumps directly on the testicle, whether they are tender or not

which of the following WBC forms is an ominous finding in the presence of severe bacterial infection? a. neutrophil b.lymphocyte c. basophil d. metamyelocyte

d. metamyelocyte

which drug listed below is NOT associated with weight gain? a. insulin b. Pioglitazone c. citalopram d. metoprolol

d. metoprolol

risk factors for carpal tunnel syndrome include all of the following except: a. pregnancy b. untreated hypothyroidism c. repetitive motion d. multiple sclerosis

d. multiple sclerosis

the most accurate dx tool for prostate cancer is: a. DRE b. PSA c. transrectal ultrasound exam d. needle biopsy

d. needle biopsy

A pt with testicular torsion will have a: a. positive cremasteric reflex on the affected side b. negative cremasteric reflex on the affected side c. a positive cremasteric reflex bilaterally d. negative cremasteric reflex bilaterially

d. negative cremasteric reflex bilaterially

a recently dx pt with type 2 diabetes presents today with fever and burning with urination. She is dx with a UTI. Her urine dipstick is positive for protein. Which statement is correct? a. this pt has microalbuminuria secondary to diabetes b. the finding of protein is an incidental finding c. the proteinuria is related to the UTI d. no specific conclusions can be drawn about the proteinuria

d. no specific conclusions can be drawn about the proteinuria

a 38 y/o nonsmoking man presents with s/s consistent of GERD. he has self-txed with OTC antacids and acid suppression tx with effect. his wt is stable and he denies nausea, vomiting, diarrhea, or melena. which of the following represents the most appropriate diagnostic plan for this pt? a. fecal testing for H. pylori antigen b. upper GI endoscopy c. barium swallow d. no specific diagnostic testing is needed

d. no specific diagnostic testing is needed

a 55 y/o female pt with diabetes has these fasting lipid values: Total cholesterol 200 mg/dL HDL 45 mg/dL LDL levels of 120 mg/dL Triglycerides 309 mg/dL According to the American Diabetes Association, which pt lipid value(s) meet(s) the goal for this pt? a. total cholesterol only b. total cholesterol, HDL, LDL c. LDL only d. none are at goal

d. none are at goal

a 23 y/o has a nontender "bag of worms" mass within the left scrotum that disappears when he is in the supine position. he is dx with a varicocele. what is a risk factor that may have contributed to this condition? a. younger age b. current cigarette smoker c. multiple sexual partners d. none of the above

d. none of the above

among surgical and procedural interventions, the American Academy of Orthopaedic Surgeons strongly recommends the use of which of the following for the management of symptomatic OA of the knee? a. intraarticular corticosteroid use b. hyaluronic acid injections c. arthroscopy with lavage and or debridement d. none of the above

d. none of the above

a pt with anorexia nervosa (AN) had sxs that began about 6 months ago. She presents today and is dx with AN. She has a laboratory evaluation. What might be expected? a. elevated potassium b. decreased potassium, decreased glucose c. elevated BUN, Cr d. normal lab values

d. normal lab values

a pt was exposed to HIV through sexual intercourse. he should be followed with screening tests to identify seroconversion for: a. 4-6 weeks b. 3-4 months c. about six months d. one year

d. one year

a 63 y/o male has been your pt for several years. He is a former smoker who takes simvastatin, ramipril, and an aspirin daily. His BP and lipids are well controlled. he presents to your clinic with c/o fatigue and "just not feeling well" for the last few days. His VS and exam are normal. His CBC, TSH, urinal analysis are normal. His liver enzymes are six times the upper limits of normal. What should be done next? a. order a hepatitis panel and stop his medication b. refer to gastroenterology c. refer for a toxicology evaluation d. order a hepatitis panel and stop his simvastatin and aspirin

d. order a hepatitis panel and stop his simvastatin and aspirin

which of the following is the most common serious complication of cholecystitis? a. adenocarcinoma of thegallbladder b. gallbladder empyema c. hepatic failure d. pancreatitis

d. pancreatitis

risk of benzodiazepine misuse is minimized by use of: a. agents with a shorter T 1.2 b. the drug as an as- needed rescue med for acute anxiety c. more lipophilic products d. products with long duration of action

d. products with long duration of action

characteristics of BINGE EATING do include all of the following EXCEPT: a. lack of control over the amt and type of food eating b. behavior present for at least 6 months c. marked distress, self-anger, shame, and frustration as a result of binging d. purging activity after an eating binge

d. purging activity after an eating binge usually are obese bc they done purge

J. age 76 has been given dxes of osteoporosis confirmed with a dual-energy x-ray absorptiometry (DEXA) scan. you have educated her about the importance of increasing calcium and vitamin D in her diet and starting a low impact wt bearing exercise program. you are also going to start her on medial management. J. ask you about a drug called a "SERM" that she has heard has been shown in studies to prevent vertebral fx. which of the following pharmacological tx for osteoporosis is classified as a selective estrogen receptor modulator (SERM)? a. aldendronate b. risedronate c. salmon calcitonin d. raloxifene

d. raloxifene

long-term PPI use is associated with all of the following except: a. increased risk of pneumonia in hospitalized pts. b. increased risk of C. difficile colitis in hospitalized pts c. reduced absorption of calcium and magnesium d. reduced absorption of dietary carbohydrates

d. reduced absorption of dietary carbohydrates

measures to prevent colonic diverticulosis and diverticulitis include all of the following except: a. increased whole grain intake b. regular aerobic exercise c. adequate hydration d. refraining from excessive alcohol intake

d. refraining from excessive alcohol intake

measures to prevent colonic diverticulosis and diverticulitis include all of the following except: a. increased whole grain intake b. regular aerobic exercise c. adequate hydration d. refraining from excessive alcohol intake

d. refraining from excessive alcohol intake

a pt who has been txed for hypothyroidism presents for her annual exam. Her TSH is 4.1 (normal = 0.4 - 3.8). She feels well. How should she be managed? a. continue her current dosage of thyroid replacement b. increase her replacement c. decrease her replacement d. repeat the TSH in 2-3 wks

d. repeat the TSH in 2-3 wks

in analyzing synovial fluid, a yellow-green color may indicate which of the following? a. trauma b. gout c. bacterial infection d. rheumatoid arthritis

d. rheumatoid arthritis

a 54 y/o man with an anal fissure responds inadequately to dietary intervention and standard therapy during the past two weeks. Additional txment options include all of the following EXCEPT: a. intraanal nitroglycerine ointment b. botulinum toxicum injection to the internal anal spincter c. surgical spincterotomy d. rubber band ligation of the lesion

d. rubber band ligation of the lesion

the action of which of the following is believed to be most responsible for the sensation of satiety? a. norepinephrine b. epinephrine c. dopamine d. serotonin

d. serotonin

among the preferred first-line pharmacological txment options for pts with PTSD include the use of: a. methylphenidate (ritalin) b. oxazepam (serax) c. lithium carbonate d. sertraline (zoloft)

d. sertraline (zoloft)

colonic diverticulosis most commonly occurs in the walls of the : a. ascending colon b. descending colon c. transverse colon d. sigmoid colon

d. sigmoid colon

the most commonly recommended treatment of Cushing's disease is: a. tapering or ceasing corticosteroid use b. eliminating trigger medications c. antineoplastic tx d. surgical intervention

d. surgical intervention Cushing's syndrome initial treatment is tapering corticosteroids use. Cushing's disease 1st line treatment is transsphenoidal surgery. those who aren't candidates for surgery can receive radiation

what differentiates prostate cancer sxs from BPH sxs? a. urinary frequency,hesitancy, and intermittency are much worse with prostate cancer b. nocturia is worse with BPH c. dribbling and a weak stream are more indicative of BPH d. sxs of prostate cancer in general tend to progress more rapidly than those of BPH

d. sxs of prostate cancer in general tend to progress more rapidly than those of BPH

a 54 y/o man has been recently dxed with HAV infection. You recommend all of the following except: a. eating smaller, more frequent meals to help combat nausea b. avoiding consumption of any alcohol c. reviewing current medication use for consideration of discontinuation d. taking daily acetaminophen to alleviate joint pain

d. taking daily acetaminophen to alleviate joint pain

a 38 y/o man with a recent hx of injection drug use presents with malaise, nausea, fatigue and yellow eyes for the past week. after ordering dxtic tests, you confirm the dx of acute hepatitis B. anticipated lab results include: a. the presence of hepatitis B surface antibody b. neutrophilia c. thrombocytosis d. the presence of HBsAg

d. the presence of HBsAg

a pt who is dx today with diabetes has microalbuminuria. What can be concluded about this finding? a. the pt has diabetic nephropathy b. there is renal damage c. the pt should have a repeat test in one month d. the pt might have been diabetic for a long time before the dx

d. the pt might have been diabetic for a long time before the dx

risk factors for heatstroke include all of the following exept: a. obesity b. use of beta-adrenergic antagonists c. excessive activity d. use of a vasodilator

d. use of a vasodilator

recommended exercises for pts with OA of the hip include all of the following except: a. stretching exercises of the gluteus muscles b. straight-leg raises without weights c. isometric exercises of the iliopsoas and gluteus muscles d. weight-bearing aerobic exercises

d. weight-bearing aerobic exercises

a 65 y/o is dx with an initial episode of gout. It is likely that he: a. will have elevated uric acid levels b. is consuming too much met c. will have involvement in a joint like the hip or shoulder d. will have severe inflammation in a single joint

d. will have severe inflammation in a single joint

which of the following joints is most likely to be affected by osteoarthritis? a.] writs b.] elbows c].meta- carpophalangeal joint d.] distal interphalangeal joint

d.] distal interphalangeal joint

Signs of excessive progestin in BCP

depression fatigue transient HTN breast tenderness decr. menstrual bleeding duration increased appetite

What is the main cause of secondary adrenal insufficiency? What the aftermath? What are other causes of secondary adrenal insufficiency?

diseased pituitary gland pituitary gland can't produce ACTH, no ACTH=no hormones LTU of systemic corticosteroids being abruptly discontinued

Often first line to confirm diagnosis and monitor lower extremity vascular occlusive disease

doppler ultrasonography

__________ulcer, relief of pain when eating

duodenal

Antiulcer therapy

duodenal ulcer: omeprazole every day or lansoprazole for 7 weeks H2 blockers given for 6wks

Regular insulin (Humulin R, Novolin R) *short acting* onset of action =30min-1 hr peak=2-3 hr duration=4-6 hr

duration=4-6 hr

Permanent alveolar damage & loss of elastic recoil resulting in chronic hyperinflation of lungs. Expiratory respiratory phase markedly prolonged.

emphysema

Antiviral for treatment of chronic HBV (3)

entecavi tenofovir lamivudine

As Acute epididymitis progresses, the ipsilateral testis may become involved, swelling so that the two testes cannot be distinguished; this is known as ____________.

epididymo-orchitis

Supporting laboratory findings in PID include elevated __________ or _________level and leukocytosis with neutrophilia.

erythrocyte sedimentation rate or C-reactive protein treatment: Ceftriaxone, 250 mg intramuscularly as a one-time dose, followed by doxycycline, 100 mg bid for 2 weeks with or without metronidazole 500 mg, is likely the most commonly used treatment regimen and is highly effective.

preparation

exhibits some change behaviors but doesn't feel they have the tools to move forward with the change

Clinical findings in ruptured appendix

fever greater than 102°F (>38°C). palpable abdominal mass. marked leukocytosis with total WBC greater than 20,000/mm3.

Intervention in postrenal azotemia

focused on relieving the urinary outflow tract obstruction. When postrenal azotemia is promptly detected, renal function returns to baseline after treatment.

In acute bursitis, the joint range of motion (ROM) is usually _______ (full or decreased).

full

too much iron can cause (3)

gi upset cirrhosis of liver cardiomyopathy

Barlow Test

good for 6 months Flex infants hips and knees to 90 degrees, place thumb along medial thigh, apply pressure in a posterior direction while adducting the femur. This maneuver will force the head out of the socket. evaluates hip dysplasia

Type of strain: results from minimal stretching or small tears in the ligament; mild tenderness and edema, and the patient is able to bear weight.

grade I sprain

Type of strain: results from more severe stretching and tearing of the ligament with moderate pain, edema, tenderness, and ecchymosis. Weight bearing is painful, but the patient can walk.

grade II sprain

Impetigo skin lesions are almost always caused by gram ____ organisms such as ____________ and _____________

gram (+) group A strep and staph aureus

apley is a assessment test for ________ how do you do it?

helps to differentiate between meniscal tears and ligamentous lesions patient prone with testing knee flexed to 90°.|stabilized patient's thigh to table with your knee. [Passively] distract the knee joint & rotate tibia internally and externally slowly. [THEN] -apply a compressive load to knee joint and rotate int/ext to tibia slowly|(+) [TEST]- pain or decreased motion during compression = mensical dysfunctiona|- pain or decreased motion during distraction = ligamentous dysfunction

Hyperthyroidism, Hypothyroidism or Both -tachycardia w/HTN

hyper

-Master gland -Sends signals to anterior and posterior pituitary -Produces oxytocin - uterine stimulation in labor and milk production

hypothalamus

identify the following findings as associated with hyperthyroidism, hypothyroidism, or both: secondary hypertriglyceridemia coarse, dry skin menorrhagia hyporeflexia with a characteristic slow relaxation phase, the "hung-up" reflex coarse hair with tendency to break easily thick dry nails constipation

hypothyroidism

Radioactive iodine in Thyrotoxicosis/Toxic Diffuse Goiter/Graves=

hypothyroidism for life

The nurse is performing a cardiac assessment on a 65-year-old patient 3 days after her myocardial infarction. Heart sounds are normal when she is supine, but when she is sitting and leaning forward, the nurse hears a high-pitched, scratchy sound with the diaphragm of the stethoscope at the apex. It disappears on inspiration. The nurse suspects:

inflammation of the precordium

Vulvovagintis Sx of vulvogaginitis

inflammation of the vulvus and vagina commonly caused by rich, candidiasis, trichomonas: malodorous yellow frothy discharge pruritic, strawberry patches on cervix bv: watery gray fishy smell candidiasis: thick white curd like, erythema pruritus

oligomenorrhea: menorrhagia: metrorrhagia: menometrorrhagia: intermenstrual:

infrequent, irregular polymenorrhea: frequent <18 days excessive bleeding bleed btw cycles prolonged frequent excessive irregular bleeding variable quantity btw cycles

Fat-pad sign found with

injury resulting from straight, outstretched arm falls no fracture visible on X-ray lateral view show anterior and posterior fat pad *suggest presence of occult fracture* do an oblique for further evaluation of occult fracture

What diagnosis? burning sensation or ache w/walking pain noted in calf, hip, or buttock (RELIEVED w/rest) foot pain at rest in unhealthy patient, absent PT pulse (present in healthy person) nail thickening blanching of foot with elevation, poor cap refill and dependent rubor. ( these findings are in person with poorly controlled DM)

lower extremity vascular occlusive disease

a 16 y/o female is dx with primary dysmenorrhea. She has taken over the counter ibuprofen in 800mg increments q8hrs during menses for the past 3 months with minimal relief of sxs. What intervention will provide greatest relief of dysmenorrhea sxs? a. flurbiprofen during menses b. combined oral contraceptives c. daily multivitamin with B12 supplementation d. 30 min of regular exercise daily

menses b. combined oral contraceptives

Raises risk for CV & DM (3 or more for diagnosis) 1. Waist circumference >/= 40 in/101.6 cm in men and >/= 35 inces/88.9 cm in women 2. BP >/= 130/85 3. Triglycerides >/= 150 4. FBG >/= 100 5. HDL <40 men, <50 women

metabolic syndrome?

Labs/diagnostics of diverticulitis

moderalte leukocytosis, elevated ESR, stool heme, inflamed mucosa CT or abdominal films

a rare type of thyroid cancer that should be consider in person with a rapidly growing goiter is:

non-Hodgkin's lymphome pg 855

Who commonly has Slipped Capital Femoral Epiphysis (SCFE)

older overweight/sedentary during growth/before menses

Rapid acting insulin covers? (in reference to eating)

one meal at a time

The most common risks for ___________ are biliary tract disease including gallstones (45%), excessive alcohol use (35%), and elevated triglyceride levels and idiopathic causes (20% combined).

pancreatitis

If amylase and lipase levels are initially three times the upper limit of normal and gut perforation and infarction have been ruled out, these lab values clinch the diagnosis of _____________.

pancreatitis.

With acute Hepatitis C, what medication should be given to decrease the risk of chronic hepatitis?

pegylated interferon and ribavirin (in the acute phase) IM injection This is a long-acting drug that should be given once per week in a dose of 180mg for 48 weeks [ribavirin] divided doses of 1000mg per day for those weighing less than 165 lbs and 1200mg per day in those weighing more than 165 lb

What are the main treatment option for Hep B? What are the 2 additional options?

pegylated interferon with an antiviral (entecavir, adefovir, lamivudine) secondary: telbivudine and tenofovir

Cauda Equina Syndrome..what comes to your mind

sacral nerve compression urine retention/incontinence bowel incontinence billet leg numbness/weakness most common cause: *bulging disc* *surgical emergency>ER* ****spinal decompression***

gallstones have intermittent discomfort as a result of this condition. The pain is described as being of sudden onset, usually post prandial, particularly within 1 hour of a fatty meal, in the abdominal right upper quadrant or epigastrium, occasionally radiating to the tip of the right _____________.

scapula

Diffuse maculopapular rash involving palms and soles, generalized lymphadenopathy, low-grade fever, malaise, arthralgias and myalgia, headache

secondary syphillis

What lab is usually elevated in acute pancreatitis? What labs are significant for diagnostic and in what way?

serum amylase serum amylase and lipase... if they are initially 3x the upper limit of normal, in the absence of gut perforation/infarct has been ruled out, THEN these values clinch for a dx of pancreatitis Fitzgerald pg 187

Treatment of pancreatic pseudocysts?

smaller than 10mm - imaged by CT after 1 year and less frequent if deemed stable larger than 10mm - endoscopic ultrasound to search for features of concern

Out-pt management PUD: PPIs

take 30 min before meal Lansoprazole (prevacid), raberprazole (Aciphex), pantoprazole (protonix), esmoprazole (nexium).

________ is the sensation that something is in the recta vault when it is empty "i moved my bowels but i feel like there is something there". R/t to inflammation

tenesmus

The parathyroid hormone _______________is an anabolic agent that can reduce the risk of vertebral fractures by 65% in patients with osteoporosis. This treatment is typically reserved for women with very low bone density or who have had a prior fracture. Treatment with this medication should be limited to a 2-year duration because of a potentially increased risk of osteosarcoma.

teriparatide (Forteo)

______syphilis Gumma (granulomatous lesions involving skin, mucous membranes, bone), aortic insufficiency, aortic aneurysm, Argyll Robertson pupil, seizures

tertiary

Purpose of Spurling test. Positive finding indicative of?

test for cervical nerve root compression

The use of finasteride (Proscar) and dutasteride (Avodart), 5-alpha-reductase inhibitors that block the conversion of ______ to dihydrotestosterone, helps to reduce the size of the prostate and ameliorate symptoms.

testosterone

If girl wants to know if she will get any taller...remember

that 95% of height is completed by menarche

According to Ottawa ankle rules, the examiner should order x-rays of the foot or ankle when:

the patient has ankle or midfoot pain/ bone tenderness at the base of the 5th metatarsal, or is unable to bear weight for four steps when examined.

Shortening of the sternocleidomastoid muscle neck pulled to side vision problems later

torticollis A condition where the head is pulled to one side by a painful contraction of the neck muscles (wryneck)

highest risk area of cervix where stratified squamous epithelial tissue intersects with columnar epithelial Is well-outside external os in young women and migrates in with cervical disruption (pregnancy)

transformation zone

In IBS Low-dose __________________or ________________ use can be helpful in altering the gut pain threshold, resulting in less abdominal pain; the anticholinergic effects of the __________________can help with limiting stool frequency, but may worsen constipation (so not recommended in IBS-C.)

tricyclic antidepressant or selective serotonin reuptake inhibitor

T/F: In patients with chronic bacterial prostatitis, irritative voiding symptoms, low back and perineal pain, and a history of urinary tract infection are typically reported.

true

T/F: Infection with a gram-negative rod such as E. coli and Pseudomonas species usually causes acute bacterial prostatitis in older men.

true

T/F: The use of certain medications including estrogen, progesterone/progestins, theophylline, calcium channel blockers, and nicotine; can result in a decrease in lower esophageal sphincter pressure and worsen GERD,

true

T/F: When prescribing antihypertensive therapy for a man with BPH and hypertension, the NP considers that an alpha 1 antagonist should not be used as a solo or first-line therapeutic agent.

true

Difference in ulcerative colitis and chrons

ulcerative colitis involves rectum and colon chrons involves malabsorption of upper bowel

with an 8am dose of the following insulin forms, followed by an inadequate dietary intake and/or excessive energy use, at approximately what time would hypoglycemia be most likely to occur with Insulin glargine (Lantus)?

unlikely to occur because there is no peak

Physical exam in pancreatic cancer

upper abdomen tender without guarding, rigidity abdominal distention absence of bowel sound if ileum fever, tachycardia, hypotension pallor, cool clammy skin, mild jaundice occasionally, upper and mass is palpable due to inflamed pancreas or pseudocyst

When seen with infectious diarrhea, the disease is found equally in both genders. When this condition is seen with _____________ (which male GU disease) there is a male predominance of 9:1, with most being HLA-B27 positive

urethritis

When does urine leak with urge incontinence?

urgency, involuntary urine loss, nocturia, frequency, "overactive bladder"

_______________ is the standard for diagnosing genital herpes, which requires a collection of a sample from a sore. PCR can also be used to test for the presence of viral DNA or RNA and may allow for more rapid and accurate results. Serological approaches can detect for the presence of antibodies in the blood. In symptomatic patients, the use of direct and indirect assays can differentiate between a new infection and a newly-recognized older infection. A positive virological test with a negative serological test would suggest a new infection. Positive results for both tests would indicate a recurrent infection.

viral culture

ALT > AST hepatitis or ETOH/drugs?

viral hepatitis

Which vitamin is an inhibitor of abnormal cellular growth, this vitamin is needed to help with cell differentiation and minimizing abnormal cell proliferation, a key step in cancer development. It is a stimulator of insulin secretion and plays a role in the maintenance of normoglycemia. micronutrient plays an important role as an immunomodulator and acts as a renin producer, therefore contributing to blood pressure control.

vitamin d

how do you test for Trousseau's sign?

with BP cuff 20mmHg above the normal systolic, inflate, hold for 3 minutes pg 830

Supplies parasympathetic fibers to the viscera of the chest, abdomen; motor fibers to the pharynx, larynx; sensory fibers to the ear, meninges, viscera

• CN X: Vagus

o Supplies motor fibers to the muscles of the tongue. o Observe for fasciculations when patient sticks tongue out.

• CN XII: Hypoglossal

Migraine and Oral Contraception consideration

• Migraine with and without aura increases relative risk of ischemic stroke • Migraine with aura, any age, OC risk unacceptable • Women < 35 years who have migraine without aura, no other risk factors for stroke, low dose estrogen ≤ 50 mcg OK (ACOG); (WHO: ≤ 35 mcg estrogen is ok) • Women > 35 years who have migraine + OC; risks outweigh benefits

Common Migraine Triggers

• Stress • Menses • Skipping meals • Weather • Sleep disturbances • Odors • Light, EtOH, Smoking • Foods

Behavioral disturbances of .... MAJOR NEUROCOGNITIVE DISORDER (FORMERLY DEMENTIA)•

• Wandering • Restlessness • Agitation • Aggression • Sleep wake cycle disturbances • Apathy • Difficulty concentrating • Delusions • Hallucination

Para or "P"

■ Para or "P": The number of births more than 20 weeks no matter what the outcome (including miscarriages, stillborn). Twins and multiples are counted as one pregnancy.

Pregnany HEALTH EDUCATION

■ Prenatal vitamins with 400 mcg of folic acid daily (start 3 months before conception). ■ Always wear seatbelt (lap belt below uterine fundus). ■ Avoid soft cheeses (blue cheese, brie), uncooked meats, raw milk (listeria bacteria). ■ Sex is safe except during vaginal bleeding, incompetent cervix, placenta previa, or preterm labor. ■ Cat litter or raw beef can cause toxoplasmosis (congenital infection). ■ No raw shellfi sh or raw oysters (vibrio vulnifi cus infection). ■ Careful with cold cuts, uncooked hotdogs, and deli meat (listeria bacteria). Pregnant women are 20 times more likely to become infected with listeria monocytogenes. ■ Smoking (IUGR), alcohol (fetal alcohol syndrome). ■ Regular coffee (8 ounces/day) is okay. No excessive caffeine (premature labor). ■ No hot tubs, saunas, or excessive heat.


Kaugnay na mga set ng pag-aaral

MGMT Exam #2 3013 Fundamentals of Managment Oklahoma State

View Set

Chapter 7- E Business and E Commerce

View Set

Elsevier adaptive quizzing- Professionalism

View Set

Physics 1310 Exam 2 - gregory spencer

View Set

Chapter 8: lesson 3 laws and rules

View Set